SAT test 6

126
Help | Profile | My Organizer | My Bookmarks | Logout Answers and Explanations Test Sections Section 1 Section 2 Section 4 Section 5 Section 6 Section 7 Section 8 Section 9 Section 10 Back to Score Report Essay Online - Practice Test #6 These sample essays were originally handwritten by students but are shown typed here for ease of reading. The essays are displayed exactly as students wrote them, without any corrections to spelling, punctuation, or syntax. One handwritten sample essay is provided to illustrate the need for legible and clear handwriting. Exemplars: Essay Prompt Think carefully about the issue presented in the following excerpt and the assignment below. People who like to think of themselves as tough-minded and realistic tend to take it for granted that human nature is selfishand that life is a struggle in which only the fittest may survive. According to this view, the basic law by which people must live is the law of the jungle. The fittestare those people who can bring to the struggle superior force, superior cunning, and superior ruthlessness. Adapted from S.I. Hayakawa, Language in Thought and Action Do people have to be highly competitive in order to succeed? Plan and write an essay in which you develop your point of view on this issue. Support your position with reasoning and examples taken from your reading, studies, experience, or observations. Back to Score Report Copyright © 2006 The College Board. All rights reserved. Privacy Policy Terms of Use Contact Us 页码1/1 The Official SAT Online Course 2006-11-12 file://E:\新建文件\f2.htm UnRegistered

description

SAT online course test 6 pdf

Transcript of SAT test 6

Page 1: SAT test 6

严禁用于商业用途!

Help | Profile | My Organizer | My Bookmarks | Logout

Answers and Explanations

Test Sections

Section 1

Section 2

Section 4

Section 5

Section 6

Section 7

Section 8

Section 9

Section 10

Back to Score Report  

Essay     Online - Practice Test #6

These sample essays were originally handwritten by students but are shown typed here for ease of reading. The essays are displayed exactly as students wrote them, without any corrections to spelling, punctuation, or syntax. One handwritten sample essay is provided to illustrate the need for legible and clear handwriting.

Exemplars:

Essay Prompt

Think carefully about the issue presented in the following excerpt and the assignment below.

People who like to think of themselves as tough-minded and realistic tend to take it for granted that human nature is “selfish” and that life is a struggle in which only the fittest may survive. According to this view, the basic law by which people must live is the law of the jungle. The “fittest” are those people who can bring to the struggle superior force, superior cunning, and superior ruthlessness.

Adapted from S.I. Hayakawa, Language in Thought and Action

Do people have to be highly competitive in order to succeed? Plan and write an essay in which you develop your point of view on this issue. Support your position with reasoning and examples taken from your reading, studies, experience, or observations.

Back to Score Report  

Copyright © 2006 The College Board. All rights reserved. Privacy Policy Terms of Use Contact Us

 

页码,1/1The Official SAT Online Course

2006-11-12file://E:\新建文件夹\f2.htm

UnRe

gistered

Page 2: SAT test 6

严禁用于商业用途!

Help | Profile | My Organizer | My Bookmarks | Logout

Answers and Explanations

Test Sections

Section 1

Section 2

Section 4

Section 5

Section 6

Section 7

Section 8

Section 9

Section 10

Back to Score Report  

View Answers and Explanations     Online - Practice Test #6

1 If which of the following could be a value of x ?

ANSWERS AND EXPLANATIONS Explanation for Correct Answer A : 

Choice (A) is correct. If , then . The solutions to this

equation are and . So could be .

Explanation for Incorrect Answer B :  Choice (B) is not correct. The value of could not be , because

.

Explanation for Incorrect Answer C :  Choice (C) is not correct. The value of could not be , because

.

Explanation for Incorrect Answer D :  Choice (D) is not correct. The value of could not be , because

.

Explanation for Incorrect Answer E :  Choice (E) is not correct. The value of could not be , because

.

(A)

(B)

(C)

(D)

(E)

2The length of a rectangular rug is 2 feet more than its width. If the length of the rug is 8 feet, what is the area of the rug in square feet?

ANSWERS AND EXPLANATIONS Explanation for Correct Answer B :  Choice (B) is correct. The area of the rug is equal to the length of the rug times the width of the rug. Since its length is feet, and its length is feet more than its

(A) 16

(B) 48

(C) 66

(D) 80

(E) 96

页码,1/15The Official SAT Online Course

2006-11-12file://E:\新建文件夹\f3.htm

UnRe

gistered

Page 3: SAT test 6

严禁用于商业用途!

width, the width of the rug is . So the area of the rug is feet times feet,

which is square feet.

Explanation for Incorrect Answer A :  Choice (A) is not correct. See the explanation for the correct response (B).

Explanation for Incorrect Answer C :  Choice (C) is not correct. See the explanation for the correct response (B).

Explanation for Incorrect Answer D :  Choice (D) is not correct. If the width of the rug were feet more than its length of

feet, then the width would be feet, and the area of the rug would be square feet. However, this answer is incorrect, because the width of the rug is feet less than the length.

Explanation for Incorrect Answer E :  Choice (E) is not correct. See the explanation for the correct response (B).

3 If and what is the value of 2r?

ANSWERS AND EXPLANATIONS Explanation for Correct Answer E :  Choice (E) is correct. If and , then . Since , it

follows that .

Explanation for Incorrect Answer A :  Choice (A) is not correct. The value of cannot be , because , so

.

Explanation for Incorrect Answer B :  Choice (B) is not correct. The value of cannot be , because , so

.

Explanation for Incorrect Answer C :  Choice (C) is not correct. The value of cannot be , because and

, so . The value of .

Explanation for Incorrect Answer D :  Choice (D) is not correct. The value of cannot be , because and

, so .

(A) 2

(B) 4

(C) 6

(D) 8

(E) 12

4Some integers in set X are even.

If the statement above is true, which of the following must also be true?

(A) If an integer is even, it is in set X.

If an integer is odd, it is in set X.

页码,2/15The Official SAT Online Course

2006-11-12file://E:\新建文件夹\f3.htm

UnRe

gistered

Page 4: SAT test 6

严禁用于商业用途!

ANSWERS AND EXPLANATIONS Explanation for Correct Answer E :  Choice (E) is correct. If some integers in set are even, then those even integers are members of set that are not odd. So not all integers in set are odd.

Explanation for Incorrect Answer A :  Choice (A) is not correct. Although it is true that some integers in set are even, it is not necessarily true that every even integer belongs to set . So there may be some even integers that are not in set .

Explanation for Incorrect Answer B :  Choice (B) is not correct. Although it is true that some integers in set are even, it is not necessarily true that contains any odd integers. So there may be odd integers that are not in set .

Explanation for Incorrect Answer C :  Choice (C) is not correct. Although it is true that some integers in set are even, it is not necessarily true that all integers in set are even.

Explanation for Incorrect Answer D :  Choice (D) is not correct. Since some integers in set are even, it cannot be true that all integers in set are odd.

(B)

(C) All integers in set X are even.

(D) All integers in set X are odd.

(E) Not all integers in set X are odd.

5 A triangle has a perimeter of 13 and one side of length 3. If the lengths of the other two sides are equal, what is the length of each of them?

ANSWERS AND EXPLANATIONS Explanation for Correct Answer B :  Choice (B) is correct. Since the perimeter of the triangle is and one side has

length , it follows that the sum of the lengths of the other two sides is . Since

these other two sides are of equal length and the sum of their lengths is , it

follows that the length of each of them is .

Explanation for Incorrect Answer A :  Choice (A) is not correct. If the triangle had two sides of length and one side of length , the perimeter of the triangle would be . But the perimeter

is , not . Thus, the two equal sides cannot have length .

Explanation for Incorrect Answer C :  Choice (C) is not correct. If the triangle had two sides of length and one side of

length , the perimeter of the triangle would be . But the

perimeter is , not . Thus, the two equal sides cannot have length .

Explanation for Incorrect Answer D :  Choice (D) is not correct. If the triangle had two sides of length and one side of

(A) 4

(B) 5

(C) 6

(D) 7

(E) 8

页码,3/15The Official SAT Online Course

2006-11-12file://E:\新建文件夹\f3.htm

UnRe

gistered

Page 5: SAT test 6

严禁用于商业用途!

length , the perimeter of the triangle would be . But the perimeter

is , not . Thus, the two equal sides cannot have length .

Explanation for Incorrect Answer E :  Choice (E) is not correct. If the triangle had two sides of length and one side of

length , the perimeter of the triangle would be . But the perimeter

is , not . Thus, the two equal sides cannot have length .

6

The grid above shows the number of hours worked last week by 12 students of various ages at after-school jobs. Which of the following is true, according to this grid?

ANSWERS AND EXPLANATIONS Explanation for Correct Answer D :  Choice (D) is correct. According to the information in the grid, of the year

olds worked at least hours each and of the year olds worked at least

hours each each. So more year olds than year olds worked at least hours each.

Explanation for Incorrect Answer A :  Choice (A) is not correct. According to the information in the grid, only of the students worked more than hours each. So it is not true that half of the students worked more than hours each.

Explanation for Incorrect Answer B :  Choice (B) is not correct. According to the information in the grid, none of the students worked exactly hours. So it is not true that one student worked

exactly hours.

Explanation for Incorrect Answer C :  Choice (C) is not correct. According to the information in the grid, the three

year olds each worked hours or less. So it is not true that one year old worked more than hours.

Explanation for Incorrect Answer E :  Choice (E) is not correct. According to the grid, there was only one student under

years old. So most of the students were not under years old.

(A) Half of the students worked more than 12 hours each.

(B) One student worked exactly 15 hours.

(C) One 16 year old worked more than 11 hours.

(D) More 18 year olds than 17 year olds worked at least 13 hours each.

(E) Most of the students were under 16 years old.

7 Squaring the product of and 5 gives the same result as squaring the sum of and 5. Which of the following equations could be used to find all possible values of

页码,4/15The Official SAT Online Course

2006-11-12file://E:\新建文件夹\f3.htm

UnRe

gistered

Page 6: SAT test 6

严禁用于商业用途!

ANSWERS AND EXPLANATIONS Explanation for Correct Answer D : 

Choice (D) is correct. The square of the product of and is , and the

square of the sum of and is . If squaring the product of and

gives the same result as squaring the sum of and , then the equation

is true. Solving this equation would give all possible values of

.

Explanation for Incorrect Answer A : 

Choice (A) is not correct. The square of the sum of and is , but the

expression is the product of and the square of , not the square of the

product of and . Thus, solving the equation will not give the

correct values of .

Explanation for Incorrect Answer B : 

Choice (B) is not correct. The square of the product of and is , but the

expression is the sum of the square of and the square of , not the

square of the sum of and . Thus, solving for will not give

the correct values of .

Explanation for Incorrect Answer C : 

Choice (C) is not correct. The expression is the product of the square of

and , not the square of the product of and . The expression is the

sum of the square of and the square of , not the square of the sum of and

. Thus, solving will not give the correct values of .

Explanation for Incorrect Answer E : 

Choice (E) is not correct. The expression is the product of and the square

of , not the square of the product of and . The expression is the

sum of the square of and the square of , not the square of the sum of and

. Thus, solving for will not give the correct values of .

(A)

(B)

(C)

(D)

(E)

8 If as many 7-inch pieces of wire as possible are cut from a wire that is 3 feet long, what is the total length of the wire that is left over? (12 inches = 1 foot)

ANSWERS AND EXPLANATIONS

(A) 1 inch

(B) 2 inches

(C) 3 inches

(D) 4 inches

(E) 5 inches

页码,5/15The Official SAT Online Course

2006-11-12file://E:\新建文件夹\f3.htm

UnRe

gistered

Page 7: SAT test 6

严禁用于商业用途!

Explanation for Correct Answer A :  Choice (A) is correct. Since inches is equal to foot, a piece of wire that is

feet long is inches long. The greatest number of -inch pieces of wire that can

be cut from the -inch-long wire is pieces. The total length of these pieces

of wire is inches. So the piece of wire left after cutting the five -inch pieces is inch long.

Explanation for Incorrect Answer B :  Choice (B) is not correct. The total length of wire left after cutting -inch pieces of wire from a -inch piece of wire cannot be inches long. If it were inches

long, then the -inch pieces together would measure inches. But

does not divide evenly.

Explanation for Incorrect Answer C :  Choice (C) is not correct. The total length of wire left after cutting -inch pieces of wire from a -inch piece of wire cannot be inches long. If it were inches

long, then the -inch pieces together would measure inches. But

does not divide evenly.

Explanation for Incorrect Answer D :  Choice (D) is not correct. The total length of wire left after cutting -inch pieces of wire from a -inch piece of wire cannot be inches long. If it were inches

long, then the -inch pieces together would measure inches. But

does not divide evenly.

Explanation for Incorrect Answer E :  Choice (E) is not correct. The total length of wire left after cutting -inch pieces of wire from a -inch piece of wire cannot be inches long. If it were inches

long, then the -inch pieces together would measure inches. But

does not divide evenly.

9

Which of the lettered points in the figure above has coordinates such that

ANSWERS AND EXPLANATIONS Explanation for Correct Answer B : 

Choice (B) is correct. The coordinates of point are . Since

, point has coordinates such that .

(A) A

(B) B

(C) C

(D) D

(E) E

页码,6/15The Official SAT Online Course

2006-11-12file://E:\新建文件夹\f3.htm

UnRe

gistered

Page 8: SAT test 6

严禁用于商业用途!

Explanation for Incorrect Answer A : 

Choice (A) is not correct. The coordinates of point are . Since

, point does not have coordinates such that

.

Explanation for Incorrect Answer C : 

Choice (C) is not correct. The coordinates of point are . Since

, point does not have coordinates such that

.

Explanation for Incorrect Answer D : 

Choice (D) is not correct. The coordinates of point are . Since

, point does not have coordinates such that

.

Explanation for Incorrect Answer E : 

Choice (E) is not correct. The coordinates of point are . Since

, point does not have coordinates such that

.

10

If n is a member of both set A and set B above, which of the following must be true?

I. n is an integer.

II. 4n is an integer.

III.

ANSWERS AND EXPLANATIONS Explanation for Correct Answer A :  Choice (A) is correct. If is a member of both set and set , then must be

, , or . If , then is not an integer, so is not necessarily true. If

, then , which is not an integer; is not necessarily true. If

, then , so is not necessarily true. Therefore, none of the three statements must be true.

(A) None

(B) II only

(C) I and II only

(D) I and III only

(E) I, II, and III

页码,7/15The Official SAT Online Course

2006-11-12file://E:\新建文件夹\f3.htm

UnRe

gistered

Page 9: SAT test 6

严禁用于商业用途!

Explanation for Incorrect Answer B :  Choice (B) is not correct. If is a member of both set and set , then must

be , , or . If , then is not an integer, so is not

necessarily true.

Explanation for Incorrect Answer C :  Choice (C) is not correct. If is a member of both set and set , then must

be , , or . If , then is not an integer, so is not necessarily true.

If , then , which is not an integer; so is not necessarily true.

Neither statement nor must be true.

Explanation for Incorrect Answer D :  Choice (D) is not correct. If is a member of both set and set , then

must be , , or . If , then is not an integer, so is not necessarily

true. If , then , so is not necessarily true. Neither statement nor must be true.

Explanation for Incorrect Answer E :  Choice (E) is not correct. If is a member of both set and set , then must

be , , or . If , then is not an integer, so is not necessarily true.

If , then , which is not an integer; so is not necessarily true. If

, then , so is not necessarily true. Therefore, none of the three statements must be true.

11If y is directly proportional to x and if when what is the value of y

when

ANSWERS AND EXPLANATIONS Explanation for Correct Answer E :  Choice (E) is correct. If is directly proportional to , then there is a constant

such that . If when , then , so and

. When , the value of is .

Explanation for Incorrect Answer A :  Choice (A) is not correct. If is directly proportional to , then there is a

constant such that . If when , then , so

. This is not the answer to the problem, because you must find the value of

when . This is .

Explanation for Incorrect Answer B : 

(A)

(B)

(C) 23

(D) 27

(E) 30

页码,8/15The Official SAT Online Course

2006-11-12file://E:\新建文件夹\f3.htm

UnRe

gistered

Page 10: SAT test 6

严禁用于商业用途!

Choice (B) is not correct. If is directly proportional to , then there is a constant

such that . If when , then , so and

. When , the value of cannot be , because

.

Explanation for Incorrect Answer C :  Choice (C) is not correct. If is directly proportional to , then there is a constant

such that . If when , then , so and

. When , the value of cannot be , because

.

Explanation for Incorrect Answer D :  Choice (D) is not correct. If is directly proportional to , then there is a constant

such that . If when , then , so and

. When , the value of cannot be , because

.

12 The nth term of a sequence is defined to be The 50th term is how much greater than the 45th term?

ANSWERS AND EXPLANATIONS Explanation for Correct Answer C :  Choice (C) is correct. If the th term of a sequence is defined to be , then

the th term of the sequence is . The th term of the

sequence is . So the th term of the sequence is greater than

the th term by .

Explanation for Incorrect Answer A :  Choice (A) is not correct. The number is greater than the number , but

the th term of the sequence, , is greater than the th term of the

sequence, , by .

Explanation for Incorrect Answer B :  Choice (B) is not correct. The th term of the sequence is , and the th

term of the sequence is . The difference between these numbers is not .

Explanation for Incorrect Answer D :  Choice (D) is not correct. The th term of the sequence is , and the th

term of the sequence is . The difference between these numbers is not .

Explanation for Incorrect Answer E :  Choice (E) is not correct. The th term of the sequence is , and the th

term of the sequence is . The difference between these numbers is not .

(A) 5

(B) 16

(C) 20

(D) 23

(E) 24

页码,9/15The Official SAT Online Course

2006-11-12file://E:\新建文件夹\f3.htm

UnRe

gistered

Page 11: SAT test 6

严禁用于商业用途!

13

Which of the rectangular solids shown above has a volume closest to the volume of a right circular cylinder with radius 2 and height 4?

ANSWERS AND EXPLANATIONS Explanation for Correct Answer E :  Choice (E) is correct. The volume of a right circular cylinder with radius and

height is . Since is close to , the volume of

rectangular solid , which is , is slightly less than , which is

about . Solids and , with respective volumes of and are too

small; solids and , with respective volumes of and , are too large. So

the volume of the cylinder is closest to the volume of solid .

Explanation for Incorrect Answer A :  Choice (A) is not correct. The volume of the cylinder is about , which is very

close to the volume of solid , which is . Solid , with a volume of , is too small.

Explanation for Incorrect Answer B :  Choice (B) is not correct. The volume of the cylinder is about , which is very

close to the volume of solid , which is . Solid , with a volume of , is too small.

Explanation for Incorrect Answer C :  Choice (C) is not correct. The volume of the cylinder is about , which is very

close to the volume of solid , which is . Solid , with a volume of , is too large.

Explanation for Incorrect Answer D :  Choice (D) is not correct. The volume of the cylinder is about , which is very

close to the volume of solid , which is . Solid , with a volume of , is too large.

(A) A

(B) B

(C) C

(D) D

(E) E

14

If x is a negative integer, what is the ordering of j, k, and m from least to greatest?

(A)

(B)

(C)

(D)

(E)

页码,10/15The Official SAT Online Course

2006-11-12file://E:\新建文件夹\f3.htm

UnRe

gistered

Page 12: SAT test 6

严禁用于商业用途!

ANSWERS AND EXPLANATIONS Explanation for Correct Answer D : 

Choice (D) is correct. If is a negative integer, then is negative

because is negative. Since and are positive,

it follows that and . Since is negative, it follows that

, so . Therefore,

. Putting the inequalities and

together gives .

Explanation for Incorrect Answer A :  Choice (A) is not correct. If is a negative integer, then , since is

negative and is positive. So the inequality cannot be true.

Explanation for Incorrect Answer B : 

Choice (B) is not correct. If is a negative integer, then , since is

negative and is positive. So the inequality cannot be true.

Explanation for Incorrect Answer C :  Choice (C) is not correct. If is a negative integer, then , since is

negative and is positive. So the inequality cannot be true.

Explanation for Incorrect Answer E : 

Choice (E) is not correct. If is a negative integer, then , so

. Therefore, . So the inequality

cannot be true.

15 Flour, water, and salt are mixed by weight in the ratio of 5:4:1, respectively, to produce a certain type of dough. In order to make 5 pounds of this dough, what weight of salt, in pounds, is required?

ANSWERS AND EXPLANATIONS Explanation for Correct Answer B :  Choice (B) is correct. The flour, water, and salt are mixed by weight in the ratio

, respectively. So if it requires pounds of salt to make pounds of this

dough, then ; so , and .

Explanation for Incorrect Answer A :  Choice (A) is not correct. See the explanation for the correct response (B).

Explanation for Incorrect Answer C :  Choice (C) is not correct. See the explanation for the correct response (B).

Explanation for Incorrect Answer D : 

(A)

(B)

(C)

(D) 1

(E) 2

页码,11/15The Official SAT Online Course

2006-11-12file://E:\新建文件夹\f3.htm

UnRe

gistered

Page 13: SAT test 6

严禁用于商业用途!

Choice (D) is not correct. See the explanation for the correct response (B).

Explanation for Incorrect Answer E : 

Choice (E) is not correct. As explained for the correct response (B),

pounds of water are required to make pounds of the dough. But the question asks for the number of pounds of salt, not of water, needed to make the dough.

16

In rectangle ABCD above, the area of the shaded region is given by If the area

of the shaded region is what is the total area, to the nearest whole number, of the unshaded regions of rectangle ABCD?

ANSWERS AND EXPLANATIONS Explanation for Correct Answer B :  Choice (B) is correct. The total area of the unshaded regions of rectangle is equal to the area of the entire rectangle minus the area of the shaded region. Since

the area of the shaded region is , it follows that . So the area

of the rectangle is , and the area of the shaded region is . Thus, the total

area of the unshaded regions is , which is approximately .

Explanation for Incorrect Answer A :  Choice (A) is not correct. The total area of the unshaded regions is , which

is approximately . The answer of is too small.

Explanation for Incorrect Answer C :  Choice (C) is not correct. The total area of the unshaded regions is , which

is approximately . The answer of is too large.

Explanation for Incorrect Answer D :  Choice (D) is not correct. The total area of the unshaded regions is , which

is approximately . The answer of is too large.

Explanation for Incorrect Answer E :  Choice (E) is not correct. The total area of the unshaded regions is , which

is approximately . The answer of is too large.

(A) 4

(B) 6

(C) 8

(D) 9

(E) 10

17 The city library donated some children’s books to Mr. Clark’s first-grade class. If each student takes 4 books, there will be 20 books left. If 3 students do not take a book and the rest of the students take 5 books each, there will be no books left. How many books were donated to the class?

页码,12/15The Official SAT Online Course

2006-11-12file://E:\新建文件夹\f3.htm

UnRe

gistered

Page 14: SAT test 6

严禁用于商业用途!

ANSWERS AND EXPLANATIONS Explanation for Correct Answer C :  Choice (C) is correct. If there are students in the class and if books will be

left after each student takes books, then the number of books is . If

students do not take a book and the rest take books each, there will be no books

left. So the number of books is also given by . So the number of books is

given by both and . Solving the equation

for gives . So the number of books is .

Explanation for Incorrect Answer A :  Choice (A) is not correct. See the explanation for the correct response (C).

Explanation for Incorrect Answer B :  Choice (B) is not correct. See the explanation for the correct response (C).

Explanation for Incorrect Answer D :  Choice (D) is not correct. See the explanation for the correct response (C).

Explanation for Incorrect Answer E :  Choice (E) is not correct. See the explanation for the correct response (C).

(A) 120

(B) 140

(C) 160

(D) 175

(E) 185

18

In the figure above, if line has a slope of what is the y-intercept of

ANSWERS AND EXPLANATIONS Explanation for Correct Answer D : 

Choice (D) is correct. From the figure, you can see that the point lies on line

. The slope of line is , so the equation of line is , where

is the -intercept of . Since lies on , it follows that .

Solving this equation for gives .

(A) 7

(B) 8

(C) 9

(D) 10

(E) 12

页码,13/15The Official SAT Online Course

2006-11-12file://E:\新建文件夹\f3.htm

UnRe

gistered

Page 15: SAT test 6

严禁用于商业用途!

Explanation for Incorrect Answer A :  Choice (A) is not correct. Line has slope ; if the -intercept of were ,

then would have the equation . But then the point would not

be on .

Explanation for Incorrect Answer B :  Choice (B) is not correct. Line has slope ; if the -intercept of were ,

then would have the equation . But then the point would not

be on .

Explanation for Incorrect Answer C :  Choice (C) is not correct. Line has slope ; if the -intercept of were ,

then would have the equation . But then the point would not

be on .

Explanation for Incorrect Answer E :  Choice (E) is not correct. Line has slope ; if the -intercept of were ,

then would have the equation . But then the point would

not be on .

19 For all positive integers w and y, where let the operation be defined by

For how many positive integers w is equal to 4 ?

ANSWERS AND EXPLANATIONS Explanation for Correct Answer E :  Choice (E) is correct. If is defined for all positive integers and , where

, to be , then

. So for all positive

integers greater than . Therefore, for infinitely many positive integers , which is more than .

Explanation for Incorrect Answer A :  Choice (A) is not correct. See the explanation for the correct response (E).

Explanation for Incorrect Answer B :  Choice (B) is not correct. See the explanation for the correct response (E).

Explanation for Incorrect Answer C :  Choice (C) is not correct. See the explanation for the correct response (E).

Explanation for Incorrect Answer D :  Choice (D) is not correct. See the explanation for the correct response (E).

(A) None

(B) One

(C) Two

(D) Four

(E) More than four

20

页码,14/15The Official SAT Online Course

2006-11-12file://E:\新建文件夹\f3.htm

UnRe

gistered

Page 16: SAT test 6

严禁用于商业用途!

The figure above represents four offices that will be assigned randomly to four employees, one employee per office. If Karen and Tina are two of the four employees, what is the probability that each will be assigned an office indicated with an X ?

ANSWERS AND EXPLANATIONS Explanation for Correct Answer C :  Choice (C) is correct. The probability that Karen and Tina each will be assigned an office indicated with an X is the number of ways this can happen divided by the total number of ways of assigning the four offices to the four employees. If Karen has the leftmost office and Tina has the rightmost office, there are ways to assign the other two offices. Similarly, if Tina has the leftmost office and Karen has the rightmost office, there are ways to assign the other two offices. Thus, there are ways to assign Karen and Tina offices marked with an X. Now count the total number of ways of assigning the four offices: there are ways to assign the first office, leaving ways to assign the second, leaving ways to assign the

third, leaving way to assign the fourth; this is ways. So the probability that Karen and Tina will each be assigned an office with an X is

.

Explanation for Incorrect Answer A :  Choice (A) is not correct. See the explanation for the correct response (C).

Explanation for Incorrect Answer B :  Choice (B) is not correct. This is the probability that Karen will be assigned the leftmost office and Tina will be assigned the rightmost office. But Karen could be assigned the rightmost office and Tina the leftmost office. See the explanation for the correct response (C).

Explanation for Incorrect Answer D :  Choice (D) is not correct. See the explanation for the correct response (C).

Explanation for Incorrect Answer E :  Choice (E) is not correct. See the explanation for the correct response (C).

(A)

(B)

(C)

(D)

(E)

    

Back to Score Report  

Copyright © 2006 The College Board. All rights reserved. Privacy Policy Terms of Use Contact Us

 

页码,15/15The Official SAT Online Course

2006-11-12file://E:\新建文件夹\f3.htm

UnRe

gistered

Page 17: SAT test 6

严禁用于商业用途!

Help | Profile | My Organizer | My Bookmarks | Logout

Answers and Explanations

Test Sections

Section 1

Section 2

Section 4

Section 5

Section 6

Section 7

Section 8

Section 9

Section 10

Back to Score Report  

View Answers and Explanations     Online - Practice Test #6

1 Edmund White is a ------- author: he has written novels, essays, short stories, a travel book, and a biography.

ANSWERS AND EXPLANATIONS Explanation for Correct Answer D :  Choice (D) is correct. Someone who is "versatile" embraces a variety of subjects, fields, or skills. If one were to insert this term into the text, the sentence would read "Edmund White is a versatile author: he has written novels, essays, short stories, a travel book, and a biography." The colon indicates that the information listed in the second part of the sentence will support the claim made in the first part. A writer who dabbles in so many genres can certainly be described as "versatile."

Explanation for Incorrect Answer A :  Choice (A) is incorrect. Someone who is "demonstrative" is given to open expressions of emotion. If one were to insert this term into the text, the sentence would read "Edmund White is a demonstrative author: he has written novels, essays, short stories, a travel book, and a biography." This term does not fit within the context of the sentence, because an emotional author will not necessarily write many different kinds of works.

Explanation for Incorrect Answer B :  Choice (B) is incorrect. "Nebulous" means vague or indistinct. If one were to insert this term into the text, the sentence would read "Edmund White is a nebulous author: he has written novels, essays, short stories, a travel book, and a biography." A list of the different genres in which an author writes offers no support for a claim that the author is vague.

Explanation for Incorrect Answer C :  Choice (C) is incorrect. "Meticulous" means extremely careful or precise. If one were to insert this term into the text, the sentence would read "Edmund White is meticulous author: he has written novels, essays, short stories, a travel book, and a biography." While a meticulous author may write a number of different things, there is no inherent connection between these two ideas.

Explanation for Incorrect Answer E :  Choice (D) is incorrect. "Metaphoric" means relating to metaphors or figures of speech that make an implicit comparison. If one were to insert this term into the text, the sentence would read "Edmund White is metaphoric author: he has written novels, essays, short stories, a travel book, and a biography." The fact that White has written several different types of literature is unrelated to whether or not he writes metaphorically. All of the genres listed could be written perflectly well with or without the use of metaphors.

(A) demonstrative

(B) nebulous

(C) meticulous

(D) versatile

(E) metaphoric

2 The archaeologist believed the coin she unearthed was ------- evidence, unquestionable proof that the site dated to the fourth century.

页码,1/20The Official SAT Online Course

2006-11-12file://E:\新建文件夹\f4.htm

UnRe

gistered

Page 18: SAT test 6

严禁用于商业用途!

ANSWERS AND EXPLANATIONS Explanation for Correct Answer E :  Choice (E) is correct. "Indisputable" means unquestionable. If one were to insert this term into the text, the sentence would read "The archaeologist believed the coin she unearthed was indisputable evidence, unquestionable proof that the site dated to the fourth century." The comma in the sentence separates two synonymous phrases. Only "indisputable evidence" has the same meaning as "unquestionable proof."

Explanation for Incorrect Answer A :  Choice (A) is incorrect. "Immaterial" means unimportant. If one were to insert this term into the text, the sentence would read "The archaeologist believed the coin she unearthed was immaterial evidence, unquestionable proof that the site dated to the fourth century." The sentence implies that the archaeologist's discovery is significant, so it would be illogical to call the evidence "immaterial."

Explanation for Incorrect Answer B :  Choice (B) is incorrect. "Potential" means possible. If one were to insert this term into the text, the sentence would read "The archaeologist believed the coin she unearthed was potential evidence, unquestionable proof that the site dated to the fourth century." A discovery that could potentially provide evidence about an archaeological site is a far cry from "unquestionable proof."

Explanation for Incorrect Answer C :  Choice (C) is incorrect. Something that is "incriminating" shows evidence of a crime or fault. If one were to insert this term into the text, the sentence would read "The archaeologist believed the coin she unearthed was incriminating evidence, unquestionable proof that the site dated to the fourth century." The term "incriminating" is inappropriate in this sentence because the coin does not prove anyone guilty of wrongdoing.

Explanation for Incorrect Answer D :  Choice (D) is incorrect. "Nominal" means insignificant. If one were to insert this term into the text, the sentence would read "The archaeologist believed the coin she unearthed was nominal evidence, unquestionable proof that the site dated to the fourth century." "Nominal," or skimpy, evidence would never be described as "unquestionable proof."

(A) immaterial

(B) potential

(C) incriminating

(D) nominal

(E) indisputable

3Although the rigors of ballet dancing are primarily -------, this art is also emotionally and spiritually -------.

ANSWERS AND EXPLANATIONS Explanation for Correct Answer C :  Choice (C) is correct. "Physical" means something involving or characterized by vigorous bodily activity. "Demanding" in this context means requiring much effort or attention. If one were to insert these terms into the text, the sentence would read "Although the rigors of ballet dancing are primarily physical, this art is also emotionally and spiritually demanding." The words "although" and "also" indicate that the sentence will compare two related ideas. In this case, they set up a logical

(A) illusory . . taxing

(B) exaggerated . . balanced

(C) physical . . demanding

(D) appealing . . indulgent

(E) strenuous . . dubious

页码,2/20The Official SAT Online Course

2006-11-12file://E:\新建文件夹\f4.htm

UnRe

gistered

Page 19: SAT test 6

严禁用于商业用途!

contrast between ballet's physical and non-physical demands.

Explanation for Incorrect Answer A :  Choice (A) is incorrect. "Illusory" refers to something that is based on illusion or that is deceptive. "Taxing" means burdensome or difficult. If one were to insert these terms into the text, the sentence would read "Although the rigors of ballet dancing are primarily illusory, this art is also emotionally and spiritually taxing." The first clause of this sentence implies that ballet is not as physically challenging as it appears, so it does not make sense to then claim that its emotional and spiritual elements are "also . . . taxing," or difficult.

Explanation for Incorrect Answer B :  Choice (B) is incorrect. "Exaggerated" means overstated. "Balanced" means equal or equivalent. If one were to insert these terms into the text, the sentence would read "Although the rigors of ballet dancing are primarily exaggerated, this art is also emotionally and spiritually balanced." The claim that ballet is emotionally and spiritually balanced is in no way related to the claim that its rigors are exaggerated.

Explanation for Incorrect Answer D :  Choice (D) is incorrect. "Appealing" means attractive or inviting, and "indulgent" means lenient. If one were to insert these terms into the text, the sentence would read "Although the rigors of ballet dancing are primarily appealing, this art is also emotionally and spiritually indulgent." It makes little sense to describe the rigors of ballet as attractive, especially in connection to a claim about ballet's emotional and spiritual "indulgence."

Explanation for Incorrect Answer E :  Choice (E) is incorrect. "Strenuous" means requiring great effort, energy, and exertion. "Dubious" means fraught with uncertainty or doubt. If one were to insert these terms into the text, the sentence would read "Although the rigors of ballet dancing are primarily strenuous, this art is also emotionally and spiritually dubious." It is somewhat redundant to describe rigors as strenuous, and this claim is unrelated to the assertion that ballet is emotionally and spiritually "dubious," or doubtful.

4

Studies of ------- among turtles are sometimes ------- by the fact that the subjects live so long that researchers retire before the studies can be completed.

ANSWERS AND EXPLANATIONS Explanation for Correct Answer B :  Choice (B) is correct. "Longevity" means of great age or great span of life. "Hampered" means to impede in motion or progress. If one were to insert these terms into the text, the sentence would read "Studies of longevity among turtles are sometimes hampered by the fact that the subjects live so long that researchers retire before the studies can be completed." The sentence explains that the lengthy lifespan of turtles can cause problems for researchers. "Longevity" fits the description "lives so long." That turtles outlive the careers of the scientists studying them would certainly hamper, or impede, the study.

Explanation for Incorrect Answer A :  Choice (A) is incorrect. "Extinction" means the process of becoming extinct. "Enhanced" means improved, or advanced in qualities. If one were to insert these terms into the text, the sentence would read "Studies of extinction among turtles are sometimes enhanced by the fact that the subjects live so long that researchers retire before the studies can be completed." It is illogical to claim that a study could be enhanced, or improved, by the inability of researchers to complete it.

(A) extinction. . enhanced

(B) longevity. . hampered

(C) behavior. . belied

(D) mortality. . bolstered

(E) reproduction. . confirmed

页码,3/20The Official SAT Online Course

2006-11-12file://E:\新建文件夹\f4.htm

UnRe

gistered

Page 20: SAT test 6

严禁用于商业用途!

Explanation for Incorrect Answer C :  Choice (C) is incorrect. "Behavior" means the manner in which something functions. "Belied" means pictured falsely or misrepresented. If one were to insert these terms into the text, the sentence would read "Studies of behavior among turtles are sometimes belied by the fact that the subjects live so long that researchers retire before the studies can be completed." It makes some sense to say that a study could be belied, or misrepresented, by the problem of researchers retiring, but it is unclear why a behavior study would be affected by this problem.

Explanation for Incorrect Answer D :  Choice (D) is incorrect. "Mortality" means death rate. "Bolstered" means supported or propped up with. If one were to insert these terms into the text, the sentence would read "Studies of mortality among turtles are sometimes bolstered by the fact that the subjects live so long that researchers retire before the studies can be completed." A study on turtle mortality would not be bolstered, or propped up by, the retirement of the researcher.

Explanation for Incorrect Answer E :  Choice (E) is incorrect. "Reproduction" means the act of reproducing. "Confirmed" means strengthened or made valid. If one were to insert these terms into the text, the sentence would read "Studies of reproduction among turtles are sometimes confirmed by the fact that the subjects live so long that researchers retire before the studies can be completed." The fact of researchers retiring would do nothing to confirm, or prove, anything about reproduction.

5 A model of ------- behavior, Cunningham never ate or drank to excess.

ANSWERS AND EXPLANATIONS Explanation for Correct Answer A :  Choice (A) is correct. "Temperate" in this context refers to moderation in the indulgence of appetite. If one were to insert this term into the text, the sentence would read "A model of temperate behavior, Cunningham never ate or drank to excess." The comma indicates that the second half of the sentence will illustrate Cunningham's behavior. Someone who never consumes too much does indeed exhibit "temperate behavior."

Explanation for Incorrect Answer B :  Choice (B) is incorrect. Someone who is "laconic" speaks using a minimum of words. If one were to insert this term into the text, the sentence would read "A model of laconic behavior, Cunningham never ate or drank to excess." A tendency for reserved speech has nothing to do with a moderate appetite.

Explanation for Incorrect Answer C :  Choice (C) is incorrect. "Duplicitous" behavior is marked by deceptiveness. If one were to insert this term into the text, the sentence would read "A model of duplicitous behavior, Cunningham never ate or drank to excess." Deceptiveness is not necessarily related to eating and drinking habits.

Explanation for Incorrect Answer D :  Choice (D) is incorrect. "Aesthetic" means appreciative of beauty. If one were to insert this term into the text, the sentence would read "A model of aesthetic behavior, Cunningham never ate or drank to excess." One's appreciation of beauty does not necessarily affect one's appetite.

Explanation for Incorrect Answer E :  Choice (E) is incorrect. Someone who is "voluble" is marked by a ready flow of speech. If one were to insert this term into the text, the sentence would read "A model of voluble behavior, Cunningham never ate or drank to excess." It is illogical to claim that a talkative person always eats and drinks in moderation.

(A) temperate

(B) laconic

(C) duplicitous

(D) aesthetic

(E) voluble

页码,4/20The Official SAT Online Course

2006-11-12file://E:\新建文件夹\f4.htm

UnRe

gistered

Page 21: SAT test 6

严禁用于商业用途!

6 The entrepreneur had a well-deserved reputation for -------, having accurately anticipated many changes unforeseen by established business leaders.

ANSWERS AND EXPLANATIONS Explanation for Correct Answer A :  Choice (A) is correct. "Prescience" means foresight. If one were to insert this term into the text, the sentence would read "The entrepeneur had a well-deserved reputation for prescience, having accurately anticipated many changes unforeseen by established business leaders." The part of the sentence following the comma describes the missing term. It is logical for a person who can frequently predict "unforeseen" events to develop a reputation for "prescience."

Explanation for Incorrect Answer B :  Choice (B) is incorrect. "Sincerity" means genuineness and honesty. If one were to insert this term into the text, the sentence would read "The entrepeneur had a well-deserved reputation for sincerity, having accurately anticipated many changes unforeseen by established business leaders." Sincerity is not linked to one's ability to anticipate changes.

Explanation for Incorrect Answer C :  Choice (C) is incorrect. "Avarice" means greed. If one were to insert this term into the text, the sentence would read "The entrepeneur had a well-deserved reputation for avarice, having accurately anticipated many changes unforeseen by established business leaders." The entrepreneur may be a greedy person, but this sentence describes a different trait.

Explanation for Incorrect Answer D :  Choice (D) is incorrect. "Complicity" refers to a person's participation in a questionable act or a crime. If one were to insert this term into the text, the sentence would read "The entrepeneur had a well-deserved reputation for complicity, having accurately anticipated many changes unforeseen by established business leaders." This sentence offers no evidence that the entrepreneur has a reputation for conniving behavior

Explanation for Incorrect Answer E :  Choice (E) is incorrect. "Mendacity" means untruthfulness. If one were to insert this term into the text, the sentence would read "The entrepeneur had a well-deserved reputation for mendacity, having accurately anticipated many changes unforeseen by established business leaders." The description in the second half of this sentence does not support the claim that the entrepreneur was dishonest.

(A) prescience

(B) sincerity

(C) avarice

(D) complicity

(E) mendacity

7 Scientists require observable data, not -------, to support a hypothesis; sound science is grounded in ------- results rather than speculation.

ANSWERS AND EXPLANATIONS Explanation for Correct Answer E :  Choice (E) is correct. "Conjecture" means guesswork or predicting from incomplete or uncertain evidence. "Empirical" means by experiment or according to experience. If one were to insert these terms into the text, the sentence would read "Scientists

(A) induction . . diminutive

(B) experimentation . . pragmatic

(C) intuition . . fiscal

(D) bombast . . theoretical

(E) conjecture . . empirical

页码,5/20The Official SAT Online Course

2006-11-12file://E:\新建文件夹\f4.htm

UnRe

gistered

Page 22: SAT test 6

严禁用于商业用途!

require observable data, not conjecture, to support a hypothesis; sound science is grounded in empirical results rather than speculation." It makes sense to contrast "observable data" with "conjecture" and "empirical" with "speculation." Scientific criteria usually require hypotheses to be grounded in "empirical results" and "observable data," not "conjecture" and "speculation."

Explanation for Incorrect Answer A :  Choice (A) is incorrect. "Induction" means the process of deriving general principles from particular facts or instances. "Diminutive" means tiny. If one were to insert these terms into the text, the sentence would read "Scientists require observable data, not induction, to support a hypothesis; sound science is grounded in diminutive results rather than speculation." It makes no sense to say that sound science is grounded in "diminutive," or tiny, results. Nor does it make sense to contrast tiny results with "speculation."

Explanation for Incorrect Answer B :  Choice (B) is incorrect. "Experimentation" means the act of testing under controlled conditions. "Pragmatic" means practical. If one were to insert these terms into the text, the sentence would read "Scientists require observable data, not experimentation, to support a hypothesis; sound science is grounded in pragmatic results rather than speculation." "Experimentation" can provide "observable data"; the two are not opposites.

Explanation for Incorrect Answer C :  Choice (C) is incorrect. "Intuition" means immediate cognition. "Fiscal" means relating to finances. If one were to insert these terms into the text, the sentence would read "Scientists require observable data, not intuition, to support a hypothesis; sound science is grounded in fiscal results rather than speculation." While "intuition" is clearly distinguished from "observable data," science is supposed to rely on the physical results of experimentation regardless of the "fiscal," or financial, results the experimentation may involve.

Explanation for Incorrect Answer D :  Choice (D) is incorrect. "Bombast" means pompous speech or writing. "Theoretical" means based on theory. If one were to insert these terms into the text, the sentence would read "Scientists require observable data, not bombast, to support a hypothesis; sound science is grounded in theoretical results rather than speculation." "Theoretical results" can be described as speculative; however, theoretical results and speculation are not mutually exclusive.

8The director complained that the sitcom’s theme song was downright -------, having no more pep and vigor than a -------.

ANSWERS AND EXPLANATIONS Explanation for Correct Answer C :  Choice (C) is correct. "Lugubrious" means mournful, and a "dirge" is a song of grief. If one were to insert these terms into the text, the sentence would read "The director complained that the sitcom's theme song was downright lugubrious, having no more pep and vigor than a dirge." In this sentence, the information following the comma supports the claim preceding the comma. The director's complaint that the song was "lugubrious" is perfectly supported by calling it a "dirge."

Explanation for Incorrect Answer A :  Choice (A) is incorrect. "Tedious" means boring. "Jingle" is a piece of light singsong verse or rhyme. If one were to insert these terms into the text, the sentence would read "The director complained that the sitcom's theme song was downright tedious, having no more pep and vigor than a jingle." A "jingle" is a light or catchy song,

(A) tedious . . jingle

(B) inchoate . . lullaby

(C) lugubrious . . dirge

(D) facetious . . ballad

(E) sprightly . . eulogy

页码,6/20The Official SAT Online Course

2006-11-12file://E:\新建文件夹\f4.htm

UnRe

gistered

Page 23: SAT test 6

严禁用于商业用途!

thus it makes no sense to complain that a song has no more pep than a jingle.

Explanation for Incorrect Answer B :  Choice (B) is incorrect. "Inchoate" means in an initial or early stage. "Lullaby" is a soothing song with which to lull a child to sleep. If one were to insert these terms into the text, the sentence would read "The director complained that the sitcom's theme song was downright inchoate, having no more pep and vigor than a lullaby." It does not make complete sense to complain that a song lacks "pep and vigor" because it is in its early stages of development.

Explanation for Incorrect Answer D :  Choice (D) is incorrect. "Facetious" means joking or humorous. "Ballad" means a song of a romantic or sentimental nature. If one were to insert these terms into the text, the sentence would read "The director complained that the sitcom's theme song was downright facetious, having no more pep and vigor than a ballad." A facetious song would most likely be witty or joking, and would not likely be lacking pep. Furthermore, many ballads could be described as vigorous.

Explanation for Incorrect Answer E :  Choice (E) is incorrect. "Sprightly" means full of spirit and vitality. "Eulogy" is a speech praising someone who has died. If one were to insert these terms into the text, the sentence would read "The director complained that the sitcom's theme song was downright sprightly, having no more pep and vigor than a eulogy." A sprightly, or lively, song would certainly have more pep than a eulogy, or funeral speech.

      In between school days, we gathered hazelnuts,

 fished, had long deer-hunting weekends, went to

 powwows, beaded on looms, and made quilts. I did not

Line question the necessity or value of our school

5education, but somehow I grew up knowing it wasn’t

 the only education I would need. I’m thankful for those

 experiences of my Anishinaabe heritage, because now

 I know by heart not only the national anthem, but the

 ancient song of the loon. I recognize not only the alpha-

10bet and the parts of an English sentence, but the intricate

  language of a beaver’s teeth and tail.

9 The main idea of the passage is that the author

ANSWERS AND EXPLANATIONS Explanation for Correct Answer E :  Choice (E) is correct. As the author explains in the text, "No one questioned the necessity or value of our school education, but . . . it wasn't the only education I would need." References to "deer-hunting weekends" and "powwows" make it clear

(A) preferred certain academic subjects over others

(B) succeeded in learning to speak many foreign languages

(C) valued knowledge of the natural world more than book learning

(D) loved both family trips and tribal activities

(E) learned many important things both in and out of school

页码,7/20The Official SAT Online Course

2006-11-12file://E:\新建文件夹\f4.htm

UnRe

gistered

Page 24: SAT test 6

严禁用于商业用途!

that the author received an important cultural education outside the classroom.

Explanation for Incorrect Answer A :  Choice (A) is incorrect. The author indicates that education can take on different forms but does not express a preference for any academic subject.

Explanation for Incorrect Answer B :  Choice (B) is incorrect. While the author claims to have learned the "song of the loon" and the "intricate language of a beaver's teeth and tail," the only spoken language mentioned in the passage is English.

Explanation for Incorrect Answer C :  Choice (C) is incorrect. The author implies that both forms of education described in the passage are important but does not indicate a preference for one over the other.

Explanation for Incorrect Answer D :  Choice (D) is incorrect. The passage compares what the author learned both in school and outdoors. "Family trips and tribal activities" account for only the latter.

10 The author’s overall tone in this passage is best described as one of

ANSWERS AND EXPLANATIONS Explanation for Correct Answer D :  Choice (D) is correct. The author is clearly "thankful" (line 6) for having had the opportunity to supplement a school education with outdoor lessons.

Explanation for Incorrect Answer A :  Choice (A) is incorrect. While the author is clearly pleased to have received two complementary forms of education, the tone of the passage never elevates to a level of "jubilation," or rejoicing.

Explanation for Incorrect Answer B :  Choice (B) is incorrect. The author expresses "frustration" in neither content nor tone.

Explanation for Incorrect Answer C :  Choice (C) is incorrect. While the author seems to enjoy learning, his primary tone is not one of "curiosity."

Explanation for Incorrect Answer E :  Choice (E) is incorrect. The author writes with confidence, not "uncertainty."

(A) jubilation

(B) frustration

(C) curiosity

(D) appreciation

(E) uncertainty

      Daily life is overflowing with mundane mental events.

 A paper clip gleams amid stacks of documents, a friend’s

 face shines like a beacon out of a crowd, the smell of freshly baked bread evokes childhood

页码,8/20The Official SAT Online Course

2006-11-12file://E:\新建文件夹\f4.htm

UnRe

gistered

Page 25: SAT test 6

严禁用于商业用途!

Line memories—thoughts

5and perceptions such as these flow by with monotonous

  ease.

      So it seems, anyway. Yet given what scientists know

 about how brains work, even the ability to perceive a

 paper clip on a messy desk represents an extraordinary

10 and mysterious achievement.

11 In the first paragraph, the author implicitly likens our experience of sensory impressions to

ANSWERS AND EXPLANATIONS Explanation for Correct Answer B :  Choice (B) is correct. Suggestions that "life is overflowing with mundane mental events" and that "thoughts and perceptions . . . flow by" produce an implicit comparison between people's daily experiences and an "unobstructed stream."

Explanation for Incorrect Answer A :  Choice (A) is incorrect. The author's description of "mundane" and "monotonous" thoughts directly contrasts with the turbulence associated with "an emotional roller coaster."

Explanation for Incorrect Answer C :  Choice (C) is incorrect. The author lists several different images in the first paragraph but does not suggest that any of them are repetitive.

Explanation for Incorrect Answer D :  Choice (D) is incorrect. While the author asserts that particular perceptions can evoke old memories, it would be inaccurate to claim that all thoughts are linked to nostalgia.

Explanation for Incorrect Answer E :  Choice (E) is incorrect. The author implies that most daily perceptions are boring, not "diverting," or amusing.

(A) an emotional roller coaster

(B) an unobstructed stream

(C) a repeated image

(D) a nostalgic reminiscence

(E) a diverting daydream

12The primary purpose of the passage is to

ANSWERS AND EXPLANATIONS Explanation for Correct Answer D :  Choice (D) is correct. The passage mainly serves to highlight the "complexity of

(A) celebrate life’s mundane but gratifying pleasures

(B) convey the overwhelming confusion of every-day life

(C) explore the biological implications of a person’s decisions

(D) suggest the complexity of perceptual processes

(E) present a scientific analysis of an automatic reflex

页码,9/20The Official SAT Online Course

2006-11-12file://E:\新建文件夹\f4.htm

UnRe

gistered

Page 26: SAT test 6

严禁用于商业用途!

perceptual processes" that might otherwise be taken for granted.

Explanation for Incorrect Answer A :  Choice (A) is incorrect. While the author seems to marvel at the underlying complexity of everyday perceptions, the passage's tone is more analytical than celebratory.

Explanation for Incorrect Answer B :  Choice (B) is incorrect. Far from "convey[ing] the overwhelming confusion of everyday life," the passage in fact shows how our perceptions render everyday life easy and "monotonous."

Explanation for Incorrect Answer C :  Choice (C) is incorrect. The passage does not discuss people's decisions, only their perceptions.

Explanation for Incorrect Answer E :  Choice (E) is incorrect. The passage does allude to the scientific study of perception, but it does not present actual "scientific analysis" of any kind.

The following passage is from a 1991 essay that discusses the debate over which authors should be taught in English classes.

      Now, what are we to make of this sputtering debate,

 in which charges of imperialism are met by equally

 passionate accusations of vandalism, in which each side

Linehates the other, and yet each seems to have its share of

5reason? It occurs to me that perhaps what we have here

 is one of those debates in which the opposing sides,

 unbeknownst to themselves, share a myopia that will turn

 out to be the most interesting and important feature of the

 whole discussion, a debate, for instance, like that of the

10Founding Fathers over the nature of the franchise. Think

 of all the energy and passion spent debating the question

 of property qualifications, or direct versus legislative

 elections, while all along, unmentioned and unimagined,

 was the fact—to us so central—that women and slaves

15 were never considered for any kind of vote.

      While everyone is busy fighting over what should be taught in the classroom, something is being

页码,10/20The Official SAT Online Course

2006-11-12file://E:\新建文件夹\f4.htm

UnRe

gistered

Page 27: SAT test 6

严禁用于商业用途!

  overlooked.

 That is the state of reading, and books, and literature in our

 country, at this time. Why, ask yourself, is everyone so hot

20under the collar about what to put on the required-reading

 shelf? It is because, while we have been arguing so fiercely

 about which books make the best medicine, the patient has

  been slipping deeper and deeper into a coma.

      Let us imagine a country in which reading was a popular

25voluntary activity. There, parents read books for their own

 edification and pleasure and are seen by their children at

 this silent and mysterious pastime. These parents also read

 to their children, give them books for presents, talk to them

 about books, and underwrite, with their taxes, a public

30library system that is open all day, every day. In school,

 the children study certain books together but also have an

 active reading life of their own. Years later, it may even

 be hard for them to remember if they read Jane Eyre at

  home and Judy Blume1 in class or the other way around.

35In college, young people continue to be assigned certain

 books, but far more important are the books they discover

 for themselves browsing in the library, in bookstores, on

 the shelves of friends, one book leading to another, back

 and forth in history and across languages and cultures.

40After graduation, they continue to read and in the fullness

 of time produce a new generation of readers. Oh happy

  land! I wish we all lived there.

      In that country of real readers, voluntary, active, self-determined readers, a debate like the current

页码,11/20The Official SAT Online Course

2006-11-12file://E:\新建文件夹\f4.htm

UnRe

gistered

Page 28: SAT test 6

严禁用于商业用途!

  one over the

45canon would not be taking place. Or if it did, it would be

 as a kind of parlor game: What books would you take to

 a desert island? Everyone would know that the top-ten list

 was merely a tiny fraction of the books one would read in

 a lifetime. It would not seem racist or sexist or hopelessly

50hidebound to put Nathaniel Hawthorne on the list and not

  Toni Morrison2. It would be more like putting oatmeal

 and not noodles on the breakfast menu—a choice partly

 arbitrary, partly a nod to the national past, and partly, dare

 one say it, a kind of reverse affirmative action: School

55might frankly be the place where one reads the books that

 are a little off-putting, that have gone a little cold, that you

 might overlook because they do not address, in reader-

 friendly contemporary fashion, the issues most immediately

 at stake in modern life but that, with a little study, turn out

60to have a great deal to say. Being on the list wouldn’t mean

 so much. It might even add to a writer’s cachet not to be on

 the list, to be in one way or another too heady, too daring,

 too exciting to be ground up into institutional fodder for

 teenagers. Generations of high school kids have been turned

65 off to George Eliot3 by being forced to read Silas Marner

 at a tender age. One can imagine a whole new readership

 for her if grown-ups were left to approach Middlemarch

 and Daniel Deronda with open minds, at their leisure.

1 Jane Eyre, by Charlotte Brontë, is a nineteenth-century

页码,12/20The Official SAT Online Course

2006-11-12file://E:\新建文件夹\f4.htm

UnRe

gistered

Page 29: SAT test 6

严禁用于商业用途!

 

novel. Judy Blume writes contemporary young adult novels. 2  Hawthorne was a nineteenth-century American writer. Toni Morrison is a contemporary American writer. 3 George Eliot was the pseudonym of a nineteenth-century female British novelist.

13 According to the author, too much energy today is spent debating

ANSWERS AND EXPLANATIONS Explanation for Correct Answer D :  Choice (D) is correct. The author suggests that more time should be spent reviving students' interest in reading, not debating what they should read in school.

Explanation for Incorrect Answer A :  Choice (A) is incorrect. The text suggests that the frenzy regarding English curricula is misplaced, but it does not imply that there is an excessive push for system-wide change.

Explanation for Incorrect Answer B :  Choice (B) is incorrect. It is more likely that the author would argue for increased efforts “to make literature seem relevant,” and therefore appealing, to students.

Explanation for Incorrect Answer C :  Choice (C) is incorrect. The author refers to different writers but does not assess the relative quality of their work.

Explanation for Incorrect Answer E :  Choice (E) is incorrect. The author mentions several classic novels in the passage but does not imply that the merits of these books are debated.

(A) how to improve the education system

(B) how to make literature seem relevant

(C) who the better writers are

(D) what students should read in school

(E) whether or not to teach classic works

14

In the first two paragraphs of the passage (lines 1-23), the author suggests that both sides of the debate

ANSWERS AND EXPLANATIONS Explanation for Correct Answer A :  Choice (A) is correct. The author asserts that the debate swirling around academic reading lists widely ignores the crucial fact that modern youth have a deteriorating relationship with books.

(A) neglect a fundamental issue

(B) disregard a key piece of evidence

(C) ignore opposing views

(D) lack a historical perspective

(E) dismiss a valuable tradition

页码,13/20The Official SAT Online Course

2006-11-12file://E:\新建文件夹\f4.htm

UnRe

gistered

Page 30: SAT test 6

严禁用于商业用途!

Explanation for Incorrect Answer B :  Choice (B) is incorrect. The first two paragraphs allude to various arguments, but they do not indicate that any of the arguments lack evidence.

Explanation for Incorrect Answer C :  Choice (C) is incorrect. The author argues that the debate ignores a key issue. At no point, however, does the passage suggest that the debate's participants "ignore" opposing viewpoints.

Explanation for Incorrect Answer D :  Choice (D) is incorrect. The passage contains an example drawn from United States history, but the author does not suggest that either side of the debate lacks "historical perspective."

Explanation for Incorrect Answer E :  Choice (E) is incorrect. The author imagines a world in which reading is a valuable domestic tradition, passed from one generation to the next, but does not claim that both sides of the debate have dismissed any traditions.

15 The author invokes “the Founding Fathers” (lines 9-10) chiefly in order to

ANSWERS AND EXPLANATIONS Explanation for Correct Answer B :  Choice (B) is correct. Lines 9–10 set up a "parallel," or comparison, between the modern debate about reading lists and historical debates concerning property rights and electoral procedures. All, the author argues, fail to address crucial issues.

Explanation for Incorrect Answer A :  Choice (A) is incorrect. A mere mention of the "Founding Fathers" may ignite a spark of "patriotism" in some readers, but the text does not suggest that this is the author's purpose in employing the phrase.

Explanation for Incorrect Answer C :  Choice (C) is incorrect. The author argues that the modern debate over reading lists is reminiscent of the early American debate regarding “direct versus legislative elections.” The role of legislation in the passage, however, does not extend beyond this reference.

Explanation for Incorrect Answer D :  Choice (D) is incorrect. Although the passage implies that ideas often change over time, the author's reference to "the Founding Fathers" does not indicate that attitudes regarding the state of reading in the United States will change in the future.

Explanation for Incorrect Answer E :  Choice (E) is incorrect. "Progress" may be compatible with certain traditions, but the author does not make such a claim at any point in the passage.

(A) appeal to the reader’s sense of patriotism

(B) introduce a historical parallel

(C) examine the history of legislative debate

(D) remind the reader how attitudes change over time

(E) suggest that progress is compatible with tradition

16 In line 18, “state” most nearly means

(A) government

(B) territory

(C) condition

(D) scale

页码,14/20The Official SAT Online Course

2006-11-12file://E:\新建文件夹\f4.htm

UnRe

gistered

Page 31: SAT test 6

严禁用于商业用途!

ANSWERS AND EXPLANATIONS Explanation for Correct Answer C :  Choice (C) is correct. "State" in this context refers to a condition or mode of being. The author suggests throughout the passage that the "state" of Americans' relationships with books is deteriorating.

Explanation for Incorrect Answer A :  Choice (A) is incorrect. While the term "state" refers to a mode of government in some cases, this definition is inappropriate in the context of line 18.

Explanation for Incorrect Answer B :  Choice (B) is incorrect. "State" can mean a territorial unit, but this meaning is unlikely within the context of the text.

Explanation for Incorrect Answer D :  Choice (D) is incorrect. It would be illogical to suggest that the author would refer to the "scale," or unit of measurement, of reading.

Explanation for Incorrect Answer E :  Choice (E) is incorrect. "State" can mean a mental or an emotional "condition," but activities such as reading cannot be described as having either.

(E) mood

17 In line 23, the “coma” represents the

ANSWERS AND EXPLANATIONS Explanation for Correct Answer B :  Choice (B) is correct. "Comatose" can also mean lethargic or apathetic. In creating a parallel between the "coma" in line 23 and the "state of reading" in line 18, the author is suggesting that Americans' enthusiasm for books is "slipping deeper and deeper."

Explanation for Incorrect Answer A :  Choice (A) is incorrect. Debate participants may be "hot under the collar," but the text makes no reference to upset or rebellious students.

Explanation for Incorrect Answer C :  Choice (C) is incorrect. There is little evidence in the passage to support the claim that books on academic reading lists are "scarce," or in short supply.

Explanation for Incorrect Answer D :  Choice (D) is incorrect. There is no mention of "archaic language" at any point in the text.

Explanation for Incorrect Answer E :  Choice (E) is incorrect. Certain books reflect popular culture, but the author does not draw any comparisions between "popular media" and a comatose patient.

(A) rebellion of students against traditional texts

(B) lack of enthusiasm for reading in general

(C) scarcity of books on official reading lists

(D) difficulty of understanding archaic language

(E) negative effects of popular media

18 In lines 24–27 (“Let . . . pastime”), the country described is noteworthy because

页码,15/20The Official SAT Online Course

2006-11-12file://E:\新建文件夹\f4.htm

UnRe

gistered

Page 32: SAT test 6

严禁用于商业用途!

ANSWERS AND EXPLANATIONS Explanation for Correct Answer B :  Choice (B) is correct. Lines 24–27 describe an imagined world in which children routinely observe their parents reading for "pleasure."

Explanation for Incorrect Answer A :  Choice (A) is incorrect. Reading may expose people to new interests or activities, but the author does not address this possibility.

Explanation for Incorrect Answer C :  Choice (C) is incorrect. While children who notice their parents' enjoyment of books may be more likely to read at an early age, the author makes no such claim.

Explanation for Incorrect Answer D :  Choice (D) is incorrect. The passage does not describe "many activities" shared between adults and children, only reading.

Explanation for Incorrect Answer E :  Choice (E) is incorrect. Reading and writing skills are often linked, but lines 24–27 address only the former.

(A) people have allowed new interests to develop from their reading

(B) parents demonstrate their enjoyment of reading

(C) children learn to read at an early age

(D) children and parents share many activities

(E) writing is viewed as a valuable skill

19 Lines 30–39 (“In school . . . cultures”) present a model of education where students learn to

ANSWERS AND EXPLANATIONS Explanation for Correct Answer E :  Choice (E) is correct. In lines 30–39, the author describes a "happy land" where students are encouraged to explore books that represent different eras, languages, and cultures.

Explanation for Incorrect Answer A :  Choice (A) is incorrect. The author encourages students to read culturally diverse texts in line 39 but does not imply that “diversity” is more important than “tradition.”

Explanation for Incorrect Answer B :  Choice (B) is incorrect. The selected lines construct a world in which the canon debate does not exist.

Explanation for Incorrect Answer C :  Choice (C) is incorrect. The students mentioned in the quoted passage may “reflect critically” on literature and culture, but the passage does not describe them analyzing the nature of the American educational system.

Explanation for Incorrect Answer D : 

(A) value cultural diversity over tradition

(B) respect the views of both sides of the debate

(C) reflect critically on the nature of American schooling

(D) differentiate between classic and contemporary works

(E) explore the world through wide-ranging reading

页码,16/20The Official SAT Online Course

2006-11-12file://E:\新建文件夹\f4.htm

UnRe

gistered

Page 33: SAT test 6

严禁用于商业用途!

Choice (D) is incorrect. Although the students mentioned in the quoted passage may be able to differentiate between classic and modern works, this ability is not discussed in the passage.

20 In lines 33–34, the author cites Jane Eyre and Judy Blume primarily in order to

ANSWERS AND EXPLANATIONS Explanation for Correct Answer A :  Choice (A) is correct. The third paragraph describes an idealized reality "in which reading was a popular voluntary activity." The author suggests that in this world students read books so voraciously that the distinction between books read for school and books read for pleasure becomes blurred.

Explanation for Incorrect Answer B :  Choice (B) is incorrect. Although Jane Eyre and Judy Blume novels were composed centuries apart, the passage does not mention these two authors to suggest that young readers are incapable of "distinguishing between literature of different eras."

Explanation for Incorrect Answer C :  Choice (C) is incorrect. Jane Eyre's popularity has endured through time, but the author does not refer to this nineteenth-century classic in an assertion that modern novels, such as those written by Judy Blume, have only short-term appeal.

Explanation for Incorrect Answer D :  Choice (D) is incorrect. Certain literary classics such as Jane Eyre may appeal to "even reluctant readers," but lines 33–34 are concerned with only "voluntary, active, self-determined" readers.

Explanation for Incorrect Answer E :  Choice (E) is incorrect. The author suggests that students may forget the impetus behind their reading of Jane Eyre or of Judy Blume novels but does not imply that Jane Eyre and the works of Judy Blume are interchangeable.

(A) propose that a love of reading might blur a commonly perceived distinction

(B) show that younger readers cannot distinguish between literature of different eras

(C) argue that most modern novels have no lasting impact on readers

(D) observe that classic literature has great appeal for even reluctant readers

(E) indicate that certain works are interchangeable

21 In lines 35-39 (“In college . . . cultures”), the education illustrated is best described as

ANSWERS AND EXPLANATIONS Explanation for Correct Answer C :  Choice (C) is correct. "Eclectic" means made up of elements from a variety of sources. An education supplemented by the independent discovery of books "in the library, in bookstores, on the shelves of friends" and beyond would indeed be "eclectic."

(A) elitist

(B) philanthropic

(C) eclectic

(D) methodical

(E) rudimentary

页码,17/20The Official SAT Online Course

2006-11-12file://E:\新建文件夹\f4.htm

UnRe

gistered

Page 34: SAT test 6

严禁用于商业用途!

Explanation for Incorrect Answer A :  Choice (A) is incorrect. "Elitist" means snobbish, or privileged, but the education illustrated in these lines is available to the masses.

Explanation for Incorrect Answer B :  Choice (B) is incorrect. It is illogical to describe the education illustrated in the passage as "philanthropic," or charitable.

Explanation for Incorrect Answer D :  Choice (D) is incorrect. "Methodical" means systematic. The education described in the passage, however, is more loosely structured.

Explanation for Incorrect Answer E :  Choice (E) is incorrect. The education mentioned in the quoted passage is hardly "rudimentary," or basic.

22 In lines 54–60 (“School . . . say”), the author describes a world in which schools teach books that are

ANSWERS AND EXPLANATIONS Explanation for Correct Answer D :  Choice (D) is correct. The reading list described in lines 54–60 comprises books that do not address "the issues most immediately at stake in modern life" and that thus are “not obviously relevant.”

Explanation for Incorrect Answer A :  Choice (A) is incorrect. Some readers may find the books described in lines 54–60 “interesting,” but the author does not imply that they are particularly engaging. In fact, the author says they are “off-putting” and “cold.”

Explanation for Incorrect Answer B :  Choice (B) is incorrect. The ideal literary canon described in the final paragraph may include “celebrated” books, but the author does not address their popularity.

Explanation for Incorrect Answer C :  Choice (C) is incorrect. While the passage suggests that academic reading lists have caused a good deal of controversy, the author does not discuss whether or not the books on an ideal list would be controversial.

Explanation for Incorrect Answer E :  Choice (E) is incorrect. The author does not indicate how inspirational the books on an ideal reading list would be.

(A) interesting

(B) celebrated

(C) uncontroversial

(D) not obviously relevant

(E) not likely to inspire

23 Lines 60-64 (“Being . . . teenagers”) suggest that excluding a book from a reading list might

ANSWERS AND EXPLANATIONS

(A) enhance the reputation of the book’s author

(B) encourage students to protest the decision

(C) influence course curricula nationwide

(D) appease conservative parents

(E) disappoint the book’s fans

页码,18/20The Official SAT Online Course

2006-11-12file://E:\新建文件夹\f4.htm

UnRe

gistered

Page 35: SAT test 6

严禁用于商业用途!

Explanation for Correct Answer A :  Choice (A) is correct. Lines 60–64 suggest that exclusion from the list would contribute to an author's reputation for being "daring" and "exciting."

Explanation for Incorrect Answer B :  Choice (B) is incorrect. Lines 60–64 give no indication that students would "protest" the exclusion of any book.

Explanation for Incorrect Answer C :  Choice (C) is incorrect. Though the exclusion of a book from the author's ideal canon could affect what students read in school, lines 60–64 are more concerned with the influence that such an exclusion would have on a writer's reputation.

Explanation for Incorrect Answer D :  Choice (D) is incorrect. While "conservative parents" may support the exclusion of "heady" or "daring" authors from academic reading lists, lines 60–64 make no reference to parents at all.

Explanation for Incorrect Answer E :  Choice (E) is incorrect. Lines 60–64 say nothing about the "fans" of any particular book.

24 The main purpose of the passage is to

ANSWERS AND EXPLANATIONS Explanation for Correct Answer A :  Choice (A) is correct. The main goal of the passage is to shift the reading debate's focus from the content of academic reading lists to ways that Americans' ailing relationships with books might be improved.

Explanation for Incorrect Answer B :  Choice (B) is incorrect. Instead of promoting "one side" of an existing debate, the author suggests that both sides are ignoring "the most interesting and important feature of the whole discussion."

Explanation for Incorrect Answer C :  Choice (C) is incorrect. The passage provides less of a practical solution to a problem than a new perspective from which to view the problem.

Explanation for Incorrect Answer D :  Choice (D) is incorrect. The author makes no attempt at reviving "a discredited idea" at any point in the passage.

Explanation for Incorrect Answer E :  Choice (E) is incorrect. The author's focus is on reading rather than on writing.

(A) shift the focus of a debate

(B) support one side in a debate

(C) suggest a practical solution

(D) revive a discredited idea

(E) promote certain kinds of writing

    

Back to Score Report  

Copyright © 2006 The College Board. All rights reserved. Privacy Policy Terms of Use Contact Us

 

页码,19/20The Official SAT Online Course

2006-11-12file://E:\新建文件夹\f4.htm

UnRe

gistered

Page 36: SAT test 6

严禁用于商业用途!

页码,20/20The Official SAT Online Course

2006-11-12file://E:\新建文件夹\f4.htm

UnRe

gistered

Page 37: SAT test 6

严禁用于商业用途!

Help | Profile | My Organizer | My Bookmarks | Logout

Answers and Explanations

Test Sections

Section 1

Section 2

Section 4

Section 5

Section 6

Section 7

Section 8

Section 9

Section 10

Back to Score Report  

View Answers and Explanations     Online - Practice Test #6

1 Inside famed actor Lily Langtry’s private railroad car were a drawing room with a piano, bath fixtures of silver, and there were draperies trimmed with Brussels lace.

ANSWERS AND EXPLANATIONS Explanation for Correct Answer B :  Choice (B) is correct. It avoids the error of the original by making the third item in the series a noun phrase, as the other two items are.

Explanation for Incorrect Answer A :  Choice (A) involves an error in parallelism. The third item in the series should be a noun phrase like the other two ("a drawing room with a piano," "bath fixtures of silver"), not an independent clause.

Explanation for Incorrect Answer C :  Choice (C) involves an error in parallelism. The third item in the series should be a noun phrase like the other two ("a drawing room with a piano," "bath fixtures of silver"), not an independent clause.

Explanation for Incorrect Answer D :  Choice (D) involves an error in parallelism. The third item in the series should be a noun phrase like the other two ("a drawing room with a piano," "bath fixtures of silver"), not an independent clause.

Explanation for Incorrect Answer E :  Choice (E) involves an error in parallelism. The third item in the series should be a noun phrase like the other two ("a drawing room with a piano," "bath fixtures of silver"), not an independent clause.

(A) there were draperies trimmed with Brussels lace

(B) draperies trimmed with Brussels lace

(C) trimmed with Brussels lace were draperies

(D) the draperies were trimmed with Brussels lace

(E) draperies trimmed with Brussels lace were there

2Samuel Adams was by no means the first American to espouse the democratic cause, but he has been the first who conceived the party machinery that made it practical.

ANSWERS AND EXPLANATIONS Explanation for Correct Answer D :  Choice (D) is correct. It avoids the error of the original by using the past tense consistently.

(A) has been the first who conceived

(B) had been the first who conceived

(C) was the first having conceived

(D) was the first to conceive

(E) having been the first to conceive

页码,1/21The Official SAT Online Course

2006-11-12file://E:\新建文件夹\f5.htm

UnRe

gistered

Page 38: SAT test 6

严禁用于商业用途!

Explanation for Incorrect Answer A :  Choice (A) involves an error in verb tense. It shifts from past tense ("was") to present perfect tense ("has been") for no reason.

Explanation for Incorrect Answer B :  Choice (B) involves an error in verb tense. It shifts from past tense ("was") to present perfect tense ("had been") for no reason.

Explanation for Incorrect Answer C :  Choice (C) involves an error in verb form. The participle "Having conceived" should be the infinitive form "to conceive."

Explanation for Incorrect Answer E :  Choice (E) creates a fragment. There is no subject for the verb "having been."

3 The plans were made too hastily, without enough thought behind it.

ANSWERS AND EXPLANATIONS Explanation for Correct Answer C :  Choice (C) is correct. It avoids the error of the original by using a plural pronoun ("them") to refer to the plural noun "plans."

Explanation for Incorrect Answer A :  Choice (A) involves an error in agreement. The singular pronoun "it" cannot correctly refer to the plural noun "plans."

Explanation for Incorrect Answer B :  Choice (B) involves improper modification. To modify the verb "made," "hasty" should be "hastily."

Explanation for Incorrect Answer D :  Choice (D) involves improper modification. To modify the verb "made," "hasty" should be "hastily."

Explanation for Incorrect Answer E :  Choice (E) involves an error in agreement. The singular pronoun "it" cannot correctly refer to the plural noun "plans."

(A) too hastily, without enough thought behind it

(B) too hasty, without enough thought behind it

(C) too hastily, without enough thought behind them

(D) too hasty, and there is not enough thought behind them

(E) too hastily, and there is not enough thought behind it

4

Many psychologists do not use hypnosis in their practices, it is because they know very little about it and are wary of it as a result.

(A) practices, it is because they know very little about it and are wary of it as a result.

(B) practices because they know very little about it and are therefore wary of it.

(C) practices for the reason that they know very little about it, with resulting wariness

(D) practices because of knowing very little about it and therefore they are wary of it

(E) practices, their knowledge of it being very little results in wariness of it

页码,2/21The Official SAT Online Course

2006-11-12file://E:\新建文件夹\f5.htm

UnRe

gistered

Page 39: SAT test 6

严禁用于商业用途!

ANSWERS AND EXPLANATIONS Explanation for Correct Answer B :  Choice (B) is correct. It avoids the error of the original by using a second verb phrase ("are therefore wary of it as a result") instead of a second independent clause.

Explanation for Incorrect Answer A :  Choice (A) involves improper coordination. Two complete thoughts ("Many...practices" and "it...result") are joined by only a comma.

Explanation for Incorrect Answer C :  Choice (C) involves improper phrasing. Whose "wariness" is being referred to is not clear.

Explanation for Incorrect Answer D :  Choice (D) involves improper coordination. Two complete thoughts ("Many...it" and" therefore...result") are joined only by a conjuction ("and"), without a comma to proceed it.

Explanation for Incorrect Answer E :  Choice (E) involves improper pronoun case. "It" should be "its."

5 No two of the specimens was sufficiently alike to warrant them being called members of a single species.

ANSWERS AND EXPLANATIONS Explanation for Correct Answer E :  Choice (E) is correct. It avoids the error of the original by using a plural verb ("were") for the subject "two."

Explanation for Incorrect Answer A :  Choice (A) involves an error in agreement. The subject "two" requires a plural verb ("were").

Explanation for Incorrect Answer B :  Choice (B) involves an error in agreement. The subject "two" requires a plural verb ("were").

Explanation for Incorrect Answer C :  Choice (C) involves an error in agreement. The subject "two" requires a plural verb ("were").

Explanation for Incorrect Answer D :  Choice (D) involves wordiness. The wordy phrase "the calling of them" should simply be "calling them."

(A) was sufficiently alike to warrant them being called

(B) was sufficiently alike to warrant the calling of them

(C) was sufficiently alike to warrant their being called

(D) were sufficiently alike to warrant the calling of them

(E) were sufficiently alike to warrant calling them

6 My grandson thinks he can cook better than any other person at the fair; and he has the blue ribbons to prove it.

(A) My grandson thinks he can cook better than any other person at the fair; and he

页码,3/21The Official SAT Online Course

2006-11-12file://E:\新建文件夹\f5.htm

UnRe

gistered

Page 40: SAT test 6

严禁用于商业用途!

ANSWERS AND EXPLANATIONS Explanation for Correct Answer B :  Choice (B) is correct. It avoids the error of the original by using a comma to link two independent clauses joined by the conjunction "and."

Explanation for Incorrect Answer A :  Choice (A) involves improper coordination. It uses a semicolon where a comma is necessary.

Explanation for Incorrect Answer C :  Choice (C) creates an illogical sentence. It illogically suggests that the grandson's having blue ribbons is a result of his thinking that he is the best cook at the fair.

Explanation for Incorrect Answer D :  Choice (D) results in an illogical sentence. There is no relationship between the idea in the first part of the sentence ("To think . . . fair") and the idea in the last part of the sentence ("my grandson . . . to prove it.").

Explanation for Incorrect Answer E :  Choice (E) creates an illogical sentence. It does not make sense to say that the blue ribbons that the grandson had somehow prove that he was "thinking he can cook better . . . fair."

(B) My grandson thinks he can cook better than any person at the fair, and he

(C) My grandson thinks he can cook better than any person at the fair, consequently he

(D) To think he can cook better than any other person at the fair, my grandson

(E) Thinking he can cook better than any other person at the fair, my grandson

7 Differing only slightly from the Greeks were the Roman theaters, which were often freestanding rather than part of a hillside.

ANSWERS AND EXPLANATIONS Explanation for Correct Answer B :  Choice (B) is correct. It avoids the error of the original by correctly comparing "Greek theaters" to "Roman theaters."

Explanation for Incorrect Answer A :  Choice (A) involves an illogical comparison. It compares "the Greeks" with "the Roman theaters."

Explanation for Incorrect Answer C :  Choice (C) involves an illogical comparison. It compares "the Greeks" with "the Roman theaters."

Explanation for Incorrect Answer D :  Choice (D) involves improper coordination. Two complete thoughts ("The Greeks...Romans" and "they...hillside") are joined by only a comma.

Explanation for Incorrect Answer E : 

(A) Differing only slightly from the Greeks were the Roman theaters, which

(B) Differing only slightly from Greek theaters, Roman theaters

(C) Differing only in the slightest from the Greeks were the Roman theaters, which

(D) The Greeks differed only slightly from the Romans, they

(E) The Greek theaters differed from the Roman theaters only slightly, where they

页码,4/21The Official SAT Online Course

2006-11-12file://E:\新建文件夹\f5.htm

UnRe

gistered

Page 41: SAT test 6

严禁用于商业用途!

Choice (E) involves an error in pronoun reference. There is no place to which "where" can refer.

8 When chronological order is followed too mechanically, they are obscuring rather than clarifying important relationships.

ANSWERS AND EXPLANATIONS Explanation for Correct Answer D :  Choice (D) is correct. It avoids the error of the original by omitting the plural pronoun ("they"), which did not agree with the singular noun "chronological order."

Explanation for Incorrect Answer A :  Choice (A) involves an error in agreement. The plural pronoun "they" is used incorrectly to refer to the singular noun "chronological order."

Explanation for Incorrect Answer B :  Choice (B) involves an error in verb form. The verb "clarifying" should be "clarifies."

Explanation for Incorrect Answer C :  Choice (C) involves a pronoun error. The pronoun "it" is unnecessary.

Explanation for Incorrect Answer E :  Choice (E) involves a pronoun error. It is not clear what the pronoun "it" is meant to refer to.

(A) When chronological order is followed too mechanically, they are obscuring rather than clarifying important relationships.

(B) When chronological order is followed too mechanically, it obscures rather than clarifying important relationships.

(C) Chronological order, if too mechanically followed, obscures rather than it clarifies important relationships.

(D) Chronological order, if followed too mechanically, obscures rather than clarifies important relationships.

(E) If you follow a too mechanical chronological order, it obscures rather than clarifies important relationships.

9 Small marine crustaceans known as krill are often fed to farm animals, but there is not much human consumption.

ANSWERS AND EXPLANATIONS Explanation for Correct Answer C :  Choice (C) is correct. It avoids the error of the original by avoiding unclear pronoun usage.

Explanation for Incorrect Answer A :  Choice (A) involves an error in pronoun reference. "There" does not refer to anything in the sentence.

Explanation for Incorrect Answer B :  Choice (B) involves wordiness. The wordy phrase "consumption is not done much by people" should be simply "are rarely consumed by people" or "are rarely eaten by people."

(A) animals, but there is not much human consumption

(B) animals, but consumption is not done much by people

(C) animals but are rarely eaten by people

(D) animals, but eating them is rarely done by humans

(E) animals, but among people there is not much consumption

页码,5/21The Official SAT Online Course

2006-11-12file://E:\新建文件夹\f5.htm

UnRe

gistered

Page 42: SAT test 6

严禁用于商业用途!

Explanation for Incorrect Answer D :  Choice (D) involves an error in pronoun reference. The pronoun "them" incorrectly refers to the closest previous plural noun, "farm animals."

Explanation for Incorrect Answer E :  Choice (E) involves an unclear reference. The object of "consumption" is not clear.

10 The educator’s remarks stressed that well-funded literacy programs are needed if everyone is to gain the skills required for survival in society.

ANSWERS AND EXPLANATIONS Explanation for Correct Answer A :  Choice (A) is correct. It correctly uses a relative clause (introduced by "that") to indicate what the educator was stressing.

Explanation for Incorrect Answer B :  Choice (B) involves improper verb form. The verb "needed" lacks an auxillary verb ("are").

Explanation for Incorrect Answer C :  Choice (C) involves improper coordination. Two complete thoughts ("The educator's...programs" and "everyone...society") are joined by only a conjunction ("for"), without the comma that should precede it.

Explanation for Incorrect Answer D :  Choice (D) involves improper coordination. Two complete thoughts ("The educator's...programs" and "everyone...society") are joined by only a conjunction ("and"), without the comma that should precede it.

Explanation for Incorrect Answer E :  Choice (E) creates an illogical sentence. A noun can be "stressed," but an adverb ("why") cannot.

(A) that well-funded literacy programs are needed if everyone is to gain

(B) that well-funded literacy programs needed in gaining

(C) there is a need of well-funded literacy programs for everyone will gain

(D) a need for well-funded literacy programs and everyone will gain

(E) why well-funded literacy programs being necessary for everyone in gaining

11The Portuguese musical tradition known as fado, or “fate,” has been called the Portuguese blues because of their songs that bemoan someone’s misfortune, especially the loss of romantic love.

ANSWERS AND EXPLANATIONS Explanation for Correct Answer C :  Choice (C) is correct. It avoids the error of the original by using the pronoun "its," which correctly refers to the noun "The Portuguese...fado."

(A) of their songs that bemoan someone’s

(B) of their songs bemoaning their

(C) its songs bemoan

(D) the songs that bemoaned

(E) of how it bemoans their

页码,6/21The Official SAT Online Course

2006-11-12file://E:\新建文件夹\f5.htm

UnRe

gistered

Page 43: SAT test 6

严禁用于商业用途!

Explanation for Incorrect Answer A :  Choice (A) involves an error in pronoun reference. The pronoun "their" does not refer to anything that comes before it.

Explanation for Incorrect Answer B :  Choice (B) involves an error in pronoun reference. The pronoun "their" does not refer to anything that comes before it.

Explanation for Incorrect Answer D :  Choice (D) creates a fragment. The subordinate clause "that...love" is not completed.

Explanation for Incorrect Answer E :  Choice (E) involves an error in pronoun reference. The pronoun "their" does not refer to anything that comes before it.

12

Every year, toy manufacturers groups of children into playrooms, observing

their choices of toys   new products will become the

ANSWERS AND EXPLANATIONS Corrected Sentence: Every year, toy manufacturers gather groups of children into playrooms, oberving their choices of toys and predicting which new products will become the most popular.

Explanation for Correct Answer B :  The error in this sentence occurs at (B), where there is an incomplete verb form. The word "as" cannot properly join the verbs "observing" and "predicting" to form the compound predicate. Instead, the conjunction "and" should be used.

Explanation for Incorrect Answer A :  There is no error at (A). The verb "gather" appropriately agrees with its subject, "toy manufacturers."

Explanation for Incorrect Answer C :  There is no error at (C). The relative pronoun "which" appropriately refers to the noun "new products" in describing the kinds of products that "will become most popular."

Explanation for Incorrect Answer D :  There is no error at (D). The adverb "most" properly modifies the adjective "popular."

Explanation for Incorrect Answer E :  There is an error in the sentence.

13

fifty years, we   radio communication for granted, but the

mere suggestion that we could communicate a fashion must once have

seemed outlandish. 

gather

as predicting which

most popular No error

During the last come to take

in such

No error

页码,7/21The Official SAT Online Course

2006-11-12file://E:\新建文件夹\f5.htm

UnRe

gistered

Page 44: SAT test 6

严禁用于商业用途!

ANSWERS AND EXPLANATIONS Corrected Sentence: During the last fifty years, we have come to take radio communication for granted, but the mere suggestion that we could communicate in such a fashion must once have seemed outlandish.

Explanation for Correct Answer B :  The error in this sentence occurs at (B), where there is an improper verb tense. The introductory prepositional phrase "During the last fifty years" establishes that the action of the main clause (taking radio communications for granted) takes place over the whole period. Therefore, the present-tense verb, "come" should be in the present perfect tense ("have come").

Explanation for Incorrect Answer A :  There is no error at (A). The preposition "During" is an appropriate and necessary part of the prepositional phrase "During the last fifty years."

Explanation for Incorrect Answer C :  There is no error at (C). The infinitive "to take" appropriately introduces its object, "radio communications."

Explanation for Incorrect Answer D :  There is no error at (D). The preposition "in" appropriately introduces the prepositional phrase "in such a fashion," and the adjective "such" appropriately modifies "a fashion."

Explanation for Incorrect Answer E :  There is an error in the sentence.

14

The tone of a recent city hall ordinance the blocking of

emergency vehicles in traffic jams a stern warning motorists. 

ANSWERS AND EXPLANATIONS Corrected Sentence: The uncompromising tone of a recent city call ordinance concerning the blocking of emergency vehicles in traffic jams carries a stern warning to motorists.

Explanation for Correct Answer C :  The error in this sentence occurs at (C), where there is subject-verb disagreement. The plural verb "carry" cannot refer to the singular subject, "The uncompromising tone." The singular verb "carries" is needed.

Explanation for Incorrect Answer A :  There is no error at (A). The adjective "uncompromising" appropriately modifies the noun "tone."

Explanation for Incorrect Answer B :  There is no error at (B). The participle "concerning" appropriately modifies the preceding noun "ordinance."

Explanation for Incorrect Answer D :  There is no error at (D). The preposition "to" properly links the noun phrase "a stern warning" to the noun "motorists."

Explanation for Incorrect Answer E :  There is an error in the sentence.

uncompromising concerning

carry to No error

页码,8/21The Official SAT Online Course

2006-11-12file://E:\新建文件夹\f5.htm

UnRe

gistered

Page 45: SAT test 6

严禁用于商业用途!

15

Formed by volcanic eruptions the last five million years, the Hawaiian Islands

an variety of species—many found on Earth.

 

ANSWERS AND EXPLANATIONS Corrected Sentence: Formed by volcanic eruptions over the last five million years, the Hawaiian Islands contain an incredibly wide variety of species--many found nowhere else on Earth.

Explanation for Correct Answer B :  The error in this sentence occurs at (B), where a sentence fragment is created. The use of the participle "containing," instead of the present-tense "contain," leaves the sentence without a main verb.

Explanation for Incorrect Answer A :  There is no error at (A). The preposition "over" properly introduces the prepositional phrase "over the last five million years," which establishes when the volcanic eruptions took place.

Explanation for Incorrect Answer C :  There is no error at (C). The adverb "incredibly" appropriately modifies the adjective "wide."

Explanation for Incorrect Answer D :  There is no error at (D). The phrase "nowhere else" properly indicates that the "wide variety of species" are found mainly in one place.

Explanation for Incorrect Answer E :  There is an error in the sentence.

16

Because the owl   virtually noiseless in flight,

  by the casual observer.  

ANSWERS AND EXPLANATIONS Corrected Sentence: Because the owl is usually nocturnal and is virtually noiseless in flight, it is seldom seen by the casual observer.

Explanation for Correct Answer B :  The error in this sentence occurs at (B), where there is an awkward construction. The phrase "and is" should be used in place of "plus being" to properly join one characteristic (nocturnal behavior) with the other (noiselessness in flight).

Explanation for Incorrect Answer A :  There is no error at (A). The singular verb, "is," agrees with its singular subject, "the owl," and the adverb "usually" properly modifies the adjective "nocturnal."

Explanation for Incorrect Answer C :  There is no error at (C). The pronoun "it" refers correctly to the noun "the owl."

over

containing incredibly wide nowhere else

No error

is usually nocturnal plus being

it is seldom seen No error

页码,9/21The Official SAT Online Course

2006-11-12file://E:\新建文件夹\f5.htm

UnRe

gistered

Page 46: SAT test 6

严禁用于商业用途!

Explanation for Incorrect Answer D :  There is no error at (D). The singular verb "is" agrees with its singular subject ("the owl"), and the adverb "seldom" appropriately modifies the verb "seen."

Explanation for Incorrect Answer E :  There is an error in the sentence.

17

An recycling center to the public, responsive to

community needs, and current federal regulations waste

disposal.  

ANSWERS AND EXPLANATIONS Corrected Sentence: An economical and efficient recycling center is accessible to the public, responsive to community needs, and complies with current federal regulations governing waste disposal.

Explanation for Correct Answer C :  The error in this sentence occurs at (C), where there is subject-verb disagreement. The plural verb "comply" does not agree with its singular subject ("recycling center").

Explanation for Incorrect Answer A :  There is no error at (A). The compound adjective, "economical and efficient," appropriately modifies "recycling center."

Explanation for Incorrect Answer B :  There is no error at (B). The singular verb "is" agrees with its singular subject, "recycling center," and the adjective "accessible" appropriately modifies "recycling center" to indicate one aspect (accessibility) of an "economical and efficient" recycling center.

Explanation for Incorrect Answer D :  There is no error at (D). The participle "governing" appropriately indicates the kind of regulations being discussed (those governing waste disposal).

Explanation for Incorrect Answer E :  There is an error in the sentence.

18

Jean Toomer was the author of Cane, a novel whose publication

  the beginning of the Harlem Renaissance, but also a

respected advisor . 

ANSWERS AND EXPLANATIONS Corrected Sentence: The sentence contains no error.

Explanation for Correct Answer E :  There is no error in this sentence.

economical and efficient is accessible

comply with governing

No error

not only

has been viewed as marking

among Quakers No error

页码,10/21The Official SAT Online Course

2006-11-12file://E:\新建文件夹\f5.htm

UnRe

gistered

Page 47: SAT test 6

严禁用于商业用途!

Explanation for Incorrect Answer A :  There is no error at (A). The phrase "not only" operates appropriately as the first part of the correlative construction "not only . . . but also."

Explanation for Incorrect Answer B :  There is no error at (B). The tense of the verb correctly indicates that the action described (the way the novel is viewed) is still going on.

Explanation for Incorrect Answer C :  There is no error at (C). The adverb "as" joins with the participle "marking" to produce an appropriate idiom.

Explanation for Incorrect Answer D :  There is no error at (D). The preposition "among" joins with the plural noun "Quakers" to correctly describe where Toomer was "a respected advisor."

19

Election results from upstate New York , but the results from New

York City   .  

ANSWERS AND EXPLANATIONS Corrected Sentence: Election results came in from upstate New York quite rapidly, but the results from New York City were known even faster.

Explanation for Correct Answer B :  The error in this sentence occurs at (B), where an adjective ("rapid") is incorrectly used to modify the verb "came." The adverb "rapidly" is needed.

Explanation for Incorrect Answer A :  There is no error at (A). The preposition "in" appropriately modifies the verb "came" to indicate where the election results came from.

Explanation for Incorrect Answer C :  There is no error at (C). The past tense of the verb is consistent with the tense previously established by the past-tense verb "came."

Explanation for Incorrect Answer D :  There is no error at (D). The adverb "even" appropriately modifies the adjective "faster" to indicate how quickly the results were known.

Explanation for Incorrect Answer E :  There is an error in the sentence.

20

we rely more and more on the Internet, need for effective security planning

and design data  

ANSWERS AND EXPLANATIONS Corrected Sentence: As we rely more and more on the Internet, the need for effective security planning and design to safeguard data has increased.

Explanation for Correct Answer B : 

came in quite rapid

were known even faster No error

As your

to safeguard has increased. No error

页码,11/21The Official SAT Online Course

2006-11-12file://E:\新建文件夹\f5.htm

UnRe

gistered

Page 48: SAT test 6

严禁用于商业用途!

The error in this sentence occurs at (B), where there is an error in pronoun use. There is nothing in the sentence to which the possessive pronoun "your" can logically refer.

Explanation for Incorrect Answer A :  There is no error at (A). The subordinating conjuntion "As" appropriately introduces the dependent clause ("As we rely . . . on the Internet").

Explanation for Incorrect Answer C :  There is no error at (C). The infinitive "to safeguard" appropriately indicates the purpose of "effective . . . design" (to safeguard data).

Explanation for Incorrect Answer D :  There is no error at (D). The tense of the verb "has increased" appropriately indicates that the action described (the increasing need for effective planning) is ongoing.

Explanation for Incorrect Answer E :  There is an error in the sentence.

21

The book a detailed and record happened

to the protestors. 

ANSWERS AND EXPLANATIONS Corrected Sentence: 

Explanation for Correct Answer E :  There is no error in this sentence.

Explanation for Incorrect Answer A :  There is no error at (A). The singular verb "is" agrees with its singular subject "the book," and the adverb "essentially" appropriately modifies the verb "is."

Explanation for Incorrect Answer B :  There is no error at (B). The adverbs "very" and "well" combine to appropriately modify the adjective "documented."

Explanation for Incorrect Answer C :  There is no error at (C). The preposition "of" combines with the relative pronoun "what" to produce an appropriate idiom indicating what was documented in the book.

Explanation for Incorrect Answer D :  There is no error at (D). The pronoun "each" combines with the preposition "of" to form an appropriate idiom linking "what happened" to the group to which it happened.

22

Experts permanently modifying eating and exercise habits rather than

for brief periods   weight. 

is essentially very well documented of what

each of No error

agree that

merely dieting are the key to controlling No error

页码,12/21The Official SAT Online Course

2006-11-12file://E:\新建文件夹\f5.htm

UnRe

gistered

Page 49: SAT test 6

严禁用于商业用途!

ANSWERS AND EXPLANATIONS Corrected Sentence: Experts agree that permanently modifying eating and exercise habits rather than merely dieting for brief periods is the key to controlling weight.

Explanation for Correct Answer C :  The error in this sentence occurs at (C), where there is an incorrect verb form. The plural verb "are" should be the singular "is" to agree with the singular gerund, "modifying eating and exercise habits."

Explanation for Incorrect Answer A :  There is no error at (A). The plural verb ("agree") agrees with its subject, "Experts," and the relative pronoun "that" appropriately introduces the phrase that immediately follows.

Explanation for Incorrect Answer B :  There is no error at (B). The gerund "dieting" is appropriately modified by the adverb "merely."

Explanation for Incorrect Answer D :  There is no error at (D). The preposition "to" properly links the noun "the key" with the gerund "controlling weight."

Explanation for Incorrect Answer E :  There is an error in the sentence.

23

The ability the plots of our dreams a skill, researchers , that

we can learn if to change recurrent dreams. 

ANSWERS AND EXPLANATIONS Corrected Sentence: The ability to control the plots of our dreams is a skill, researchers have shown, that we can learn if we want to change recurrent dreams.

Explanation for Correct Answer D :  The error in this sentence occurs at (D), where there is improper prounoun use. The second-person pronoun "you" should be changed to the first-person plural "we" to be consistent with the earlier use of "we."

Explanation for Incorrect Answer A :  There is no error at (A). The infinitive "to control" properly modifies the preceding noun, "The ability."

Explanation for Incorrect Answer B :  There is no error at (B). The singular verb "is" agrees with its singular subject, "The ability."

Explanation for Incorrect Answer C :  There is no error at (C). The verb tense appropriately establishes that the action described (what researchers have determined about the ability to control dreams) has already taken place.

Explanation for Incorrect Answer E :  There is an error in the sentence.

24

to control is have shown

you want No error

页码,13/21The Official SAT Online Course

2006-11-12file://E:\新建文件夹\f5.htm

UnRe

gistered

Page 50: SAT test 6

严禁用于商业用途!

swimming soccer, Evangelina proved time after time to be

competitor Juanita.  

ANSWERS AND EXPLANATIONS Corrected Sentence: In swimming as in soccer, Evangelina proved time after time to be an abler competitor than Juanita.

Explanation for Correct Answer B :  The error in this sentence occurs at (B), where there is an improper idiom. The preposition "in" should be used with the noun "soccer" to convey the idea that what happens "in swimming" is also what happens "in soccer."

Explanation for Incorrect Answer A :  There is no error at (A). The preposition "in" combines with the gerund "swimming" to create an appropriate idiom.

Explanation for Incorrect Answer C :  There is no error at (C). The comparative form of the adjective "able" is correct and properly modifies the noun "competitor."

Explanation for Incorrect Answer D :  There is no error at (D). The conjunction "than" is used correctly to introduce the object of the comparison ("Juanita").

Explanation for Incorrect Answer E :  There is an error in the sentence.

25

The common cold is indiscriminate diseases; no

  , millionaires and paupers, or athletes and couch

potatoes.  

ANSWERS AND EXPLANATIONS Corrected Sentence: 

Explanation for Correct Answer E :  There is no error in this sentence.

Explanation for Incorrect Answer A :  There is no error at (A). The pronoun "one" is used correctly to refer to "The common cold," and the preposition "of" appropriately introduces the prepositional phrase that modifies "The common cold."

Explanation for Incorrect Answer B :  There is no error at (B). The pronoun "it" correctly refers to the noun "The common cold," and the singular verb "makes" agrees with its singular subject, "it."

Explanation for Incorrect Answer C :  There is no error at (C). The noun "distinction" links appropriately with the preposition "between" to create an appropriate idiom.

In as to an abler

than No error

one of our most it makes

distinction between you and me

No error

页码,14/21The Official SAT Online Course

2006-11-12file://E:\新建文件夹\f5.htm

UnRe

gistered

Page 51: SAT test 6

严禁用于商业用途!

Explanation for Incorrect Answer D :  There is no error at (D). Both pronouns ("you" and "me") are properly in the objective case.

26

cookbooks, in his new book Chef Louis offers lengthy explanations

    basic cooking principles. 

ANSWERS AND EXPLANATIONS Corrected Sentence: As he does in his other cookbooks, in his new book Chef Louis offers lengthy explanations of what he considers to be basic cooking principles.

Explanation for Correct Answer A :  The error in this sentence occurs at (A), where there is an illogical comparison. It would be appropriate to compare what is in "his other cookbooks" with what is in "his new book," but instead the sentence illogically compares a thing ("his other cookbooks") with a person ("Chef Louis").

Explanation for Incorrect Answer B :  There is no error at (B). The preposition "of" combines with the pronoun "what" to create an appropriate idiom.

Explanation for Incorrect Answer C :  There is no error at (C). The pronoun "he" correctly refers to its antecedent "Chef Louis," and the singular verb "considers" agrees with its singular subject, "he."

Explanation for Incorrect Answer D :  There is no error at (D). The infinitive "to be" joins with the verb "considers" to appropriately indicate what the "lengthy explanations" are about.

Explanation for Incorrect Answer E :  There is an error in the sentence.

27

Paul Ecke, flower grower and hybridizer,   “Mr. Poinsettia” after

developing new varieties of the flower and it of

Christmas.  

ANSWERS AND EXPLANATIONS Corrected Sentence: Paul Ecke, flower grower and hybridizer, became known as "Mr. Poinsettia" after developing new varieties of the flower and pioneering it as a living symbol of Christmas.

Explanation for Correct Answer C :  The error in this sentence occurs at (C), where there are excess words. The preposition "by" is unnecessary.

Explanation for Incorrect Answer A : 

Like his other

of what he considers to be No error

became known as

by pioneering as a living symbol

No error

页码,15/21The Official SAT Online Course

2006-11-12file://E:\新建文件夹\f5.htm

UnRe

gistered

Page 52: SAT test 6

严禁用于商业用途!

There is no error at (A). The past-tense verb "became" appropriately establishes that Mr. Ecke was known a "Mr. Poinsettia" only after he developed "new varieties of the flower."

Explanation for Incorrect Answer B :  There is no error at (B). The adjective "known" appropriately modifies the subject of the sentence "Paul Ecke."

Explanation for Incorrect Answer D :  There is no error at (D). The prepositon "as" joins with the noun phrase "a living symbol" to create an appropriate idiom.

Explanation for Incorrect Answer E :  There is an error in the sentence.

28

Long   where just borrowed books, libraries

their images dramatically over the last few years.  

ANSWERS AND EXPLANATIONS Corrected Sentence: Long thought of as quiet, stuffy places where people just borrowed books, libraries have been changing their images dramatically over the last few years.

Explanation for Correct Answer B :  The error in this sentence occurs at (B), where there is noun-noun disagreement. To agree with the plural "libraries," the singular noun "place" should be changed to the plural "places."

Explanation for Incorrect Answer A :  There is no error at (A). The verb "thought" combines with the preposition "of" to create an appropriate idiom.

Explanation for Incorrect Answer C :  There is no error at (C). The plural noun "people" is logical here because more than one library is being talked about.

Explanation for Incorrect Answer D :  There is no error at (D). The present perfect progressive tense of the verb appropriately indicates that the action described may be ongoing.

Explanation for Incorrect Answer E :  There is an error in the sentence.

29

twentieth-century economic practices, be sufficiently

Keynesian theories, whether one agrees or not. 

ANSWERS AND EXPLANATIONS Corrected Sentence: To understand twentieth-century economic practices, one must be sufficiently familiar with Keynesian theories, whether one agrees with them or not.

Explanation for Correct Answer B :  The error in this sentence occurs at (B), where there is incorrect pronoun use. The

thought of as a quiet, stuffy place people

have been changing No error

To understand we must

familiar with with them No error

页码,16/21The Official SAT Online Course

2006-11-12file://E:\新建文件夹\f5.htm

UnRe

gistered

Page 53: SAT test 6

严禁用于商业用途!

first-person plural pronoun "we" is not consistent with the later pronoun "one."

Explanation for Incorrect Answer A :  There is no error at (A). The infinitive "To understand" appropriately indicates that a familiarity with Keynesian theories will facilitate one's understanding of "twentieth-century economic practices."

Explanation for Incorrect Answer C :  There is no error at (C). The adjective "familiar" joins with the preposition "with" to form an appropriate idiom.

Explanation for Incorrect Answer D :  There is no error at (D). In the prepositional phrase "with them," the object of the preposition, "them," must be in the objective case, as it is here.

Explanation for Incorrect Answer E :  There is an error in the sentence.

(1) Not many children leave elementary school and they have not heard of Pocahontas’ heroic rescue of John Smith from her own people, the Powhatans. (2) Generations of Americans have learned the story of a courageous Indian princess who threw herself between the Virginia colonist and the clubs raised to end his life. (3) The captive himself reported the incident. (4) According to that report, Pocahontas held his head in her arms and laid her own upon his to save him from death. 

(5) But can Smith’s account be trusted? (6) Probably it cannot, say several historians interested in dispelling myths about Pocahontas. (7) According to these experts, in his eagerness to find patrons for future expeditions, Smith changed the facts in order to enhance his image. (8) Portraying himself as the object of a royal princess’ devotion may have merely been a good public relations ploy. (9) Research into Powhatan culture suggests that what Smith described as an execution might have been merely a ritual display of strength. (10) Smith may have been a character in a drama in which even Pocahontas was playing a role. 

(11) As ambassador from the Powhatans to the Jamestown settlers, Pocahontas headed off confrontations between mutually suspicious parties.  (12) Later, after her marriage to colonist John Rolfe, Pocahontas traveled to England, where her diplomacy played a large part in gaining support for the Virginia Company. 

30What is the best way to deal with sentence 1 (reproduced below) ? Not many children leave elementary school and they have not heard of Pocahontas’ heroic rescue of John Smith from her own people, the Powhatans.

ANSWERS AND EXPLANATIONS Explanation for Correct Answer D :  Choice (D) is correct. It properly explains that most children hear the Pocahontas story before they leave elementary school.

Explanation for Incorrect Answer A :  Choice (A) is unsatisfactory because the original sentence connects the two main ideas--children leaving school and the Pocahontas story--with only the conjunction "and." The sentence thus offers no clue about the relationship between the two

(A) Leave it as it is

(B) Switch its position with that of sentence 2.

(C) Change “leave” to “have left.”

(D) Change “and they have not heard” to “without having heard.”

(E) Remove the comma and insert “known as the.”

页码,17/21The Official SAT Online Course

2006-11-12file://E:\新建文件夹\f5.htm

UnRe

gistered

Page 54: SAT test 6

严禁用于商业用途!

ideas.

Explanation for Incorrect Answer B :  Choice (B) is unsatisfactory because it is logical to give the names of the principal figures in a story or event before telling the story, not after--especially when the names are familiar.

Explanation for Incorrect Answer C :  Choice (C) is unsatisfactory because it repeats the error of the original in failing to explain the relationship between the children and the story.

Explanation for Incorrect Answer E :  Choice (E) is unsatisfactory because the original correctly refers to Pocahontas's tribe.

31 In context, which of the following is the best way to revise the underlined wording in order to combine sentences 3 and 4 (reproduced below)? The captive himself reported the incident. According to that report Pocahontas held his head in her arms and laid her own upon his to save him from death.

ANSWERS AND EXPLANATIONS Explanation for Correct Answer E :  Choice (E) is correct. The resulting sentence maintains the sense of the original while eliminating the redundancy.

Explanation for Incorrect Answer A :  Choice (A) is unsatisfactory because it contains an unclear referent: "which" seems to refer to the incident itself rather than to the report.

Explanation for Incorrect Answer B :  Choice (B) is unsatisfactory because the word "which" appears to refer to the incident, when it can logically refer only to the report of the incident.

Explanation for Incorrect Answer C :  Choice (C) is unsatisfactory because the word "consequently" suggests incorrectly that Smith's report is a consequence of the legend.

Explanation for Incorrect Answer D :  Choice (D) is unsatisfactory because it uses the unnecessary phrase "it seems" to relate a fact.

(A) The captive himself reported the incident, according to which

(B) Since then, the captive reported the incident, which said that

(C) Consequently, the captive himself reports that

(D) It seems that in the captive’s report of the incident he says that

(E) According to the captive’s own report of the incident,

32

Which of the following phrases is the best to insert at the beginning of sentence 10 to link it to sentence 9?

(A) Far from being in mortal danger,

(B) If what he says is credible,

(C) What grade school history never told you is this:

(D) They were just performing a ritual, and

(E) But quite to the contrary,

页码,18/21The Official SAT Online Course

2006-11-12file://E:\新建文件夹\f5.htm

UnRe

gistered

Page 55: SAT test 6

严禁用于商业用途!

ANSWERS AND EXPLANATIONS Explanation for Correct Answer A :  Choice (A) is correct. It links sentence 10 to the rest of the paragraph by explaining the harmlessness of the "ritual display" mentioned in the previous sentence (and thus clarifies the contrast between Smith's account and the probable facts).

Explanation for Incorrect Answer B :  Choice (B) is unsatisfactory because sentence 10 outlines a scenario that challenges Smith's "life-or-death" account, implying that Smith is not a credible source.

Explanation for Incorrect Answer C :  Choice (C) is unsatisfactory because the inserted phrase unhelpfully interrupts the connection between the "ritual display" introduced in sentence 9 and the explanation of it in sentence 10.

Explanation for Incorrect Answer D :  Choice (D) is unsatisfactory because the use of "and" implies that the "ritual" and the "drama" are two different events, whereas the "drama" actually refers to the "ritual display."

Explanation for Incorrect Answer E :  Choice (E) is unsatisfactory because nothing in sentence 10 is contrary to sentence 9; the latter sentence logically follows the former.

33 Which of the following best describes the relationship between sentences 9 and 10?

ANSWERS AND EXPLANATIONS Explanation for Correct Answer B :  Choice (B) is correct. Sentence 10 elaborates on the information about what may have really happened to Smith presented in sentence 9.

Explanation for Incorrect Answer A :  Choice (A) is unsatisfactory because sentence 10 offers only support for the claim made in sentence 9.

Explanation for Incorrect Answer C :  Choice (C) is unsatisfactory because the information in sentence 10 is not an "example"; rather, it is a reasoned clarification of what may have happened to Smith.

Explanation for Incorrect Answer D :  Choice (D) is unsatisfactory because nothing about sentence 10 contradicts sentence 9.

Explanation for Incorrect Answer E :  Choice (E) is unsatisfactory because sentence 10 does not make use of any new sources.

(A) Sentence 10 concludes that the theory mentioned in sentence 9 is wrong.

(B) Sentence 10 adds to information reported in sentence 9.

(C) Sentence 10 provides an example to illustrate an idea presented in sentence 9.

(D) Sentence 10 poses an argument that contradicts the point made in sentence 9.

(E) Sentence 10 introduces a new source that confirms the claims made in sentence 9.

页码,19/21The Official SAT Online Course

2006-11-12file://E:\新建文件夹\f5.htm

UnRe

gistered

Page 56: SAT test 6

严禁用于商业用途!

34 Which of the following would be the best to insert before sentence 11 to introduce the third paragraph?

ANSWERS AND EXPLANATIONS Explanation for Correct Answer C :  Choice (C) is correct. The third paragraph gives two detailed examples of Pocahontas's political successes in later life.

Explanation for Incorrect Answer A :  Choice (A) is unsatisfactory because the passage does not mention any of Pocahontas's shortcomings.

Explanation for Incorrect Answer B :  Choice (B) is unsatisfactory because focusing on the believability of historical facts is odd and unnecessary.

Explanation for Incorrect Answer D :  Choice (D) is unsatisfactory because the information in paragraph 3 deals primarily with Pocahontas's public life, not her private life.

Explanation for Incorrect Answer E :  Choice (E) is unsatisfactory because the third paragraph gives detailed information about Pocahontas that is not in dispute.

(A) It is crucial to consider the political successes as well as the shortcomings of Pocahontas.

(B) The Pocahontas of legend is the most interesting, but the historical Pocahontas is more believable.

(C) If legend has overemphasized the bravery of Pocahontas, it has underplayed her political talents.

(D) To really know Pocahontas, we must get beyond myth and legend to the real facts about her private life.

(E) Perhaps we will never really know the real Pocahontas.

35 What information is most logical to add immediately after sentence 12 ?

ANSWERS AND EXPLANATIONS Explanation for Correct Answer E :  Choice (E) is correct. This information is consistent with the information presented in the first two sentences of the paragraph.

Explanation for Incorrect Answer A :  Choice (A) is unsatisfactory because the first two sentences of the paragraph present events chronologically, and it would be illogical to describe Pocahontas's marriage, which occurred before the events described in sentence 12, at the end of the paragraph

Explanation for Incorrect Answer B :  Choice (B) is unsatisfactory because the passage has moved from a discussion of Smith's account to a discussion of Pocahontas's life; to return to Smith at this point would be illogical.

(A) How Rolfe and Pocahontas happened to meet and marry

(B) Details about other versions of the legend concerning John Smith

(C) Reasons for the confrontations between the Powhatans and the Jamestown settlers

(D) An account of Rolfe’s life and work in Virginia

(E) A brief summary of the other public events in Pocahontas’ life

页码,20/21The Official SAT Online Course

2006-11-12file://E:\新建文件夹\f5.htm

UnRe

gistered

Page 57: SAT test 6

严禁用于商业用途!

Explanation for Incorrect Answer C :  Choice (C) is unsatisfactory because this information would need to be presented directly after its introduction in sentence 11.

Explanation for Incorrect Answer D :  Choice (D) is unsatisfactory because the focus of the paragraph is Pocahontas, not her husband.

    

Back to Score Report  

Copyright © 2006 The College Board. All rights reserved. Privacy Policy Terms of Use Contact Us

 

页码,21/21The Official SAT Online Course

2006-11-12file://E:\新建文件夹\f5.htm

UnRe

gistered

Page 58: SAT test 6

严禁用于商业用途!

Help | Profile | My Organizer | My Bookmarks | Logout

Answers and Explanations

Test Sections

Section 1

Section 2

Section 4

Section 5

Section 6

Section 7

Section 8

Section 9

Section 10

Back to Score Report  

View Answers and Explanations     Online - Practice Test #6

1

The figure above shows five lines. If which of the following is NOT equal to 90 ?

ANSWERS AND EXPLANATIONS Explanation for Correct Answer A :  Choice (A) is correct. Since , and since the angle with measure and the

angle with measure are corresponding angles, . Therefore, is not

equal to .

Explanation for Incorrect Answer B :  Choice (B) is not correct. Since the angle with measure is supplementary to a

right angle, .

Explanation for Incorrect Answer C :  Choice (C) is not correct. Since , and since the angle with measure and

the angle with measure are corresponding angles, . Since the angle with

measure is supplementary to a right angle, it follows that . So .

Explanation for Incorrect Answer D :  Choice (D) is not correct. Since , the angle with measure and a right

angle are corresponding angles. So .

Explanation for Incorrect Answer E :  Choice (E) is not correct. Since the angle with measure is supplementary to a right angle, it follows that .

(A) r

(B) s

(C) t

(D) u

(E) v

2 Which of the following is divisible by 3 and by 5 but is not divisible by 10 ?

(A) 30

(B) 35

(C) 40

页码,1/11The Official SAT Online Course

2006-11-12file://E:\新建文件夹\f6.htm

UnRe

gistered

Page 59: SAT test 6

严禁用于商业用途!

ANSWERS AND EXPLANATIONS Explanation for Correct Answer D : 

Choice (D) is correct. Since , is divisible by and . However,

is not divisible by .

Explanation for Incorrect Answer A :  Choice (A) is not correct. Since , is divisible by and .

However, since , is also divisible by .

Explanation for Incorrect Answer B :  Choice (B) is not correct. Since , is divisible by but not divisible

by .

Explanation for Incorrect Answer C : 

Choice (C) is not correct. Since , is divisible by but not divisible

by .

Explanation for Incorrect Answer E : 

Choice (E) is not correct. Since , is divisible by and .

However, since , is also divisible by .

(D) 45

(E) 60

3

The table above gives values of the function f for several values of t. If the graph of f is a line, Which of the following defines ?

ANSWERS AND EXPLANATIONS Explanation for Correct Answer D : 

Choice (D) is correct. Since is a linear function, it has the form ,

where and are constants. From the table, you have , so

, and thus . Since , the equation

holds, so , and thus . Therefore, .

Explanation for Incorrect Answer A : 

Choice (A) is not correct. From the table, you have . If were defined

, then . But then would be equal to , which is false. So

cannot define .

(A)

(B)

(C)

(D)

(E)

页码,2/11The Official SAT Online Course

2006-11-12file://E:\新建文件夹\f6.htm

UnRe

gistered

Page 60: SAT test 6

严禁用于商业用途!

Explanation for Incorrect Answer B : 

Choice (B) is not correct. From the table, you have . If were defined as

, then . But then would be equal to , which is false. So

cannot define .

Explanation for Incorrect Answer C : 

Choice (C) is not correct. From the table, you have . If were defined as

, then . But then would be equal to , which is false. So

cannot define .

Explanation for Incorrect Answer E : 

Choice (E) is not correct. From the table, you have . If were defined

as , then . But then would be equal to , which is

false. So cannot define .

4

In the figure above, the intersection of ray and ray is

ANSWERS AND EXPLANATIONS Explanation for Correct Answer B :  Choice (B) is correct. Ray is the set whose members are point and all

points to the right of on the line. Ray is the set whose members are point and all points to the left of on the line. The intersection of these two sets is

the set whose members are points and and all points on the line between and . This set of points is segment .

Explanation for Incorrect Answer A :  Choice (A) is not correct. Segment is the set whose members are points

and and all points on the line between and . Ray is the set whose

members are point and all points to the left of on the line. Any point between

and on the line is a member of segment but not a member of ray .

Therefore, segment cannot be the intersection of ray and ray .

Explanation for Incorrect Answer C :  Choice (C) is not correct. Ray is the set whose members are point and all

points to the right of on the line. Ray is the set whose members are point and all points to the left of on the line. Any point on the line to the right of

point is a member of ray , but not a member of ray . Therefore, ray

cannot be the intersection of ray and ray .

Explanation for Incorrect Answer D :  Choice (D) is not correct. Ray is the set whose members are point and all

points to the right of on the line. Ray is the set whose members are point and all points to the left of on the line. Any point on the line to the left of

point is a member of ray , but not a member of ray . Therefore, ray

cannot be the intersection of ray and ray .

Explanation for Incorrect Answer E : 

(A) Segment

(B) Segment

(C) Ray

(D) Ray

(E) Line

页码,3/11The Official SAT Online Course

2006-11-12file://E:\新建文件夹\f6.htm

UnRe

gistered

Page 61: SAT test 6

严禁用于商业用途!

Choice (E) is not correct. The intersection of ray and ray is the set whose

members are points and and all points on the line between and . Any

point on line to the left of point or to the right of point is not in the

intersection of ray and ray . Therefore, line cannot be the

intersection of ray and ray .

5

According to the graph above, if there are 6,000 registered voters aged 60 and over in Washington County, how many registered voters are under the age of 30?

ANSWERS AND EXPLANATIONS Explanation for Correct Answer E :  Choice (E) is correct. According to the graph, of the registered voters in

Washington County are age or over. If is the total number of registered

voters in Washington County, then of is . So .

Solving for gives . Of all the registered voters, are of ages

to , and are under age . Thus, of the registered voters are

under the age of ; this is of , which is .

Explanation for Incorrect Answer A :  Choice (A) is not correct. From the explanation for the correct response (E), you have that there are a total of registered voters, and from the graph, you

have that of these registered voters are under the age . Since of

is , the number of registered voters under the age

of is . However, you were to find the number of registered voters under

the age of .

Explanation for Incorrect Answer B :  Choice (B) is not correct. See the explanation for the correct response (E).

Explanation for Incorrect Answer C :  Choice (C) is not correct. See the explanation for the correct response (E).

Explanation for Incorrect Answer D :  Choice (D) is not correct. From the explanation for the correct response (E), you have that there are a total of registered voters, and from the graph, you

have that of these registered voters are of ages to . Since of

(A) 1,000

(B) 2,000

(C) 3,000

(D) 4,000

(E) 5,000

页码,4/11The Official SAT Online Course

2006-11-12file://E:\新建文件夹\f6.htm

UnRe

gistered

Page 62: SAT test 6

严禁用于商业用途!

is , the number of registered voters of ages of to

is . However, you were to find the number of registered voters under

the age of , which includes those under the age of .

6

Based on the graph of the function f above, what are the values of x for which f(x) is positive?

ANSWERS AND EXPLANATIONS Explanation for Correct Answer C :  Choice (C) is correct. From the graph of , you can see that for any value of

strictly between and , the value of is greater than . For any value of

greater than and less than or equal to , the value of is greater than .

For any other value of between and inclusive, the value of is less

than or equal to . So the values of for which is positive are those for

which or .

Explanation for Incorrect Answer A :  Choice (A) is not correct. For the values of such that , the value of

is positive. However, if , the value of is negative.

Explanation for Incorrect Answer B :  Choice (B) is not correct. The values of for which or are

the values of for which is negative.

Explanation for Incorrect Answer D :  Choice (D) is not correct. For some values of in the interval the

value of is positive. However, this is not true for all values of in that

interval. For example, if , then is in the interval , but you can

see from the graph that .

Explanation for Incorrect Answer E :  Choice (E) is not correct. For some values of in the interval the value

of is positive. However, this is not true for all values of in that interval. For

example, if , then is in the interval but you can see from the

graph that .

(A) or

(B) or

(C) or

(D)

(E)

页码,5/11The Official SAT Online Course

2006-11-12file://E:\新建文件夹\f6.htm

UnRe

gistered

Page 63: SAT test 6

严禁用于商业用途!

7 Bernardo drives to work at an average speed of 50 miles per hour and returns along the same route at an average speed of 25 miles per hour. If his total travel time is 3 hours, what is the total number of miles in the round-trip?

ANSWERS AND EXPLANATIONS Explanation for Correct Answer C :  Choice (C) is correct. If Bernardo’s route to work is miles long, and if he takes

hours to drive to work at an average speed of miles per hour, then

. His total travel time going to and from work is hours, so he takes

hours to drive back. Since he returns along the same route, .

Therefore, . Solving this equation gives . Since

, the value of is . The total number of miles in the round-trip is

thus .

Explanation for Incorrect Answer A :  Choice (A) is not correct. See the explanation for the correct response (C).

Explanation for Incorrect Answer B :  Choice (B) is not correct. See the explanation for the correct response (C). (Keep in mind that Bernardo’s average speed for the round-trip was not the average of

and miles per hour. Since he spent hour traveling at miles per hour

and hours traveling at miles per hour, he spent twice as much time at the

lower speed. Therefore, his average speed for the round-trip was less than miles per hour.)

Explanation for Incorrect Answer D :  Choice (D) is not correct. See the explanation for the correct response (C).

Explanation for Incorrect Answer E :  Choice (E) is not correct. The number of miles Bernardo traveled each way , but

the total number of miles in the round-trip was .

(A) 225

(B) 112.5

(C) 100

(D) 62.5

(E) 50

8

If x and y are integers such that and which of the following could

be true?

I.

II.

III.

ANSWERS AND EXPLANATIONS Explanation for Correct Answer C : 

(A) I only

(B) II only

(C) I and III only

(D) II and III only

(E) I, II, and III

页码,6/11The Official SAT Online Course

2006-11-12file://E:\新建文件夹\f6.htm

UnRe

gistered

Page 64: SAT test 6

严禁用于商业用途!

Choice (C) is correct. If is an integer such that , then or

. So could be , which means could be true. If is an integer such

that , then must be , so cannot be true. could be true,

because could be and is , in which case . So

could be true and could be true, but cannot be true.

Explanation for Incorrect Answer A :  Choice (A) is not correct. From the explanation for the correct answer, could be true, but could also be true.

Explanation for Incorrect Answer B :  Choice (B) is not correct. From the explanation for the correct answer, cannot be true. The value of must be . Also, could be true and could be true.

Explanation for Incorrect Answer D :  Choice (D) is not correct. From the explanation for the correct answer, cannot be true. Also, could be true.

Explanation for Incorrect Answer E :  Choice (E) is not correct. From the explanation for the correct answer, could be true and could be true. However, cannot be true. The value of must be

.

9When a certain number is multiplied by and the product is then multiplied by 32,

the result is 60. What is the number?

Your Response:  

Correct Response(s):  7.5, 15/2

Explanation:  

The correct answer is or . If is the number, then when is

multiplied by and the product is then multiplied by , the result is . So

. Therefore, , so the number is .

10 What is the greatest integer value of x for which

Your Response:  

Correct Response(s):  9

Explanation:  

The correct answer is . If , then , and so . The

greatest integer value of for which is .

11An object thrown upward from a height of h feet with an initial velocity of v feet per

second will reach a maximum height of feet. If the object is thrown upward

from a height of 6 feet with an initial velocity of 32 feet per second, what will be its maximum height, in feet?

页码,7/11The Official SAT Online Course

2006-11-12file://E:\新建文件夹\f6.htm

UnRe

gistered

Page 65: SAT test 6

严禁用于商业用途!

Your Response:  

Correct Response(s):  22

Explanation:  

The correct answer is . If the object is thrown upward from a height of feet,

with an initial velocity of feet per second, then and in the

formula . The formula gives the maximum height, in feet, that the object

will reach, so this height, in feet, is .

12 The three angles of a triangle have measures of and , where If x

and y are integers, what is one possible value of

Your Response:  

Correct Response(s):  3 or 6 or 9 or 12

Explanation:  

The correct answer is , , , or . The sum of the measures of the three

angles of a triangle is equal to so If

then so Since and are positive integers and

it follows that must be a multiple of

. So the only possible values for are , , , or .

13

The incomplete table above is an expense sheet for Carmen’s business trip. If her hotel expenses were the same each day, what were her total expenses for Friday, in dollars? (Disregard the $ sign when gridding your answer.)

Your Response:  

Correct Response(s):  96

Explanation:  

The correct answer is . If Carmen’s hotel expenses were dollars each day,

then her total expenses in dollars for the trip were . The table

gives her total expenses as , so . Solve this

equation for : , so , and . Carmen’s total

expenses for Friday, in dollars, were .

14

页码,8/11The Official SAT Online Course

2006-11-12file://E:\新建文件夹\f6.htm

UnRe

gistered

Page 66: SAT test 6

严禁用于商业用途!

In above, and What is the length of

Your Response:  

Correct Response(s):  8

Explanation:  

is the correct answer. From the information in the figure, you know that

is a right triangle. Since and , by the Pythagorean theorem, the

length of is . Since is a right angle, is

also a right angle, and is a right triangle. Since and ,

by the Pythagorean theorem, the length of is

.

15 On Monday morning Mr. Smith had a certain amount of money that he planned to spend during the week. On each subsequent morning, he had one fourth the amount of the previous morning. On Saturday morning, 5 days later, he had $1. How many dollars did Mr. Smith originally start with on Monday morning? (Disregard the $ sign when gridding your answer.)

Your Response:  

Correct Response(s):  1024

Explanation:  

The correct answer is . If Mr. Smith had dollars on Monday morning and

had one fourth that amount on Tuesday morning, then he had dollars on

Tuesday morning. On Wednesday morning, he had one fourth the amount he had

on Tuesday morning, or dollars. Continuing in this way, you see that he

had dollars on Saturday morning. If he had on

Saturday morning, then . So .

16The median of a list of 99 consecutive integers is 60. What is the greatest integer in the list?

Your Response:  

Correct Response(s):  109

Explanation:  

页码,9/11The Official SAT Online Course

2006-11-12file://E:\新建文件夹\f6.htm

UnRe

gistered

Page 67: SAT test 6

严禁用于商业用途!

The correct answer is . If the median of a list of consecutive integers is

, then there are integers in the list that are less than and integers

in the list that are greater than . Since the integers are consecutive, the

greatest integer in the list is .

17 When the positive integer m is divided by 5, the remainder is 3. What is the remainder when 20m is divided by 25?

Your Response:  

Correct Response(s):  10

Explanation:  

The correct answer is . When the positive integer is divided by , the

remainder is . So there is a positive integer such that . Then

. Therefore, the remainder is

when is divided by .

18

The figure above shows three squares with sides of length 5, 7, and x, respectively. If A, B, and C lie on line what is the value of x ?

Your Response:  

Correct Response(s):  49/5, 9.8

Explanation:  

The correct answer is or . From the figure and the information in the question, you can draw the figure above. Triangle is similar to triangle

, because they are right triangles with . To find ,

you can set up and solve the proportion . You have and

, so . Therefore, , so , and

.

    

Back to Score Report  

Copyright © 2006 The College Board. All rights reserved. Privacy Policy Terms of Use Contact Us

页码,10/11The Official SAT Online Course

2006-11-12file://E:\新建文件夹\f6.htm

UnRe

gistered

Page 68: SAT test 6

严禁用于商业用途!

 

页码,11/11The Official SAT Online Course

2006-11-12file://E:\新建文件夹\f6.htm

UnRe

gistered

Page 69: SAT test 6

严禁用于商业用途!

Help | Profile | My Organizer | My Bookmarks | Logout

Answers and Explanations

Test Sections

Section 1

Section 2

Section 4

Section 5

Section 6

Section 7

Section 8

Section 9

Section 10

Back to Score Report  

View Answers and Explanations     Online - Practice Test #6

1 The success of Notes of a Native Son ------- author James Baldwin as one of the most ------- essayists of his time.

ANSWERS AND EXPLANATIONS Explanation for Correct Answer B :  Choice (B) is correct. “Establish” means to cause to be recognized and accepted. “Prominent” means widely and popularly known. If one were to insert these terms into the text, the sentence would read “The success of Notes of a Native Son established author James Baldwin as one of the most prominent essayists of his time.” The success of Notes of a Native Son gave Baldwin recognition and acceptance within literary circles.

Explanation for Incorrect Answer A :  Choice (A) is incorrect. “Buoyed” means supported or uplifted. “Irrelevant” means unrelated or insignificant. If one were to insert these terms into the text, the sentence would read “The success of Notes of a Native Son buoyed author James Baldwin as one of the most irrelevant essayists of his time.” Success can enhance a writer’s reputation, but it is illogical to claim that it would cause Baldwin to become insignificant.

Explanation for Incorrect Answer C :  Choice (C) is incorrect. “Surrendered” means abandoned. “Prolific” means to produce abundant works or results. If one were to insert these terms into the text, the sentence would read “The success of Notes of a Native Son surrendered author James Baldwin as one of the most prolific essayists of his time. Many successful authors are prolific, but it does not make sense to say that success abandoned Baldwin as a “prolific” essayist.

Explanation for Incorrect Answer D :  Choice (D) is incorrect. “Decried” means denounced or belittled. “Cynical” means negative or pessimistic. If one were to insert these terms into the text, the sentence would read “The success of Notes of a Native Son decried author James Baldwin as one of the most cynical essayists of his time.” Some critics may have criticized Baldwin for being cynical, but a book cannot denounce its author.

Explanation for Incorrect Answer E :  Choice (E) is incorrect. “Categorized” means classified, and “mundane” means ordinary. If one were to insert these terms into the text, the sentence would read “The success of Notes of a Native Son categorized author James Baldwin as one of the most mundane essayists of his time.” It is unlikely that success would classify or label an author as ordinary.

(A) buoyed . . irrelevant

(B) established . . prominent

(C) surrendered . . prolific

(D) decried . . cynical

(E) categorized . . mundane

2 In many parts of the world, people use rice as a central rather than a ------- part of their daily diets.

(A) pivotal

页码,1/21The Official SAT Online Course

2006-11-12file://E:\新建文件夹\f7.htm

UnRe

gistered

Page 70: SAT test 6

严禁用于商业用途!

ANSWERS AND EXPLANATIONS Explanation for Correct Answer D :  Choice (D) is correct. "Supplementary" means something added or in addition to. If one were to insert this term into the text, the sentence would read "In many parts of the world, people use rice as a central rather than a supplementary part of their daily diets." The phrase "rather than" indicates that the blanked word will mean the opposite of the word "central." In this context, "supplementary" does indeed have the opposite meaning of "central."

Explanation for Incorrect Answer A :  Choice (A) is incorrect. "Pivotal" means being of vital or significant importance. If one were to insert this term into the text, the sentence would read "In many parts of the world, people use rice as a central rather than a pivotal part of their daily diets." "Pivotal" and "central" are synonymous, thus it makes little sense to claim that people use rice as a central rather than a "pivotal" part of their diet.

Explanation for Incorrect Answer B :  Choice (B) is incorrect. "Ritualistic" means advocating or practicing ritual. If one were to insert this term into the text, the sentence would read "In many parts of the world, people use rice as a central rather than a ritualistic part of their daily diets." A "ritualistic," or ceremonial, use of rice does not necessarily mean that the rice is not a "central" part of the diet.

Explanation for Incorrect Answer C :  Choice (C) is incorrect. "Salient" means prominent. If one were to insert this term into the text, the sentence would read "In many parts of the world, people use rice as a central rather than a salient part of their daily diets." A "salient" part of a diet would be an important part. It is illogical to contrast the use of rice as a "central" part of the diet with the use of rice as an important part of the diet.

Explanation for Incorrect Answer E :  Choice (E) is incorrect. "Solemn" means somberly or gravely impressive. If one were to insert this term into the text, the sentence would read "In many parts of the world, people use rice as a central rather than a solemn part of their daily diets." It makes little sense to describe rice as a "solemn," or somber, part of a diet.

(B) ritualistic

(C) salient

(D) supplementary

(E) solemn

3

Victor gained a reputation for being a ------- because he constantly bullied other children.

ANSWERS AND EXPLANATIONS Explanation for Correct Answer B :  Choice (B) is correct. "Ruffian" describes a person who is a hoodlum or bully. If one were to insert this term into the text, the sentence would read "Victor gained a reputation for being a ruffian because he constantly bullied other children." Bullying other children is precisely the behavior that would give Victor a reputation as a ruffian.

(A) bungler

(B) ruffian

(C) stickler

(D) daredevil

(E) naysayer

页码,2/21The Official SAT Online Course

2006-11-12file://E:\新建文件夹\f7.htm

UnRe

gistered

Page 71: SAT test 6

严禁用于商业用途!

Explanation for Incorrect Answer A :  Choice (A) is incorrect. "Bungler" describes a person who is clumsy or inept in behavior. If one were to insert this term into the text, the sentence would read "Victor gained a reputation for being a bungler because he constantly bullied other children." The claim that Victor had a reputation for being a "bungler" does not fit with the claim that "he bullied other children," as a bully is not necessarily clumsy.

Explanation for Incorrect Answer C :  Choice (C) is incorrect. "Stickler" describes a person who insists on something unyieldingly. If one were to insert this term into the text, the sentence would read "Victor gained a reputation for being a stickler because he constantly bullied other children." Although Victor may be a "stickler," or perfectionist, his reputation is based on his tendency to bully other children.

Explanation for Incorrect Answer D :  Choice (D) is incorrect. "Daredevil" describes a person who is recklessly bold. If one were to insert this term into the text, the sentence would read "Victor gained a reputation for being a daredevil because he constantly bullied other children." While Victor may be a "daredevil," or risktaker, the sentence offers no evidence of this.

Explanation for Incorrect Answer E :  Choice (E) is incorrect. "Naysayer" describes a person who opposes in order to take a pessimistic view. If one were to insert this term into the text, the sentence would read "Victor gained a reputation for being a naysayer because he constantly bullied other children." The sentence offers no evidence that Victor is a "naysayer," or contrarian.

4 Paradoxically, the senator was both a ------- and -------: she publicly defended the rights and wisdom of the people, but she often spoke with a disdainful air of superiority.

ANSWERS AND EXPLANATIONS Explanation for Correct Answer C :  Choice (C) is correct. A "populist" is an advocate for the interests and rights of the common people. An "elitist" believes that certain persons deserve favored treatment by virtue of their perceived superiority. If one were to insert these terms into the text, the sentence would read "Paradoxically, the senator was both a populist and an elitist: she publicly defended the rights and wisdom of the people, but she often spoke with a disdainful air of superiority." The term "paradoxically" indicates that the two missing terms will be contradictory. The senator's behavior is inconsistent because she supports common people publically, while believing in the superiority of certain classes of people.

Explanation for Incorrect Answer A :  Choice (A) is incorrect. A "demagogue" is a leader who obtains power by means of impassioned appeals to the emotions and prejudices of the populace. A "maverick" is one that resists adherence to a group. If one were to insert these terms into the text, the sentence would read "Paradoxically, the senator was both a demagogue and a maverick: she publicly defended the rights and wisdom of the people, but she often spoke with a disdainful air of superiority." A "demagogue," a person who stirs up emotion, can also be a "maverick" or independent politician. The two are not necessarily opposed to one another, and so the behavior of a person who exhibits both of these characteristics would not logically be described as paradoxical.

Explanation for Incorrect Answer B :  Choice (B) is incorrect. "Conservative" is one favoring traditional values and views. "Anarchist" is an advocate of political disorder and confusion. If one were to insert these terms into the text, the sentence would read "Paradoxically, the senator was both a conservative and an anarchist: she publicly defended the rights and wisdom of the people, but she often spoke with a disdainful air of superiority." A

(A) demagogue . . a maverick

(B) conservative . . an anarchist

(C) populist . . an elitist

(D) moderate . . a reactionary

(E) partisan . . a snob

页码,3/21The Official SAT Online Course

2006-11-12file://E:\新建文件夹\f7.htm

UnRe

gistered

Page 72: SAT test 6

严禁用于商业用途!

"conservative" politician does not necessarily defend the rights of the common people. An "anarchist" opposes the idea of government and is unlikely to hold a government office.

Explanation for Incorrect Answer D :  Choice (D) is incorrect. "Moderate" is one who holds average views or opinions. "Reactionary" means an opponent of progress or liberalism. If one were to insert these terms into the text, the sentence would read "Paradoxically, the senator was both a moderate and a reactionary: she publicly defended the rights and wisdom of the people, but she often spoke with a disdainful air of superiority." "Moderate," or middle-ground, and "reactionary" are not the polar opposites implied by the term "paradoxically."

Explanation for Incorrect Answer E :  Choice (E) is incorrect. "Partisan" is a supporter or proponent of a party, cause, faction, person, or idea. A "snob" looks down on people he or she regards as socially inferior. If one were to insert these terms into the text, the sentence would read "Paradoxically, the senator was both a partisan and snob: she publicly defended the rights and wisdom of the people, but she often spoke with a disdainful air of superiority." While a "snob" would speak with an air of superiority, it makes little sense to contrast a "snob" with a "partisan," or someone favoring a particular political party. Furthermore, a partisan would not necessarily defend the rights of the common people.

5 The geologist speculated that eons ago, before the area was -------, the present-day island was actually a hilltop in a vast forest.

ANSWERS AND EXPLANATIONS Explanation for Correct Answer A :  Choice (A) is correct. "Inundated" means to cover completely as in a flood. If one were to insert this term into the text, the sentence would read "The geologist speculated that eons ago, before the area was inundated, the present-day island was actually a hilltop in a vast forest." The words "before" and "actually" in this sentence suggest that something happened to transform the hilltop into an island. An inundation of water would be the most logical cause of such a transformation.

Explanation for Incorrect Answer B :  Choice (B) is incorrect. "Situated" means having a location. If one were to insert this term into the text, the sentence would read "The geologist speculated that eons ago, before the area was situated, the present-day island was actually a hilltop in a vast forest." The location, or where the island is "situated," has not changed.

Explanation for Incorrect Answer C :  Choice (C) is incorrect. "Rejuvenated" means restored to an original or new condition. If one were to insert this term into the text, the sentence would read "The geologist speculated that eons ago, before the area was rejuvenated, the present-day island was actually a hilltop in a vast forest." While parts of the area may have been rejuvenated, or restored, this would not have turned the area from a hilltop into an island.

Explanation for Incorrect Answer D :  Choice (D) is incorrect. "Supplanted" means displaced and substituted for. If one were to insert this term into the text, the sentence would read "The geologist speculated that eons ago, before the area was supplanted, the present-day island was actually a hilltop in a vast forest." The information in the sentence does not support the idea that the area was supplanted, or displaced.

Explanation for Incorrect Answer E :  Choice (E) is incorrect. "Excavated" means removed or exposed by digging. If one were to insert this term into the text, the sentence would read "The geologist speculated that eons ago, before the area was excavated, the present-day island

(A) inundated

(B) situated

(C) rejuvenated

(D) supplanted

(E) excavated

页码,4/21The Official SAT Online Course

2006-11-12file://E:\新建文件夹\f7.htm

UnRe

gistered

Page 73: SAT test 6

严禁用于商业用途!

was actually a hilltop in a vast forest." Although the area may have been excavated, or exhumed, this does not explain how the hilltop became an island.

Passage 1

Passage 2

      Any wildlife biologist can tell you how many deer

 a given area can support—how much browse there is

 for the deer to eat before they begin to suppress the

Linereproduction of trees, before they begin to starve in

5the winter. Any biologist can calculate how many

  wolves a given area can support too, in part by

 counting the number of deer. And so on, up and

 down the food chain. It’s not an exact science, but

 it comes pretty close—at least compared to figuring

10out the carrying capacity of Earth for human beings,

 which is an art so dark that anyone with any sense

  stays away from it.

      Estimates of the number of humans that Earth can

 sustain have ranged in recent decades from fewer than

15a billion to more than a trillion. Such elasticity is prob-

 ably unavoidable, since “carrying capacity” is essentially

 a subjective term. It makes little sense to talk about carry-

 ing capacity in relationship to humans, who are capable of

 adapting and altering both their culture and their physical

20environment, and can thus defy any formula that might

 settle the matter. The number of people that Earth can

 support depends on how we on Earth want to live, on

 what we want to consume, and on what we regard as

页码,5/21The Official SAT Online Course

2006-11-12file://E:\新建文件夹\f7.htm

UnRe

gistered

Page 74: SAT test 6

严禁用于商业用途!

  a crowd.

6 Both passages support which of the following conclusions about Earth’s carrying capacity for humans?

ANSWERS AND EXPLANATIONS Explanation for Correct Answer B :  Choice (B) is correct. Both passages discuss the difficulty of accurately estimating how many people the Earth can support. Passage 1 asserts that estimating the Earth's "carrying capacity" is a "dark art," and Passage 2 explains why such estimates are problematic.

Explanation for Incorrect Answer A :  Choice (A) is incorrect. Passage 2 asserts that biologists' estimations of the Earth's "carrying capacity" for humans cover a broad range, but neither author claims that these existing "estimates" are too low.

Explanation for Incorrect Answer C :  Choice (C) is incorrect. Nothing in these passages suggests that the debate regarding human "carrying capacity" is a new one.

Explanation for Incorrect Answer D :  Choice (D) is incorrect. Both passages clearly question the value of estimating the Earth's "carrying capacity" for humans. Passage 1 declares that "anyone with any sense stays away" from the concept, and Passage 2 says that "it makes little sense to talk about carrying capacity in relationship to humans."

Explanation for Incorrect Answer E :  Choice (E) is incorrect. The first passage does not discuss recent "technological innovations" at all, and the second passage supports this conclusion indirectly at best.

(A) It is routinely underestimated by biologists.

(B) It cannot be easily determined, given numerous variables and unknowns.

(C) It has only recently become the subject of considerable scientific debate.

(D) It is a valuable concept despite its apparent shortcomings.

(E) It has increased as a result of recent technological innovations.

7The author of Passage 1 refers to “Any wildlife biologist” in line 1 and “Any biologist” in line 5 to emphasize the point that

ANSWERS AND EXPLANATIONS Explanation for Correct Answer A :  Choice (A) is correct. The author of Passage 1 contrasts the ease of calculating how many animals an area can support with the difficulty of making such a calculation for people. The use of the word "any" conveys the author's belief that predicting an animal population's "carrying capacity" is a relatively simple task for biologists.

Explanation for Incorrect Answer B :  Choice (B) is incorrect. Passage 1 does not include any reference to the "general public."

(A) a particular type of calculation can be made with great confidence

(B) scientific findings often meet with resistance from the general public

(C) certain beliefs are rarely questioned by scientists

(D) most biologists are concerned with issues related to wildlife mortality

(E) all biologists must be skilled at applying mathematical formulas

页码,6/21The Official SAT Online Course

2006-11-12file://E:\新建文件夹\f7.htm

UnRe

gistered

Page 75: SAT test 6

严禁用于商业用途!

Explanation for Incorrect Answer C :  Choice (C) is incorrect. The author questions biologists' ability to accurately estimate the Earth's "carrying capacity" for humans but makes no reference to "beliefs" of any kind.

Explanation for Incorrect Answer D :  Choice (D) is incorrect. It may be true that the majority of biologists care about "wildlife mortality," but the author of Passage 1 makes no such claim.

Explanation for Incorrect Answer E :  Choice (E) is incorrect. While one might infer that the biologists mentioned in Passage 1 need to be skilled in mathematics, this is certainly not a point of emphasis in the passage.

8 Both authors would agree that the “Estimates” (Passage 2, line 13) are

ANSWERS AND EXPLANATIONS Explanation for Correct Answer E :  Choice (E) is correct. Passage 2 highlights the "elasticity" of scientists' "estimates," suggesting that such figures are fundamentally "unreliable." The author of Passage 1 echoes this assertion in describing the estimation of the Earth's "carrying capacity" for humans as an inexact science.

Explanation for Incorrect Answer A :  Choice (A) is incorrect. Passage 2 asserts that biologists' estimations of the Earth's "carrying capacity" for humans cover a broad range, but neither author claims that these existing estimates are too high.

Explanation for Incorrect Answer B :  Choice (B) is incorrect. Neither author claims that the "estimates" in Passage 2 are generally "undocumented."

Explanation for Incorrect Answer C :  Choice (C) is incorrect. While the authors question the accuracy of certain carrying capacity "estimates," there is no suggestion that these figures have been "misunderstood."

Explanation for Incorrect Answer D :  Choice (D) is incorrect. Neither author ascribes a political motivation to the scientists responsible for the "estimates" mentioned in Passage 2.

(A) overly generous

(B) largely undocumented

(C) often misunderstood

(D) politically motivated

(E) essentially unreliable

9 Which of the following best describes the relationship between the two passages?

ANSWERS AND EXPLANATIONS Explanation for Correct Answer C : 

(A) Passage 1 offers a hypothesis that is explicitly refuted in Passage 2.

(B) Passage 1 describes a popular misconception that is exemplified by Passage 2.

(C) Passage 2 presents an argument that elaborates on a point made in Passage 1.

(D) Passage 2 defends a position that is attacked in Passage 1.

(E) Passage 2 provides an anecdote that confirms the theory advanced in Passage 1.

页码,7/21The Official SAT Online Course

2006-11-12file://E:\新建文件夹\f7.htm

UnRe

gistered

Page 76: SAT test 6

严禁用于商业用途!

Choice (C) is correct. Passage 1 claims that estimating the Earth's "carrying capacity" for humans is "not an exact science." Passage 2 elaborates on this point by explaining how "carrying capacity" is a subjective concept when applied to humans.

Explanation for Incorrect Answer A :  Choice (A) is incorrect. Passage 2 supports, not refutes, the claim made in Passage 1 that scientists should stay away from "carrying capacity" estimates.

Explanation for Incorrect Answer B :  Choice (B) is incorrect. While the notion that human sustainability can be easily estimated may be a popular misunderstanding, Passage 2 counters, rather than exemplifies, such a "misconception."

Explanation for Incorrect Answer D :  Choice (D) is incorrect. Passage 1 criticizes scientists who attempt to calculate the Earth's "carrying capacity" for humans, but the author of Passage 2 agrees with this criticism.

Explanation for Incorrect Answer E :  Choice (E) is incorrect. Passage 2 contains nothing that resembles an "anecdote," or retelling of an interesting incident.

The passage below is excerpted from the introduction to a collection of essays published in 1994.

      My entry into Black women’s history was serendipitous.

 In the preface to Black Women in America: An Historical

 Encyclopedia, I recount the story of exactly how Shirley

LineHerd (who, in addition to teaching in the local school sys-

5tem, was also president of the Indianapolis chapter of the

 National Council of Negro Women) successfully provoked

 me into changing my research and writing focus. Although

 I dedicate this volume to her and to her best friend, fellow

 club woman and retired primary school teacher Virtea

10Downey, I still blush at the fact that I went to graduate

 school to become a historian in order to contribute to the

 Black Struggle for social justice and yet met her request to

 write a history of Black women in Indiana with condescen-

 sion. I had never even thought about Black women as his-

15torical subjects with their own relations to a state’s history,

页码,8/21The Official SAT Online Course

2006-11-12file://E:\新建文件夹\f7.htm

UnRe

gistered

Page 77: SAT test 6

严禁用于商业用途!

 and I thought her invitation and phone call extraordinarily

 intrusive. Only later did I concede how straightforward

 and reasonable had been her request to redress a historical

 omission. Black women were conspicuous by their absence.

20None of the social studies texts or state histories that Herd

 and Downey had used to teach their students made mention

 of the contributions of Black women. Since historians had

 left them out, Herd reasoned, only a “real” historian could

  put them in, and since I was the only tenured*

Black woman

25historian in the state of Indiana at that time, the task was

  mine.

      Herd rejected my reservations and completely ignored

 my admonitions that she could not call up a historian and

 order a book the way you drive up to a fast-food restaurant

30and order a hamburger. In spite of my assertions of igno-

 rance about the history of Black women in Indiana and my

 confession of having never studied the subject in any his-

 tory course or examined any manuscript sources pertaining

 to their lives, Herd persevered. Black women, as historical

35subjects and agents, were as invisible to me as they had

  been to school textbook writers.

      Undaunted by my response, Herd demanded that I con-

 nect (thankfully without perfect symmetry) my biology

 and autobiography, my race and gender, my being a Black

40woman, to my skill as a historian, and write for her and for

 the local chapter members of the National Council a history

 of Black women in Indiana. I relented and wrote the book,

页码,9/21The Official SAT Online Course

2006-11-12file://E:\新建文件夹\f7.htm

UnRe

gistered

Page 78: SAT test 6

严禁用于商业用途!

 When the Truth Is Told: Black Women’s Culture and

 Community in Indiana, 1875-1950, as requested. In the

45process, I was both humbled and astounded by the array of

 rich primary source materials Herd, Downey, and the other

 club women had spent two years collecting. There were

 diaries, club notes, church souvenir booklets, photographs,

 club minutes, birth, death, and marriage certificates, letters,

50and handwritten county and local histories. Collectively

 this material revealed a universe I never knew existed in

 spite of having lived with Black women all of my life . . .

 and being one myself. Or perhaps more accurately, I knew

 a universe of Black women existed. I simply had not envi-

55 sioned its historical meaning.

 

* tenure: a permanent position, often granted to a teacher after a specified number of years of demonstrated competence

10The primary purpose of the passage is to show how the author

ANSWERS AND EXPLANATIONS Explanation for Correct Answer E :  Choice (E) is correct. The passage mainly discusses the process by which the author comes to realize that Black women are a worthy subject of historical study.

Explanation for Incorrect Answer A :  Choice (A) is incorrect. The author explains that she did not discover her interest in Black women's history until she was already a tenured historian. It was only after her discussions with Herd and Downey that the author became interested in the subject.

Explanation for Incorrect Answer B :  Choice (B) is incorrect. Although the author does mention that she "went to graduate school to become a historian in order to contribute to the Black Struggle for social justice," the passage was not written to support this claim.

(A) discovered Black women’s history when she was in graduate school

(B) became a historian to help Black people in America achieve social justice

(C) developed her research skills by undertaking a challenging project

(D) became a more renowned scholar due to the influence of two interesting individuals

(E) came to view Black women as a worthy subject for historical analysis

页码,10/21The Official SAT Online Course

2006-11-12file://E:\新建文件夹\f7.htm

UnRe

gistered

Page 79: SAT test 6

严禁用于商业用途!

Explanation for Incorrect Answer C :  Choice (C) is incorrect. The author may have honed her research skills while preparing for her book, but the primary purpose of the passage is to show the development of her beliefs as a historian.

Explanation for Incorrect Answer D :  Choice (D) is incorrect. Although the author does mention the impact of Herd and Downey on her project, there is no indication that her collaboration with them enhanced her status as a historian.

11 The first sentence indicates that the author’s “entry” (line 1) was

ANSWERS AND EXPLANATIONS Explanation for Correct Answer D :  Choice (D) is correct. The author describes her introduction to Black women's history as "serendipitous," indicating that she regarded her "entry" into the field as both unexpected and fortunate.

Explanation for Incorrect Answer A :  Choice (A) is incorrect. The author would almost certainly agree that her entry into Black women's history was "worthwhile," but the term "serendipitous" does not suggest that it was particularly "troublesome."

Explanation for Incorrect Answer B :  Choice (B) is incorrect. The first sentence alone does not support the claim that the author's initiation into Black women's history was "challenging."

Explanation for Incorrect Answer C :  Choice (C) is incorrect. "Serendipitous" means the opposite of "inevitable," or unavoidable.

Explanation for Incorrect Answer E :  Choice (E) is incorrect. The author may have been startled to find so much value in a topic that she had previously ignored, but "serendipitous" does not mean "provocative," or stimulating.

(A) troublesome but worthwhile

(B) challenging but rewarding

(C) fortunate and inevitable

(D) unexpected but agreeable

(E) startling and provocative

12 The author initially responded to Herd’s request “with condescension” (lines 13-14) because the author

ANSWERS AND EXPLANATIONS Explanation for Correct Answer E :  Choice (E) is correct. The author clearly considered Herd's request "presumptuous" and "intrusive," but she also believed it to be irrelevant because she "had never even thought about Black women as historical subjects."

(A) knew that Herd had not been to graduate school

(B) believed that historians should avoid controversial projects

(C) had too many other projects requiring her attention

(D) rejected Herd’s contention that such a history would address the Black struggle for social justice

(E) viewed Herd’s request as irrelevant and presumptuous

页码,11/21The Official SAT Online Course

2006-11-12file://E:\新建文件夹\f7.htm

UnRe

gistered

Page 80: SAT test 6

严禁用于商业用途!

Explanation for Incorrect Answer A :  Choice (A) is incorrect. The author does not mention Herd's educational credentials.

Explanation for Incorrect Answer B :  Choice (B) is incorrect. The author gives no indication that she thinks historians should avoid controversy.

Explanation for Incorrect Answer C :  Choice (C) is incorrect. The author does not specifically mention whether she had any other time-consuming commitments.

Explanation for Incorrect Answer D :  Choice (D) is incorrect. The author does not indicate that Herd explicitly discussed a "struggle for social justice."

13 The comparison in lines 27-30 (“Herd . . . hamburger”) primarily demonstrates the author’s belief that historians

ANSWERS AND EXPLANATIONS Explanation for Correct Answer C :  Choice (C) is correct. The author's admonishment indicates that she was not accustomed to undertaking academic projects upon request.

Explanation for Incorrect Answer A :  Choice (A) is incorrect. The author does not mention the issue of payment at any point in the passage.

Explanation for Incorrect Answer B :  Choice (B) is incorrect. The author does not imply that historians are modest about their work. On the contrary, these lines reflect her initial arrogance regarding Herd's proposed project.

Explanation for Incorrect Answer D :  Choice (D) is incorrect. While historians may spend many years working on one project, the hamburger comparison does not address this idea.

Explanation for Incorrect Answer E :  Choice (E) is incorrect. Although it may be rare for a historian to accept a project idea from a member of the public, there is no evidence to support the claim that historians do not "interact with members of the public" in general.

(A) do not usually accept pay for their work

(B) are frequently unassuming about their profession

(C) do not generally undertake projects on request

(D) spend a comparatively long time on their projects

(E) do not generally interact with members of the public

14Lines 30-34 (“In spite . . . persevered”) suggest that the author believed that

ANSWERS AND EXPLANATIONS

(A) her lack of scholarly training on this topic was a reason to be embarrassed

(B) primary source materials on this subject would be difficult to find

(C) historians should conduct research in the areas in which they have expertise

(D) the lives of Black women in Indiana were historically interesting and complex

(E) Herd wanted her to conduct research on a topic of general interest

页码,12/21The Official SAT Online Course

2006-11-12file://E:\新建文件夹\f7.htm

UnRe

gistered

Page 81: SAT test 6

严禁用于商业用途!

Explanation for Correct Answer C :  Choice (C) is correct. The author tried to excuse herself from fulfilling Herd's request by professing her ignorance of Black women's history. Inherent in this action is the assumption that historians should work within their areas of expertise.

Explanation for Incorrect Answer A :  Choice (A) is incorrect. The author does not imply that her lack of scholarly training in this previously neglected subject is a source of embarrassment.

Explanation for Incorrect Answer B :  Choice (B) is incorrect. While the author had not studied any primary sources before accepting Herd's proposal, she does not imply that she had assumed they would be difficult to find.

Explanation for Incorrect Answer D :  Choice (D) is incorrect. The author clearly states that she had never before "thought about Black women as historical subjects."

Explanation for Incorrect Answer E :  Choice (E) is incorrect. Nothing in this sentence or in the passage as a whole suggests that Black women's history was considered to be a mainstream subject.

15 The last two sentences (“Or perhaps . . . meaning”) primarily indicate that the author

ANSWERS AND EXPLANATIONS Explanation for Correct Answer A :  Choice (A) is correct. The final two sentences of the passage clarify the author's original misconception of Black women's role in history. She writes that she had been aware of "a universe of Black women" without comprehending its historical significance.

Explanation for Incorrect Answer B :  Choice (B) is incorrect. The author does not indicate that the book was particularly challenging to write.

Explanation for Incorrect Answer C :  Choice (C) is incorrect. The passage asserts that Black women's history has been widely neglected by academics, but ultimately argues that it is a worthy book topic.

Explanation for Incorrect Answer D :  Choice (D) is incorrect. The author does not discuss the history of Black women's political power in these sentences or elsewhere in the passage.

Explanation for Incorrect Answer E :  Choice (E) is incorrect. The last two sentences are less about diversity than they are about Black women's historical significance.

(A) knew that Black women contributed to society, but she did not understand the significance of their contributions

(B) believed that the diversity of Black women’s experiences would make them difficult to write about

(C) assumed that because Black women are not frequently studied by historians, they would not be an acceptable topic for a book

(D) believed that Black women wield political power only in contemporary times

(E) was aware of the diversity of Black women’s lives, but was not willing to write about them

This passage, from a short story published in 1978, describes a visit to a planetarium,

页码,13/21The Official SAT Online Course

2006-11-12file://E:\新建文件夹\f7.htm

UnRe

gistered

Page 82: SAT test 6

严禁用于商业用途!

a building in which images of stars, planets, and other astronomical phenomena are projected onto a domed ceiling.

      Inside, we sat on wonderfully comfortable seats that

 were tilted back so that you lay in a sort of a hammock,

 attention directed to the bowl of the ceiling, which soon

Lineturned dark blue, with a faint rim of light around the edge.

5There was some splendid, commanding music. The adults

 all around were shushing the children, trying to make them

 stop crackling their potato chip bags. Then a man’s voice,

 an eloquent professional voice, began to speak slowly, out

 of the walls. The voice reminded me a little of the way

10radio announcers used to introduce a piece of classical

 music or describe the progress of the Royal Family to

 Westminster Abbey on one of their royal occasions.

  There was a faint echo-chamber effect.

      The dark ceiling was filled with stars. They came out not

15all at once but one after another, the way stars really

 do come out at night, though more quickly. The Milky

 Way galaxy appeared, was moving closer; stars swam

 into brilliance and kept on going, disappearing beyond

 the edges of the sky-screen or behind my head. While the

20flow of light continued, the voice presented the stunning

 facts. From a few light-years away, it announced, the Sun

 appears as a bright star, and the planets are not visible.

 From a few dozen light-years away, the Sun is not visible,

 either, to the naked eye. And that distance—a few dozen

25light-years—is only about a thousandth part of the distance from the Sun to the center of our galaxy, one

页码,14/21The Official SAT Online Course

2006-11-12file://E:\新建文件夹\f7.htm

UnRe

gistered

Page 83: SAT test 6

严禁用于商业用途!

  galaxy, which

 itself contains about two hundred billion stars. And is, in

 turn, one of millions, perhaps billions, of galaxies. Innu-

 merable repetitions, innumerable variations. All this

30 rolled past my head, too, like balls of lightning.

      Now realism was abandoned, for familiar artifice.

 A model of the solar system was spinning away in its ele-

 gant style. A bright bug took off from the Earth, heading

 for Jupiter. I set my dodging and shrinking mind sternly

35to recording facts. The mass of Jupiter two and a half

 times that of all the other planets put together. The Great

 Red Spot. The thirteen moons. Past Jupiter, a glance at

 the eccentric orbit of Pluto, the icy rings of Saturn. Back

 to Earth and moving in to hot and dazzling Venus. Atmo-

40spheric pressure ninety times ours. Moonless Mercury

 rotating three times while circling the Sun twice; an odd

 arrangement, not as satisfying as what they used to tell us

 —that it rotated once as it circled the Sun. No perpetual

 darkness after all. Why did they give out such confident

45information, only to announce later that it was quite wrong?

 Finally, the picture already familiar from magazines: the

  red soil of Mars, the blooming pink sky.

      When the show was over I sat in my seat while children

 clambered over me, making no comments on anything they

50had just seen or heard. They were pestering their keepers

 for eatables and further entertainments. An effort had been

 made to get their attention, to take it away from canned

  drinks and potato chips and fix it on various

页码,15/21The Official SAT Online Course

2006-11-12file://E:\新建文件夹\f7.htm

UnRe

gistered

Page 84: SAT test 6

严禁用于商业用途!

knowns and

 unknowns and horrible immensities, and it seemed to have

55failed. A good thing, too, I thought. Children have a natural

 immunity, most of them, and it shouldn’t be tampered with.

 As for the adults who would deplore it, the ones who pro-

 moted this show, weren’t they immune themselves to the

 extent that they could put in the echo-chamber effects,

60the music, the solemnity, simulating the awe that they

 supposed they ought to feel? Awe—what was that sup-

 posed to be? A fit of the shivers when you looked out

 the window? Once you knew what it was, you wouldn’t

  be courting it.

16 Which best describes the overall structure of the passage?

ANSWERS AND EXPLANATIONS Explanation for Correct Answer A :  Choice (A) is correct. The bulk of the passage recreates the narrator's visit to a planetarium, but the text switches from "description" to "commentary" in the final paragraph.

Explanation for Incorrect Answer B :  Choice (B) is incorrect. The passage seems to be a "reminiscence," or a recollection, of past events, but there is no indication that the narrator jumps from the past to the present in either verb tense or content.

Explanation for Incorrect Answer C :  Choice (C) is incorrect. The narrator suggests that the pursuit of "awe" is fundamentally problematic but does not offer a "solution" of any kind.

Explanation for Incorrect Answer D :  Choice (D) is incorrect. The narrator uses a myriad of details, not generalizations, to depict the sensory experience created inside of the planetarium.

Explanation for Incorrect Answer E :  Choice (E) is incorrect. The only viewpoint presented in the passage is that of the narrator.

(A) Narrative description followed by commentary

(B) Reminiscence followed by present-day application

(C) An account of a problem followed by a suggested solution

(D) A generalization followed by specific examples

(E) A discussion of opposing viewpoints followed by an attempt to reconcile them

页码,16/21The Official SAT Online Course

2006-11-12file://E:\新建文件夹\f7.htm

UnRe

gistered

Page 85: SAT test 6

严禁用于商业用途!

17 Lines 5-7 (“The adults . . . bags”) primarily illustrate the children’s feelings of

ANSWERS AND EXPLANATIONS Explanation for Correct Answer B :  Choice (B) is correct. "Restless" means antsy or excitable. The image of "adults all around . . . shushing the children" suggests that the children are too restless to remain quiet in the darkened room.

Explanation for Incorrect Answer A :  Choice (A) is incorrect. The adults may feel helpless in their efforts to quiet the noisy children, but nothing in these lines indicates that the children themselves are helpless.

Explanation for Incorrect Answer C :  Choice (C) is incorrect. The children's agitation contrasts with the feelings of "awe" that the planetarium show is presumably meant to inspire.

Explanation for Incorrect Answer D :  Choice (D) is incorrect. While some of the children in the planetarium audience may look forward to the show, their general noisiness indicates inattention more than "anticipation."

Explanation for Incorrect Answer E :  Choice (E) is incorrect. Some of the children may act out of mild boredom, but the narrator does not suggest that they are largely irritated, or annoyed.

(A) helplessness

(B) restlessness

(C) awe

(D) anticipation

(E) irritation

18 In line 11, “progress” most nearly means

ANSWERS AND EXPLANATIONS Explanation for Correct Answer E :  Choice (E) is correct. The word "progress" in this context means a ceremonial journey. Line 11 refers to members of British royalty making such a trip to a famous London cathedral.

Explanation for Incorrect Answer A :  Choice (A) is incorrect. The term "progress" is sometimes used to refer to "evolution," or gradual development, but it is illogical to describe the "evolution" of a group of people to a place.

Explanation for Incorrect Answer B :  Choice (B) is incorrect. "Progress" sometimes indicates a positive change, but it makes little sense to describe the Royal Family's procession to Westminster Abbey as an "improvement."

Explanation for Incorrect Answer C :  Choice (C) is incorrect. While it may be said that one who prospers is progressing

(A) evolution

(B) improvement

(C) prosperity

(D) promotion

(E) advance

页码,17/21The Official SAT Online Course

2006-11-12file://E:\新建文件夹\f7.htm

UnRe

gistered

Page 86: SAT test 6

严禁用于商业用途!

financially or personally, this use of the term "progress" does not fit in the context of line 11.

Explanation for Incorrect Answer D :  Choice (D) is incorrect. The term "promotion" typically refers to a professional progression, which has nothing to do with the Royal Family's ceremonial journey.

19 The first paragraph of the passage establishes a mood of

ANSWERS AND EXPLANATIONS Explanation for Correct Answer C :  Choice (C) is correct. The first paragraph attempts to recreate the tension that exists in a darkened planetarium as audience members settle into their chairs and wait for the show to begin. The author's evocative description of the domed screen coming to life as "commanding music" swells under an "eloquent," disembodied voice contributes to the feeling of "dramatic anticipation."

Explanation for Incorrect Answer A :  Choice (A) is incorrect. At no point does the narrator appear to be "jaded," or tired, of the planetarium experience.

Explanation for Incorrect Answer B :  Choice (B) is incorrect. The first paragraph is peppered with adjectives such as "comfortable," "splendid," and "eloquent." These terms create a mood that is neither nervous nor apprehensive.

Explanation for Incorrect Answer D :  Choice (D) is incorrect. The passage does not suggest that the narrator has any concerns about the upcoming presentation.

Explanation for Incorrect Answer E :  Choice (E) is incorrect. It is unlikely that an author would describe a mundane, routine event or feeling in such colorful detail.

(A) jaded dismissal

(B) nervous apprehension

(C) dramatic anticipation

(D) initial concern

(E) mundane routine

20 The words “dodging and shrinking” (line 34) primarily suggest that the author was

ANSWERS AND EXPLANATIONS Explanation for Correct Answer B :  Choice (B) is correct. "Stunning facts" illustrating the immensity of the universe "rolled past" the narrator's head in the second paragraph, creating a dizzying web of information. The narrator's mind seems to dodge and shrink away from the deluge of figures and concepts. Only when the facts become less abstract and more familiar is the narrator capable of mechanically recording information.

(A) somewhat bothered by the children in the audience

(B) initially overwhelmed by the information being presented

(C) unable to admit to some troubling feelings about astronomy

(D) refusing to acknowledge the implications of space travel

(E) unwilling to believe the studies being discussed

页码,18/21The Official SAT Online Course

2006-11-12file://E:\新建文件夹\f7.htm

UnRe

gistered

Page 87: SAT test 6

严禁用于商业用途!

Explanation for Incorrect Answer A :  Choice (A) is incorrect. The reference to the narrator's "dodging and shrinking mind" appears in line 34. It is unlikely that the children who are briefly mentioned in line 6 are responsible for the narrator's mental state almost 30 lines later.

Explanation for Incorrect Answer C :  Choice (C) is incorrect. The narrator readily admits that the astronomical facts are "troubling" in their complexity.

Explanation for Incorrect Answer D :  Choice (D) is incorrect. No significant mention of "space travel" appears at any point in the passage.

Explanation for Incorrect Answer E :  Choice (E) is incorrect. Although the narrator acknowledges in lines 44–45 that "confident" scientific information is often revised, nothing in the passage suggests that the narrator's "dodging and shrinking mind" is related to any doubts regarding the show's accuracy.

21 In lines 40-43 (“Moonless . . . Sun”), the narrator’s comment about the “arrangement” demonstrates a preference for

ANSWERS AND EXPLANATIONS Explanation for Correct Answer C :  Choice (C) is correct. The narrator is disappointed to learn from the planetarium show that Mercury's orbit is more complicated than once thought. Apparently the moonless planet rotates three times per every two trips around the sun, "an odd arrangement" compared to the outdated theory that Mercury rotates just once per solar orbit. The narrator's preference for the simpler "arrangement" implies a preference for "symmetry," or balance.

Explanation for Incorrect Answer A :  Choice (A) is incorrect. Lines 40–43 do not demonstrate the narrator's sense of "irony."

Explanation for Incorrect Answer B :  Choice (B) is incorrect. The narrator's reaction to Mercury's "odd arrangement" may be unusual, but it does not particularly demonstrate "inventiveness," or creativity.

Explanation for Incorrect Answer D :  Choice (D) is incorrect. The narrator includes numerous details in the passage but presents them simply. It is somewhat inaccurate, then, to claim that the narrator's style as it appears in lines 40–43 is ornamental, or embellished.

Explanation for Incorrect Answer E :  Choice (E) is incorrect. The narrator's feelings regarding Mercury's complicated orbit suggest a discomfort with "ambiguity," or confusion, not a preference for it.

(A) irony

(B) inventiveness

(C) symmetry

(D) ornamentation

(E) ambiguity

22 In line 53, “fix” most nearly means

(A) focus

(B) prepare

(C) repair

(D) decide

influence

页码,19/21The Official SAT Online Course

2006-11-12file://E:\新建文件夹\f7.htm

UnRe

gistered

Page 88: SAT test 6

严禁用于商业用途!

ANSWERS AND EXPLANATIONS Explanation for Correct Answer A :  Choice (A) is correct. "Fix" in this context means to direct one's attention. Lines 51–55 specifically refer to the adults' effort to catch the children's attention and direct it to the planetarium show.

Explanation for Incorrect Answer B :  Choice (B) is incorrect. "Fix" sometimes means to "prepare," but this definition is illogical within the context of line 53.

Explanation for Incorrect Answer C :  Choice (C) is incorrect. A common definition of "fix" is to "repair," but it is illogical to claim that adults tried to "repair" children's attention to something.

Explanation for Incorrect Answer D :  Choice (D) is incorrect. The term "fix" is occasionally used to reflect a decision, but a child's attention cannot be decided on something else.

Explanation for Incorrect Answer E :  Choice (E) is incorrect. An event or outcome that has been improperly influenced is sometimes described as "fixed," but this sense of the word does not fit the context of line 53.

(E)

23 The phrase “horrible immensities” (line 54) primarily indicates

ANSWERS AND EXPLANATIONS Explanation for Correct Answer E :  Choice (E) is correct. The astronomical "realities" portrayed in the planetarium show suggest a universe so immense that its sheer size is "incomprehensible" and horribly intimidating to the narrator.

Explanation for Incorrect Answer A :  Choice (A) is incorrect. Facts that indicate the vastness of the universe may seem "exaggerated," but there is nothing to suggest that the information presented at the planetarium is overstated.

Explanation for Incorrect Answer B :  Choice (B) is incorrect. The presentation does not focus on "unforeseen," or unexpected, events.

Explanation for Incorrect Answer C :  Choice (C) is incorrect. The presentation does not address a single monstrosity, historical or otherwise.

Explanation for Incorrect Answer D :  Choice (D) is incorrect. Nowhere in the passage does the narrator directly refer to "controversial debates."

(A) exaggerated information

(B) unforeseen events

(C) historical monstrosities

(D) controversial debates

(E) incomprehensible realities

页码,20/21The Official SAT Online Course

2006-11-12file://E:\新建文件夹\f7.htm

UnRe

gistered

Page 89: SAT test 6

严禁用于商业用途!

24 The author suggests that the “echo-chamber effects, the music, the solemnity” (lines 59-60) are evidence that

ANSWERS AND EXPLANATIONS Explanation for Correct Answer D :  Choice (D) is correct. The narrator criticizes the show's promoters by suggesting that they used special effects such as "echo-chamber effects" to create a mere simulation of "the awe that they.. . ought to feel" at the immensity of the universe.

Explanation for Incorrect Answer A :  Choice (A) is incorrect. The narrator does not imply that the majority of adults appreciate the universe's "horrible immensities," but instead suggests that the adults responsible for infusing the show with special effects are "immune themselves" to the awesome realities of the universe.

Explanation for Incorrect Answer B :  Choice (B) is incorrect. The narrator does not speculate about the popularity of planetarium shows among adults.

Explanation for Incorrect Answer C :  Choice (C) is incorrect. Contemporary scientists likely contributed to the planetarium presentation, but the narrator does not discuss how these scientists feel about their work.

Explanation for Incorrect Answer E :  Choice (E) is incorrect. The show's creators may have included special effects to appeal to children, but the narrator does not mention such a theory. The only implication is that such effects illustrate the promoters' immunity to the awesome nature of the universe.

(A) most adults have feelings of great appreciation for the universe

(B) most adults would rather not attend planetarium shows

(C) contemporary scientists have an inflated view of the importance of their work

(D) the show’s promoters do not fully appreciate the true nature of the universe

(E) the show’s promoters understand that children are entranced by special effects

    

Back to Score Report  

Copyright © 2006 The College Board. All rights reserved. Privacy Policy Terms of Use Contact Us

 

页码,21/21The Official SAT Online Course

2006-11-12file://E:\新建文件夹\f7.htm

UnRe

gistered

Page 90: SAT test 6

严禁用于商业用途!

Help | Profile | My Organizer | My Bookmarks | Logout

Answers and Explanations

Test Sections

Section 1

Section 2

Section 4

Section 5

Section 6

Section 7

Section 8

Section 9

Section 10

Back to Score Report  

View Answers and Explanations     Online - Practice Test #6

1 If notebooks cost $2 each and backpacks cost $32 each, which of the following represents the cost, in dollars, of n notebooks and b backpacks?

ANSWERS AND EXPLANATIONS Explanation for Correct Answer D : 

Choice (D) is correct. If notebooks cost each, then the cost of notebooks is

dollars. If backpacks cost each, then the cost of backpacks is

dollars. So, the cost, in dollars, of notebooks and notebooks is

Explanation for Incorrect Answer A :  Choice (A) is not correct. See the explanation for the correct response (D).

Explanation for Incorrect Answer B :  Choice (B) is not correct. See the explanation for the correct response (D).

Explanation for Incorrect Answer C : 

Choice (C) is not correct. Since this would be the cost, in

dollars, of notebooks and backpacks if each notebook cost and each

backpack cost .

Explanation for Incorrect Answer E : 

Choice (E) is not correct. Since this would be the cost, in

dollars, of notebooks and backpacks if each notebook cost and each

backpack cost .

(A)

(B)

(C)

(D)

(E)

2The average (arithmetic mean) of 6, 19, and x is 19. What is the value of x?

ANSWERS AND EXPLANATIONS Explanation for Correct Answer D : 

Choice (D) is correct. If the average of and , then .

(A) 19

(B) 25

(C) 31

(D) 32

(E) 57

页码,1/13The Official SAT Online Course

2006-11-12file://E:\新建文件夹\f8.htm

UnRe

gistered

Page 91: SAT test 6

严禁用于商业用途!

So , and .

Explanation for Incorrect Answer A :  Choice (A) is not correct. Since the average of and is and ,

must be greater than . See the explanation for the correct response (D).

Explanation for Incorrect Answer B :  Choice (B) is not correct. See the explanation for the correct response (D).

Explanation for Incorrect Answer C :  Choice (C) is not correct. See the explanation for the correct response (D).

Explanation for Incorrect Answer E :  Choice (E) is not correct. The sum of and is equal to , but is not

equal to . See the explanation for the correct response (D).

3 Ali, Ben, and Carla made a total of 20 sandwiches. Ben made 3 times as many as Ali, and Carla made twice as many as Ben. How many sandwiches did Ali make?

ANSWERS AND EXPLANATIONS Explanation for Correct Answer A :  Choice (A) is correct. If Ali made sandwiches, and Ben made three times as many as Ali, then Ben made sandwiches. Carla made twice as many

sandwiches as Ben, so Carla made sandwiches. The total number of

sandwiches they made was . This is also equal to , so

. Solving for gives , so Ali made sandwiches.

Explanation for Incorrect Answer B :  Choice (B) is not correct. See the explanation for the correct response (A).

Explanation for Incorrect Answer C :  Choice (C) is not correct. See the explanation for the correct response (A).

Explanation for Incorrect Answer D :  Choice (D) is not correct. See the explanation for the correct response (A). Ben made times as many sandwiches as Ali, and Ali made sandwiches, so is the number of sandwiches Ben made. However, the question asked how many sandwiches Ali made.

Explanation for Incorrect Answer E :  Choice (E) is not correct. See the explanation for the correct response (A).

(A) Two

(B) Four

(C) Five

(D) Six

(E) Ten

4If 0.03 percent of n is 3, what is 3 percent of n?

(A) 900

(B) 600

(C) 300

(D) 0.006

(E) 0.003

页码,2/13The Official SAT Online Course

2006-11-12file://E:\新建文件夹\f8.htm

UnRe

gistered

Page 92: SAT test 6

严禁用于商业用途!

ANSWERS AND EXPLANATIONS Explanation for Correct Answer C : 

Choice (C) is correct. If percent of is , then . So

, and . Thus, percent of is

.

Explanation for Incorrect Answer A :  Choice (A) is not correct. From the explanation for the correct response (C), you have is . So percent of . However, the question asked for percent of .

Explanation for Incorrect Answer B :  Choice (B) is not correct. From the explanation for the correct response (C), you have is . So is percent of . However, the question asked for

percent of .

Explanation for Incorrect Answer D :  Choice (D) is not correct. From the explanation for the correct response (C), you have is . So is percent of . However, the question

asked for percent of .

Explanation for Incorrect Answer E :  Choice (E) is not correct. From the explanation for the correct response (C), you have is . So is percent of . However, the question

asked for percent of .

5

What is an equation of line in the figure above?

ANSWERS AND EXPLANATIONS Explanation for Correct Answer B : 

Choice (B) is correct. An equation of line is , where is the slope

of and is the -intercept of . From the figure, you can see that the points

(A)

(B)

(C)

(D)

(E)

页码,3/13The Official SAT Online Course

2006-11-12file://E:\新建文件夹\f8.htm

UnRe

gistered

Page 93: SAT test 6

严禁用于商业用途!

and are on , so the slope is . Since intersects the -

axis at the point , the -intercept of is . So and , and

has the equation .

Explanation for Incorrect Answer A : 

Choice (A) is not correct. You can see from the figure that the point is on line

. But . So cannot be an equation

of .

Explanation for Incorrect Answer C : 

Choice (C) is not correct. You can see from the figure that the point is on line

. But .

Explanation for Incorrect Answer D : 

Choice (D) is not correct. You can see from the figure that the point is on line

. But .

Explanation for Incorrect Answer E : 

Choice (E) is not correct. You can see from the figure that the point is on line

. But .

6

If the tick marks on the number line above are equally spaced, which of the lettered

points A through E is between and

ANSWERS AND EXPLANATIONS Explanation for Correct Answer B :  Choice (B) is correct. If the portion of the number line between and is divided into eighths, the result is

The only lettered point between and is .

(A) A

(B) B

(C) C

(D) D

(E) E

页码,4/13The Official SAT Online Course

2006-11-12file://E:\新建文件夹\f8.htm

UnRe

gistered

Page 94: SAT test 6

严禁用于商业用途!

Explanation for Incorrect Answer A :  Choice (A) is not correct. If the portion of the number line between and is divided into eighths, as in the figure above, you can see that point is less than

.

Explanation for Incorrect Answer C :  Choice (C) is not correct. If the portion of the number line between and is

divided into eighths, as in the figure above, you can see that point is greater

than .

Explanation for Incorrect Answer D :  Choice (D) is not correct. If the portion of the number line between and is divided into eighths, as in the figure above, you can see that point is greater

than .

Explanation for Incorrect Answer E :  Choice (E) is not correct. If the portion of the number line between and is divided into eighths, as in the figure above, you can see that point is greater

than .

7 If what does h equal in terms of x ?

ANSWERS AND EXPLANATIONS Explanation for Correct Answer D : 

Choice (D) is correct. If , then , so .

Explanation for Incorrect Answer A :  Choice (A) is not correct. See the explanation for the correct response (D).

Explanation for Incorrect Answer B :  Choice (B) is not correct. See the explanation for the correct response (D).

Explanation for Incorrect Answer C :  Choice (C) is not correct. See the explanation for the correct response (D).

Explanation for Incorrect Answer E :  Choice (E) is not correct. See the explanation for the correct response (D).

(A)

(B)

(C)

(D) x

(E)

8

页码,5/13The Official SAT Online Course

2006-11-12file://E:\新建文件夹\f8.htm

UnRe

gistered

Page 95: SAT test 6

严禁用于商业用途!

In the figure above, what is the value of y?

ANSWERS AND EXPLANATIONS Explanation for Correct Answer D :  Choice (D) is correct. Since , it follows that and

. Vertical angles are equal, so . Thus, .

Explanation for Incorrect Answer A :  Choice (A) is not correct. See the explanation for the correct response (D).

Explanation for Incorrect Answer B :  Choice (B) is not correct. See the explanation for the correct response (D).

Explanation for Incorrect Answer C :  Choice (C) is not correct. See the explanation for the correct response (D).

Explanation for Incorrect Answer E :  Choice (E) is not correct. See the explanation for the correct response (D).

(A) 40

(B) 45

(C) 50

(D) 60

(E) 72

9 If which of the following is a possible value of

ANSWERS AND EXPLANATIONS Explanation for Correct Answer C : 

Choice (C) is correct. If , then . So

, and either or . If , then

.

Explanation for Incorrect Answer A : 

Choice (A) is not correct. If , then . So

, and either or . If , then

. If , then . So

cannot equal .

Explanation for Incorrect Answer B : 

Choice (B) is not correct. If , then . So

, and either or . If , then

. If , then . So

(A)

(B) 10

(C) 20

(D) 30

(E) 870

页码,6/13The Official SAT Online Course

2006-11-12file://E:\新建文件夹\f8.htm

UnRe

gistered

Page 96: SAT test 6

严禁用于商业用途!

cannot equal .

Explanation for Incorrect Answer D : 

Choice (D) is not correct. If , then . So

, and either or . If , then

. If , then . So

cannot equal .

Explanation for Incorrect Answer E : 

Choice (E) is not correct. If , then . So

, and either or . If , then

. If , then . So

cannot equal .

10 Mark began a 4-mile bicycle trip by riding slowly uphill for 1 mile. He rested for 10 minutes and then rode quickly downhill for the rest of the trip. Which of the following graphs could correctly represent his trip?

ANSWERS AND EXPLANATIONS Explanation for Correct Answer B :  Choice (B) is correct. This graph could show Mark’s distance traveled as a function of time. The first portion of the graph indicates that Mark rode at a constant rate of

mile in minutes ( miles per hour) and covered a distance of mile. The

(A)

(B)

(C)

(D)

(E)

页码,7/13The Official SAT Online Course

2006-11-12file://E:\新建文件夹\f8.htm

UnRe

gistered

Page 97: SAT test 6

严禁用于商业用途!

next portion of the graph is horizontal, which indicates that he rested for minutes and did not cover any distance during that period. The graph indicates that during the final minutes Mark rode at a constant rate of miles in

minutes, or miles per hour, which is faster than he rode in the first minutes.

Explanation for Incorrect Answer A :  Choice (A) is not correct. This graph could not represent Mark’s trip. One reason is that it indicates that Mark rode at a rate of miles per hour for the first

minutes and at a rate of miles per hour during the last minutes of his trip. If miles per hour is described as riding “slowly,” then miles per hour cannot

be described as riding “quickly.”

Explanation for Incorrect Answer C :  Choice (C) is not correct. This graph could not represent Mark’s trip. One reason is that it shows the total distance of the trip to be miles, but the trip was miles long.

Explanation for Incorrect Answer D :  Choice (D) is not correct. This graph could not represent Mark’s trip. One reason is that it does not have a horizontal section to indicate the -minute period during which Mark covered no distance.

Explanation for Incorrect Answer E :  Choice (E) is not correct. This could not represent Mark’s trip. One reason is that it indicates a negative distance traveled during the last minutes of the trip. This is not possible.

11 There are 6 red, 6 brown, 6 yellow, and 6 gray scarves packaged in 24 identical, unmarked boxes, 1 scarf per box. What is the least number of boxes that must be selected in order to be sure that among the boxes selected 3 or more contain scarves of the same color?

ANSWERS AND EXPLANATIONS Explanation for Correct Answer E :  Choice (E) is correct. If boxes are selected, there could be red scarves in of them, brown scarves in of them, yellow scarves in of them, and gray scarves in of them; in this case, there would not be or more scarves of the same color

among these boxes. However, if one more box is selected, the color of the scarf

in that box must be the same as the color of the scarves in of the original

boxes, and there will then be of the same color. So, is the least number of

boxes that must be selected in order to be sure that among the boxes selected or more contain scarves of the same color.

Explanation for Incorrect Answer A :  Choice (A) is not correct. If boxes are selected, there could be a scarf of a different color in each box.

Explanation for Incorrect Answer B :  Choice (B) is not correct. If boxes are selected, there could be red scarves in of them, brown scarves in of them, and yellow scarves in of them.

Explanation for Incorrect Answer C :  Choice (C) is not correct. If boxes are selected, there could be red scarves in of them, brown scarves in of them, yellow scarves in of them, and a gray

(A) 3

(B) 6

(C) 7

(D) 8

(E) 9

页码,8/13The Official SAT Online Course

2006-11-12file://E:\新建文件夹\f8.htm

UnRe

gistered

Page 98: SAT test 6

严禁用于商业用途!

scarf in of them.

Explanation for Incorrect Answer D :  Choice (D) is not correct. If boxes are selected, there could be red scarves in of them, brown scarves in of them, yellow scarves in of them, and gray scarves in of them.

12

In the figure above, ABCDEF is a regular hexagon, and its center is point O. What is the value of x ?

ANSWERS AND EXPLANATIONS Explanation for Correct Answer B :  Choice (B) is correct. Since is the center of the hexagon, it follows that

. Drawing some additional lines and labeling some angles yields additional information from the figure:

From the symmetry of the regular hexagon, it follows that . Since

the sum of these angles is , each of these angles has a measure of . So

. It is also true that , so . Since , and

, it follows that . Therefore, , and .

Explanation for Incorrect Answer A :  Choice (A) is not correct. See the explanation for the correct response (B).

Explanation for Incorrect Answer C :  Choice (C) is not correct. See the explanation for the correct response (B).

Explanation for Incorrect Answer D :  Choice (D) is not correct. See the explanation for the correct response (B).

Explanation for Incorrect Answer E :  Choice (E) is not correct. See the explanation for the correct response (B).

(A) 80

(B) 60

(C) 40

(D) 30

(E) 20

13Let the function f be defined by for all numbers x. Which of the following is

equivalent to

页码,9/13The Official SAT Online Course

2006-11-12file://E:\新建文件夹\f8.htm

UnRe

gistered

Page 99: SAT test 6

严禁用于商业用途!

ANSWERS AND EXPLANATIONS Explanation for Correct Answer C : 

Choice (C) is correct. Since for all numbers , you have

.

Explanation for Incorrect Answer A : 

Choice (A) is not correct. Since for all numbers , you have

. This expression is not equivalent to . For

example, if , then , but .

Explanation for Incorrect Answer B : 

Choice (B) is not correct. Since for all numbers , you have

. This expression is not equivalent to . For

example, if , then , but .

Explanation for Incorrect Answer D : 

Choice (D) is not correct. Since for all numbers , you have

. This expression is not equivalent to .

For example, if , then , but .

Explanation for Incorrect Answer E : 

Choice (E) is not correct. Since for all numbers , you have

. This expression is not equivalent to . For

example, if , then , but .

(A)

(B)

(C)

(D)

(E) 25pr

14

The circle above has an area of and is divided into 8 congruent regions. What is the perimeter of one of these regions?

(A)

(B)

(C)

(D)

页码,10/13The Official SAT Online Course

2006-11-12file://E:\新建文件夹\f8.htm

UnRe

gistered

Page 100: SAT test 6

严禁用于商业用途!

ANSWERS AND EXPLANATIONS Explanation for Correct Answer C :  Choice (C) is correct. The perimeter of each of the congruent regions is

, where is the radius of the circle and is the circumference of the

circle. The area of the circle is , so . The circumference of the

circle is , so . The perimeter of one of the regions, then, is

.

Explanation for Incorrect Answer A :  Choice (A) is not correct. See the explanation for the correct response (C).

Explanation for Incorrect Answer B :  Choice (B) is not correct. See the explanation for the correct response (C).

Explanation for Incorrect Answer D :  Choice (D) is not correct. See the explanation for the correct response (C).

Explanation for Incorrect Answer E :  Choice (E) is not correct. See the explanation for the correct response (C).

(E)

15

For which of the following values of k will the system of equations above have no solution?

ANSWERS AND EXPLANATIONS Explanation for Correct Answer A : 

Choice (A) is correct. If , then the system of equations is

and . The first equation is equivalent to . There are

no values and for which can equal both and . So the

system of equations has no solution if .

Explanation for Incorrect Answer B : 

Choice (B) is not correct. If , then the system of equations is

and . The solution to this system is and .

Explanation for Incorrect Answer C : 

Choice (C) is not correct. If , then the system of equations is

(A)

(B)

(C) 0

(D) 5

(E) 10

页码,11/13The Official SAT Online Course

2006-11-12file://E:\新建文件夹\f8.htm

UnRe

gistered

Page 101: SAT test 6

严禁用于商业用途!

and . The solution to this system is and .

Explanation for Incorrect Answer D : 

Choice (D) is not correct. If , then the system of equations is

and . The solution to this system is and .

Explanation for Incorrect Answer E : 

Choice (E) is not correct. If , then the system of equations is

and . The solution to this system is and .

16 RESULTS OF BEANBAG GAME

In a certain game, each person threw a beanbag at a target until the person missed the target. The table shows the results for the 25 people who played the game. For example, 4 people hit the target on their first 3 throws and missed on their 4th throw. Based on the information in the table, which of the following must be true?

I. More than half the people hit the target on their first throw.

II. For all of the throws attempted, more hit the target than missed the target.

III. No one hit the target 5 times.

ANSWERS AND EXPLANATIONS Explanation for Correct Answer E :  Choice (E) is correct. To determine the answer to the question, you must decide which of the three statements are true.       Statement : From the table, you can see that only of the people

missed the target on their first throw. So , which is more than half of , hit the target on their first throw. is true.       Statement : The total number of throws attempted was

. The total

number of throws that missed the target was . So

throws hit the target, which is more than the number of throws that missed the target. is true.       Statement : Of the people, missed before their fifth throw and

missed their fifth throw. So no one hit the target times. is true.

Explanation for Incorrect Answer A :  Choice (A) is not correct. From the explanation for the correct answer, , , and

are all true. So is not the only true statement.

(A) I only

(B) II only

(C) I and III only

(D) II and III only

(E) I, II, and III

页码,12/13The Official SAT Online Course

2006-11-12file://E:\新建文件夹\f8.htm

UnRe

gistered

Page 102: SAT test 6

严禁用于商业用途!

Explanation for Incorrect Answer B :  Choice (B) is not correct. From the explanation for the correct answer, , , and

are all true. So is not the only true statement.

Explanation for Incorrect Answer C :  Choice (C) is not correct. From the explanation for the correct answer, , , and

are all true. So and are not the only true statements.

Explanation for Incorrect Answer D :  Choice (D) is not correct. From the explanation for the correct answer, , , and

are all true. So and are not the only true statements.

    

Back to Score Report  

Copyright © 2006 The College Board. All rights reserved. Privacy Policy Terms of Use Contact Us

 

页码,13/13The Official SAT Online Course

2006-11-12file://E:\新建文件夹\f8.htm

UnRe

gistered

Page 103: SAT test 6

严禁用于商业用途!

Help | Profile | My Organizer | My Bookmarks | Logout

Answers and Explanations

Test Sections

Section 1

Section 2

Section 4

Section 5

Section 6

Section 7

Section 8

Section 9

Section 10

Back to Score Report  

View Answers and Explanations     Online - Practice Test #6

1 The new vaccine is ------- preventing certain forms of pneumonia and should, therefore, be more widely ------- in order to prevent outbreaks of the disease.

ANSWERS AND EXPLANATIONS Explanation for Correct Answer D :  Choice (D) is correct. Something that is "effective" produces a desired effect. "Administered" means dispensed. If one were to insert these terms into the text, the sentence would read "The new vaccine is effective in preventing certain forms of pneumonia and should, therefore, be more widely administered in order to prevent outbreaks of the disease." The word "therefore" indicates that the second part of the sentence should logically occur, based on the claim made in the first part. It makes sense to argue that a successful vaccine should be widely administered.

Explanation for Incorrect Answer A :  Choice (A) is incorrect. "Required" means needed or essential. "Constrained" means to keep within close bounds. If one were to insert these terms into the text, the sentence would read "The new vaccine is required for preventing certain forms of pneumonia and should, therefore, be more widely constrained in order to prevent outbreaks of the disease." It makes no sense to say that a vaccine that is required for preventing illness should be "constrained," or restricted.

Explanation for Incorrect Answer B :  Choice (B) is incorrect. "Unsuccessful" means having an unfavorable outcome. "Distributed" means delivered or handed out. If one were to insert these terms into the text, the sentence would read "The new vaccine is unsuccessful in preventing certain forms of pneumonia and should, therefore, be more widely distributed in order to prevent outbreaks of the disease." It is illogical to argue that an unsuccessful vaccine should be distributed or passed out more widely.

Explanation for Incorrect Answer C :  Choice (C) is incorrect. "Instrumental" means serving as a means or agency. "Reconstituted" means provided with a new structure. If one were to insert these terms into the text, the sentence would read "The new vaccine is instrumental in preventing certain forms of pneumonia and should, therefore, be more widely reconstituted in order to prevent outbreaks of the disease." An instrumental or helpful vaccine does not need to be reconstituted, or remade, because it is already effective.

Explanation for Incorrect Answer E :  Choice (E) is incorrect. "Unverified" means lacking proof or substantiation. "Disseminated" means dispersed throughout. If one were to insert these terms into the text, the sentence would read "The new vaccine is unverified for preventing certain forms of pneumonia and should, therefore, be more widely disseminated in order to prevent outbreaks of the disease." It makes no sense to argue that an unverified or unproven vaccine should be disseminated or dispensed more widely.

(A) required for . . constrained

(B) unsuccessful in . . distributed

(C) instrumental in . . reconstituted

(D) effective in . . administered

(E) unverified for . . disseminated

页码,1/16The Official SAT Online Course

2006-11-12file://E:\新建文件夹\f9.htm

UnRe

gistered

Page 104: SAT test 6

严禁用于商业用途!

2 In an effort to ------- the ------- theater, the troupe members contributed thousands of dollars to keep the playhouse operating.

ANSWERS AND EXPLANATIONS Explanation for Correct Answer B :  Choice (B) is correct. "Salvage" means to save from ruin, and "floundering" means struggling. If one were to insert these terms into the text, the sentence would read "In an effort to salvage the floundering theater, the troupe members contributed thousands of dollars to keep the playhouse operating." The fact that the theater required thousands of dollors to maintain operations implies that it was floundering and needed to be salvaged.

Explanation for Incorrect Answer A :  Choice (A) is incorrect. "Qualify" means to describe or to modify. "Obsolete" means no longer in use. If one were to insert these terms into the text, the sentence would read "In an effort to qualify the obsolete theater, the troupe members contributed thousands of dollars to keep the playhouse operating." The second half of the sentence indicates that the theater is failing, but not yet obsolete. Furthermore, the verb "qualify" does not make sense in this context.

Explanation for Incorrect Answer C :  Choice (C) is incorrect. "Exacerbate" means to make worse, and something that is "defunct" has ceased to exist. If one were to insert these terms into the text, the sentence would read "In an effort to exacerbate the defunct theater, the troupe members contributed thousands of dollars to keep the playhouse operating." It is illogical to assert that actors would donate money to a struggling theater in an effort to make its problems worse.

Explanation for Incorrect Answer D :  Choice (D) is incorrect. "Revitalize" means to give new life to, and "prosperous" means marked by economic well-being. If one were to insert these terms into the text, the sentence would read "In an effort to revitalize the prosperous theater, the troupe members contributed thousands of dollars to keep the playhouse operating." A "prosperous," or wealthy, theater would not need to be revitalized through financial donations.

Explanation for Incorrect Answer E :  Choice (E) is incorrect. "Commandeer" means to take by force, and "lucrative" means profitable. If one were to insert these terms into the text, the sentence would read "In an effort to commandeer the lucrative theater, the troupe members contributed thousands of dollars to keep the playhouse operating." It is unlikely that actors would attempt to forcibly take over a playhouse or that a "lucrative" theater would be in need of financial donations.

(A) qualify . . obsolete

(B) salvage . . floundering

(C) exacerbate . . defunct

(D) revitalize . . prosperous

(E) commandeer . . lucrative

3 In her writings about language, the poet Gloria Anzaldúa celebrates the ------- of English and Spanish dialects spoken by Mexican Americans, arguing that such ------- lends an empowering flexibility to expression.

ANSWERS AND EXPLANATIONS Explanation for Correct Answer A : 

(A) multiplicity . . variety

(B) proliferation . . moderation

(C) ambivalence . . focus

(D) dearth . . depletion

(E) abridgment . . imitation

页码,2/16The Official SAT Online Course

2006-11-12file://E:\新建文件夹\f9.htm

UnRe

gistered

Page 105: SAT test 6

严禁用于商业用途!

Choice (A) is correct. "Multiplicity" means a great number. "Variety" means something varying from others of the same general kind. If one were to insert these terms into text, the sentence would read "In her writings about language, the poet Gloria Anzaldúa celebrates the multiplicity of English and Spanish dialects spoken by Mexican Americans, arguing that such variety lends an empowering flexibility to expression." The word "such" suggests that the two blanked words are synonyms, both describing a celebrated feature of Mexican American speech. "Multiplicity" and "variety" describe Mexican American language in the same way. Multiple and various forms of speech would also give Mexican Americans the "empowering flexibility of expression" Anzaldúa refers to.

Explanation for Incorrect Answer B :  Choice (B) is incorrect. “Proliferation” means to increase at a rapid rate. “Moderation” means being within reasonable limits. If one were to insert these terms into the text, the sentence would read “In her writings about language, the poet Gloria Anzaldúa celebrates the proliferation of English and Spanish dialects spoken by Mexican Americans, arguing that such moderation lends an empowering flexibility to expression.” A “proliferation,” or rapid increase, of dialects would not be described as a “moderation.” It is also unlikely that moderation could give someone flexibility of expression.

Explanation for Incorrect Answer C :  Choice (C) is incorrect. "Ambivalence" means uncertainty or indecisiveness. "Focus" means a center of interest or activity. If one were to insert these terms into text, the sentence would read "In her writings about language, the poet Gloria Anzaldúa celebrates the ambivalence of English and Spanish dialects spoken by Mexican Americans, arguing that such focus lends an empowering flexibility to expression." "Ambivalence," or conflicting feelings, would tend to take away from a person's focus. Furthermore, the two words are not synonyms.

Explanation for Incorrect Answer D :  Choice (D) is incorrect. "Dearth" means a lack of or scarce supply. "Depletion" means the act of depleting. If one were to insert these terms into text, the sentence would read "In her writings about language, the poet Gloria Anzaldúa celebrates the dearth of English and Spanish dialects spoken by Mexican Americans, arguing that such depletion lends an empowering flexibility to expression." While "dearth" and "depletion" both refer to a lack of something, it is illogical to state that a lack of ways of speaking would lend "flexibility to expression."

Explanation for Incorrect Answer E :  Choice (E) is incorrect. "Abridgment" means the act of reducing or shortening. "Imitation" means something derived or copied from an original. If one were to insert these terms into text, the sentence would read "In her writings about language, the poet Gloria Anzaldúa celebrates the abridgement of English and Spanish dialects spoken by Mexican Americans, arguing that such imitation lends an empowering flexibility to expression."An "abridgment" is a reduction or shortening of something, a concept that has little to do with either "imitation" or "flexibility of expression."

4

The mountain road was distinctly -------: it twisted back and forth along the contours of the hillside.

ANSWERS AND EXPLANATIONS Explanation for Correct Answer C :  Choice (C) is correct. "Serpentine" means winding. If one were to insert this term into the text, the sentence would read "The mountain road was distinctly serpentine: it twisted back and forth along the contours of the hillside." The colon indicates that the second part of the sentence will illustrate the meaning of the missing term. "Serpentine" fits the sentence because a winding road does indeed twist "back and forth."

(A) panoramic

(B) precipitous

(C) serpentine

(D) circumscribed

(E) retrograde

页码,3/16The Official SAT Online Course

2006-11-12file://E:\新建文件夹\f9.htm

UnRe

gistered

Page 106: SAT test 6

严禁用于商业用途!

Explanation for Incorrect Answer A :  Choice (A) is incorrect. "Panoramic" refers to a complete view of a surrounding area. If one were to insert this term into the text, the sentence would read "The mountain road was distinctly panoramic: it twisted back and forth along the contours of the hillside." A winding road will not necessarily offer "panoramic" views.

Explanation for Incorrect Answer B :  Choice (B) is incorrect. "Precipitous" means extremely steep. If one were to insert this term into the text, the sentence would read "The mountain road was distinctly precipitous: it twisted back and forth along the contours of the hillside." There is no indication that the winding road in the sentence is particularly steep.

Explanation for Incorrect Answer D :  Choice (D) is incorrect. "Circumscribed" refers to something that is constricted or surrounded by a boundary. If one were to insert this term into the text, the sentence would read "The mountain road was distinctly circumscribed: it twisted back and forth along the contours of the hillside." The statement after the colon does not indicate that the road is "circumscribed" in any way.

Explanation for Incorrect Answer E :  Choice (E) is incorrect. "Retrograde" means moving backward. If one were to insert this term into the text, the sentence would read "The mountain road was distinctly retrograde: it twisted back and forth along the contours of the hillside." It does not make sense to describe the road as moving in a backward direction.

5At the family reunion Hiroko found her cousin charming and gentle, the ------- of his formerly rude and overbearing self.

ANSWERS AND EXPLANATIONS Explanation for Correct Answer B :  Choice (B) is correct. "Antithesis" means exact opposite. If one were to insert this term into the text, the sentence would read "At the family reunion Hiroko found her cousin charming and gentle, the antithesis of his formerly rude and overbearing self." The sentence indicates that Hiroko's cousin has changed dramatically. He was once "rude and overbearing," and is now "charming and gentle." "Antithesis" connotes this dramatic change perfectly.

Explanation for Incorrect Answer A :  Choice (A) is incorrect. "Remnant" means a remainder. If one were to insert this term into the text, the sentence would read "At the family reunion Hiroko found her cousin charming and gentle, the remainder of his formerly rude and overbearing self." It makes no sense to say that "charming and gentle" qualitites would be the "remnant," or remainder, of a "formerly rude and overbearing self."

Explanation for Incorrect Answer C :  Choice (C) is incorrect. "Consequence" means the relation of a result to its cause. If one were to insert this term into the text, the sentence would read "At the family reunion Hiroko found her cousin "charming and gentle," the consequence of his formerly rude and overbearing self." It is illogical to claim that charming and gentle behavior could be a consequence of "rude and overbearing" behavior. This sentence suggests that Hiroko's cousin's behavior is different from what it once was, not that it is the result of what it once was.

Explanation for Incorrect Answer D :  Choice (D) is incorrect. "Extremity" means the farthest point. If one were to insert this term into the text, the sentence would read "At the family reunion Hiroko found

(A) remnant

(B) antithesis

(C) consequence

(D) extremity

(E) mainstay

页码,4/16The Official SAT Online Course

2006-11-12file://E:\新建文件夹\f9.htm

UnRe

gistered

Page 107: SAT test 6

严禁用于商业用途!

her cousin charming and gentle, the extremity of his formerly rude and overbearing self." Hiroko's cousin's new behavior is not an extreme expression of her cousin's former self, it is completely different.

Explanation for Incorrect Answer E :  Choice (E) is incorrect. "Mainstay" means a chief support or foundational influence or characteristic. If one were to insert this term into the text, the sentence would read "At the family reunion Hiroko found her cousin charming and gentle, the mainstay of his formerly rude and overbearing self." Hiroko's cousin was exhibiting characteristics that seemed new, not foundational, to how his cousin previously knew him.

6 His conduct at the state dinner was a cavalcade of blunders, one ------- following another until the evening ended.

ANSWERS AND EXPLANATIONS Explanation for Correct Answer C :  Choice (C) is correct. "Gaffe" means a blunder. If one were to insert this term into the text, the sentence would read "His conduct at the state dinner was a cavalcade of blunders, one gaffe following another until the evening ended." The comma indicates that the missing term is synonymous with blunder. Both "blunder" and "gaffe" can be used to refer to a social error.

Explanation for Incorrect Answer A :  Choice (A) is incorrect. A "query" is a question or inquiry. If one were to insert this term into the text, the sentence would read "His conduct at the state dinner was a cavalcade of blunders, one query following another until the evening ended." It is inaccurate to equate a blunder, or mistake, with a query.

Explanation for Incorrect Answer B :  Choice (B) is incorrect. "Gibe" means a derisive remark. If one were to insert this term into the text, the sentence would read "His conduct at the state dinner was a cavalcade of blunders, one gibe following another until the evening ended." The use of gibes at a formal dinner could certainly be considered to be a social faux pas, but a blunder does not always appear in the form of a taunt.

Explanation for Incorrect Answer D :  Choice (D) is incorrect. "Tryst" typically refers to a romantic meeting. If one were to insert this term into the text, the sentence would read "His conduct at the state dinner was a cavalcade of blunders, one tryst following another until the evening ended." The term "tryst" is not a synonym of "blunder."

Explanation for Incorrect Answer E :  Choice (E) is incorrect. A "tribute" is an acknowledgment of gratitude, respect, or admiration. If one were to insert this term into the text, the sentence would read "His conduct at the state dinner was a cavalcade of blunders, one tribute following another until the evening ended." It's not likely that a tribute at a state dinner would be described as a social error.

(A) query

(B) gibe

(C) gaffe

(D) tryst

(E) tribute

Both passages discuss the issue of the intelligence of dogs. Passage 1 was adapted from a 2001 book on animal intelligence. Passage 2 was written in 2001 by a dog trainer and writer.

Passage 1

       It was no accident that nineteenth-century

页码,5/16The Official SAT Online Course

2006-11-12file://E:\新建文件夹\f9.htm

UnRe

gistered

Page 108: SAT test 6

严禁用于商业用途!

naturalist

 Charles Darwin strove to connect the mentality and

 emotionality of people with that of dogs, rather than, say,

Linedoves or horses. Neither his theory of evolution nor any

5general understanding of biology demanded that he pref-

 erentially underline our similarity to dogs over other

 species. But politically and emotionally, the choice was

 inevitable for an English gentleman who had set himself

 the task of making the idea of evolutionary continuity

10palatable. Darwin wrote that “dogs possess something

 very similar to a conscience. They certainly possess

 some power of self-command. . . . Dogs have long been

 accepted as the very type of fidelity and obedience.”

      Darwin was not alone in his beliefs that dogs possess

15human virtues. The characteristics of loyalty and obedience,

 coupled with an expressive face and body, can account

 for why dogs are such popular and valued pets in many

 cultures. Depending on the breed and the individual, dogs

 can be noble, charming, affectionate, and reliable. But

20while all dog owners should rightly appreciate these and

 other endearing traits in their pets, nothing says that the

 cleverness of a highly intelligent primate such as a chim-

 panzee is part of the package. Scientists generally believe

 the reasoning abilities of chimps to be considerably greater

25than that of dogs. But many people nonetheless believe that

 dogs are smarter than chimps precisely because of our

  familiarity and emotional ties with the dogs that

页码,6/16The Official SAT Online Course

2006-11-12file://E:\新建文件夹\f9.htm

UnRe

gistered

Page 109: SAT test 6

严禁用于商业用途!

Passage 2

we love.

 We apply the same secret rules to our fellow humans: the

 old in-group, out-group story. People in your in-group are

30those who are similar to you, either because they belong

 to the same organizations as you, or enjoy the same

 activities, or, and this is the kicker, because they are simply

 around more often. Dogs, because of their proximity to

 their owners, are definitely in. The intensity of our

35relationship with dogs causes us, quite naturally, to imbue

 them with high-level mental abilities, whether they have

 earned those extra intelligence points or not. We like them,

  so we think well of them.

      Every dog trainer that I know had the same childhood, a

40childhood filled with the brilliant, heroic dogs of literature.

 We read about dogs who regularly traveled thousands of

 miles to be reunited with owners who somehow misplaced

 them, repeatedly saved people from certain death, and

 continually exhibited a better grasp of strategic problem-

45solving than the average Ph.D. In the preface to one of his

 many dog stories, S. P. Meek a bit shamefacedly remarked

 that in writing of dogs “I endeavored to hold these heroes

 down to the level of canine intelligence, and to make them,

 above all, believable. If at times I seem to have made them

50show supercanine intelligence, it is because my enthusiasm

 has run away with me.” We forgave him, of course.

      It was something of a shock, therefore, to discover

页码,7/16The Official SAT Online Course

2006-11-12file://E:\新建文件夹\f9.htm

UnRe

gistered

Page 110: SAT test 6

严禁用于商业用途!

 how the learning theory “experts” believed dogs think

 and learn. I was told that dogs, unlike chimpanzees, have

55no real reasoning ability. Dogs don’t think: rather, they

 learn to avoid the unpleasant (negative reinforcement), seek

 the pleasant (positive reinforcement), or some combination

 of the two. To contend otherwise was to be guilty of the sin

 of anthropomorphizing, the attribution to an animal of

60motivations and consciousness that only a human being

  could possess.

      Yet as a dog trainer, I find myself siding more with the

 Meeks than I do with the learning theorists: nobody could

 believe dispassionately in the totality of positive and nega-

65tive reinforcement after seeing the pure intelligence shining

 in the face of a border collie intent upon helping a shepherd

 herd sheep. Dogs do think and reason. Granted, a dog might

 not be able to run a maze as quickly as a chimp. But a dog

 outshines any other animal that I know in the ability to

70work willingly with a human being, to communicate with

 a puzzling creature who often makes incomprehensible

 demands. Researchers have increasingly come to view

 intelligence as a complex collection of mental abilities

 that cannot be fully captured in any simple way. Dogs

75are geniuses at being useful, and it is this usefulness

 that we admire when we praise their intelligence. As

 Jonica Newby, a specialist in animal-human interaction,

 writes, “In some ways intelligence is a matter of match-

ing behavior to environment. To compare

页码,8/16The Official SAT Online Course

2006-11-12file://E:\新建文件夹\f9.htm

UnRe

gistered

Page 111: SAT test 6

严禁用于商业用途!

  intelligence

80in creatures that have evolved differently is a bit like

 deciding which has hit upon the best mode of travel: the

 dolphin or the horse.” And it is dogs, not chimps, who

 possess the most helpful mode of travel for human beings.

7 Unlike the author of Passage 1, the author of Passage 2 develops an argument by relying on

ANSWERS AND EXPLANATIONS Explanation for Correct Answer A :  Choice (A) is correct. Passage 1 examines the issue of dog intelligence from a primarily objective perspective, while Passage 2 is infused with the author's first-hand experiences.

Explanation for Incorrect Answer B :  Choice (B) is incorrect. Passage 1 relies heavily on "scientific" evidence. Passage 2 does not.

Explanation for Incorrect Answer C :  Choice (C) is incorrect. Unlike the author of Passage 1, the author of Passage 2 provides little "historical" background information.

Explanation for Incorrect Answer D :  Choice (D) is incorrect. The author of Passage 2 does not provide "statistical evidence."

Explanation for Incorrect Answer E :  Choice (E) is incorrect. Both authors use direct quotations to develop their arguments.

(A) personal experience

(B) scientific observation

(C) historical contextualization

(D) statistical evidence

(E) direct quotation

8

The phrase “It was no accident” (line 1) implies that the author of Passage 1 believes that Darwin

ANSWERS AND EXPLANATIONS Explanation for Correct Answer B :  Choice (B) is correct. The author suggests that Darwin's example of animal intelligence was intentional. His readers' tendency to hold dogs in high regard made them more likely to accept his theory of evolutionary continuity.

(A) knew that the resemblance between dogs and humans could not be accounted for by his theory

(B) exploited the sympathies of his audience to gain support for his theory

(C) considered intelligence to be largely a matter of luck

(D) believed that the way previous authors had written about dogs was inaccurate

(E) wished to convince the public to celebrate the virtues of dogs

页码,9/16The Official SAT Online Course

2006-11-12file://E:\新建文件夹\f9.htm

UnRe

gistered

Page 112: SAT test 6

严禁用于商业用途!

Explanation for Incorrect Answer A :  Choice (A) is incorrect. Darwin's theory does in fact account for the similarities between dogs and humans.

Explanation for Incorrect Answer C :  Choice (C) is incorrect. The phrase "[i]t was no accident" has no connection to the question of whether or not intelligence is a matter of luck.

Explanation for Incorrect Answer D :  Choice (D) is incorrect. The author does not mention Darwin's opinions regarding previous depictions of dogs.

Explanation for Incorrect Answer E :  Choice (E) is incorrect. Darwin's focus is on supporting his argument, not on celebrating animals.

9 In line 13, “type” most nearly means

ANSWERS AND EXPLANATIONS Explanation for Correct Answer B :  Choice (B) is correct. Darwin describes dogs as "the very type of fidelity and obedience," implying that they are a "model," or ideal example, of these qualities.

Explanation for Incorrect Answer A :  Choice (A) is incorrect. Dogs may embody "fidelity and obedience," but it is illogical to claim that they are a "category," or division, of specific behavorial traits.

Explanation for Incorrect Answer C :  Choice (C) is incorrect. While Darwin implies that dogs exemplify certain traits, it would be illogical to claim that they are a "designation," or name, of "fidelity and obedience."

Explanation for Incorrect Answer D :  Choice (D) is incorrect. Dogs may have played a "role" in Darwin's theories regarding animal intelligence, but it would make little sense to say that they are the "very role" of certain behavioral traits.

Explanation for Incorrect Answer E :  Choice (E) is incorrect. The term "figure" is not typically used in this context.

(A) category

(B) model

(C) designation

(D) role

(E) figure

10The italics in line 25 most directly emphasize

ANSWERS AND EXPLANATIONS

(A) a misguided idea that is becoming obsolete

(B) a negative view that scientists adopt toward lay people

(C) a common criticism of dog owners

(D) the controversial nature of an alternative theory

(E) the intensity of a conviction based on sentiment

页码,10/16The Official SAT Online Course

2006-11-12file://E:\新建文件夹\f9.htm

UnRe

gistered

Page 113: SAT test 6

严禁用于商业用途!

Explanation for Correct Answer E :  Choice (E) is correct. Italics draw attention to the word "believe," emphasizing people's willingness to credit the dogs they love with a depth of intelligence that they may not possess.

Explanation for Incorrect Answer A :  Choice (A) is incorrect. Passage 1 suggests that the belief expressed in line 25 is not scientifically valid, or misguided. The italics in this sentence do not, however, indicate that public opinion on canine intelligence is changing or going away.

Explanation for Incorrect Answer B :  Choice (B) is incorrect. The author does not discuss the scientists' attitudes regarding the general public at any point in the passage.

Explanation for Incorrect Answer C :  Choice (C) is incorrect. The author alludes to dog owners' opinions but does not criticize them by placing the word "believe" in italics.

Explanation for Incorrect Answer D :  Choice (D) is incorrect. The use of italics does not indicate whether or not dog owners' beliefs are controversial.

11 In line 29, the author of Passage 1 uses the word “old” to suggest that the “story” is

ANSWERS AND EXPLANATIONS Explanation for Correct Answer A :  Choice (A) is correct. The author uses the word "old" in the phrase "the old in-group, out-group story" to suggest that this explanation is a familiar one.

Explanation for Incorrect Answer B :  Choice (B) is incorrect. The word "old" can certainly mean "historic" in certain contexts, but the author of passage 1 does not indicate that this "story" has historical significance of any kind.

Explanation for Incorrect Answer C :  Choice (C) is incorrect. This story is hardly "fictitious," or false; on the contrary, the author suggests that it accurately describes the bond between humans and dogs.

Explanation for Incorrect Answer D :  Choice (D) is incorrect. The author does not suggest in any way that the "story" referred to is "tiresome."

Explanation for Incorrect Answer E :  Choice (E) is incorrect. Something that is "outdated" is no longer current, but the author suggests that "the old in-group, out-group story" is just as useful in the present as it was in the past.

(A) familiar

(B) historic

(C) fictitious

(D) tiresome

(E) outdated

12 Darwin (lines 1-13, Passage 1) and Meek (lines 45-51, Passage 2) serve as examples of

(A) writers who had the courage to voice unpopular viewpoints

(B) researchers who conducted important studies on animal behavior

页码,11/16The Official SAT Online Course

2006-11-12file://E:\新建文件夹\f9.htm

UnRe

gistered

Page 114: SAT test 6

严禁用于商业用途!

ANSWERS AND EXPLANATIONS Explanation for Correct Answer D :  Choice (D) is correct. Darwin's descriptions of dogs' supreme "fidelity and obedience" and Meek's tendency to bestow a "'supercanine intelligence'" on the dogs in his stories support the claim that these writers idealize dog behavior.

Explanation for Incorrect Answer A :  Choice (A) is incorrect. Darwin's efforts to make "the idea of evolutionary continuity palatable" indicate that some of his theories may have been "unpopular." There is no indication, however, that Meek's viewpoints were not well received.

Explanation for Incorrect Answer B :  Choice (B) is incorrect. Although Darwin was a researcher, Passage 2 only indicates that Meek was a writer of "many dog stories."

Explanation for Incorrect Answer C :  Choice (C) is incorrect. There is no indication that either Darwin or Meek deliberately misled the public about canine intelligence, and nothing in these passages suggests that the men were malicious.

Explanation for Incorrect Answer E :  Choice (E) is incorrect. Darwin might qualify as an authority on animal intelligence, but Meek seems to be a fiction writer.

(C) people who maliciously publicized misleading information about dogs

(D) individuals whose writings reflect an idealized view of dog behavior

(E) scientists who were authorities on the issue of animal intelligence

13

In line 53, the author of Passage 2 uses quotation marks to

ANSWERS AND EXPLANATIONS Explanation for Correct Answer E :  Choice (E) is correct. Writers sometimes place quotation marks around a word or phrase to cast doubt on its accuracy. The learning theory experts' "shocking" assessment of canine intelligence prompts the author of passage 2 to diminish the credibility of the learning theorists by placing the word "experts" in quotation marks.

Explanation for Incorrect Answer A :  Choice (A) is incorrect. It can be argued that the author's use of quotation marks expresses disbelief or even annoyance, but there is no conveyance of an extreme emotion such as anger.

Explanation for Incorrect Answer B :  Choice (B) is incorrect. The use of quotation marks around the word "experts" is a sign of skepticism, not "respect."

Explanation for Incorrect Answer C :  Choice (C) is incorrect. The author may view the experts' theories with uncertainty, but there is no reason to question the straightforward use of the word "experts" in this context.

(A) express anger about a prevailing belief

(B) demonstrate respect for a certain group of scientists

(C) indicate uncertainty about the precise usage of a word

(D) cite a term used in an unusual context

(E) cast doubt on the aptness of a description

页码,12/16The Official SAT Online Course

2006-11-12file://E:\新建文件夹\f9.htm

UnRe

gistered

Page 115: SAT test 6

严禁用于商业用途!

Explanation for Incorrect Answer D :  Choice (D) is incorrect. It is not surprising to encounter the term "experts" within a discussion of a particular scientific theory.

14 The “experts” (line 53) would most likely argue that which of the following is guilty of the “sin” mentioned in line 58 ?

ANSWERS AND EXPLANATIONS Explanation for Correct Answer B :  Choice (B) is correct. This is a prime example of "anthropomorphizing," or attributing human feelings to animals.

Explanation for Incorrect Answer A :  Choice (A) is incorrect. The veterinarian's motivation for refusing to treat the animal would need to be specified to suggest anthropomorphization.

Explanation for Incorrect Answer C :  Choice (C) is incorrect. There is no indication that the dog owner is unwilling to punish her dog because she believes her dog has human feelings.

Explanation for Incorrect Answer D :  Choice (D) is incorrect. "A zoologist who places the interests of people before those of animals" would not be guilty of the sin mentioned in line 58 ("anthropomorphizing").

Explanation for Incorrect Answer E :  Choice (E) is incorrect. A horse trainer's inability to recognize that his horse is hungry has nothing to do with attributing human feelings to animals.

(A) A veterinarian who is unwilling to treat a sick animal

(B) A cat owner who believes his cat misses its siblings

(C) A dog owner who is unwilling to punish her dog for misbehaving

(D) A zoologist who places the interests of people before those of animals

(E) A horse trainer who fails to recognize that his horse is hungry

15Both the author of Passage 1 and the “experts” mentioned in line 53 of Passage 2 directly support the idea that

ANSWERS AND EXPLANATIONS Explanation for Correct Answer D :  Choice (D) is correct. The author of Passage 1 asserts that many people like to believe that dogs have "high-level mental abilities, whether they have earned those extra intelligence points or not." The "experts" mentioned in Passage 2 contribute to the argument that people exaggerate canine intelligence by claiming that dogs "have no real reasoning ability."

Explanation for Incorrect Answer A :  Choice (A) is incorrect. Passage 2 implies that Meek tried to keep his canine characters "believable," though he may not have always succeeded, and the author

(A) writers of dog stories intentionally distort the truth for dramatic purposes

(B) comparing the intelligence of dogs to that of chimps is a pointless enterprise

(C) many people have an excessive emotional attachment to their dogs

(D) dogs are less intelligent than many people believe

(E) few people are familiar with learning theory as it applies to dogs

页码,13/16The Official SAT Online Course

2006-11-12file://E:\新建文件夹\f9.htm

UnRe

gistered

Page 116: SAT test 6

严禁用于商业用途!

of Passage 1 does not mention fiction writers at all.

Explanation for Incorrect Answer B :  Choice (B) is incorrect. Both the author of Passage 1 and the "experts" cited in Passage 2 seem to find the comparison between "the intelligence of dogs" and "that of chimps" to be worthwhile.

Explanation for Incorrect Answer C :  Choice (C) is incorrect. Unlike the author of Passage 1, the "experts" do not mention the "emotional attachment" of people to dogs.

Explanation for Incorrect Answer E :  Choice (E) is incorrect. Although the "experts" cited in Passage 2 specialize in "learning theory," there is no mention of how familiar people are with their field of study.

16 Based on lines 63-67 (“nobody . . . sheep”), the author of Passage 2 would most likely appear to the author of Passage 1 as

ANSWERS AND EXPLANATIONS Explanation for Correct Answer C :  Choice (C) is correct. The author of Passage 1 concedes that "dogs can be noble, charming, affectionate, and reliable" but suggests that they may not have "earned those extra intelligence points." In comparison, the description of "pure intelligence shining in the face of a border collie" in lines 63–67 may be interpreted to be an overestimation based on the writer's fondness for dogs.

Explanation for Incorrect Answer A :  Choice (A) is incorrect. The author of Passage 1 would most likely consider the author of Passage 2 to be emotionally attached to dogs and therefore incapable of assessing their intelligence objectively.

Explanation for Incorrect Answer B :  Choice (B) is incorrect. Lines 63–67 reflect a subjective perspective on canine intelligence. They do not suggest that the author of Passage 2 is particularly knowledgeable about research on the topic.

Explanation for Incorrect Answer D :  Choice (D) is incorrect. The author of Passage 2 seems to eschew the results of scientific studies in favor of personal experience, but lines 63–67 do not reflect a disrespect for "traditional" research.

Explanation for Incorrect Answer E :  Choice (E) is incorrect. The two writers may have differing views regarding the extent to which dogs are capable of reasoning, but it would be extreme to claim that the author of Passage 2 has a limited comprehension "of what constitutes intelligence."

(A) a neutral observer of animal behavior

(B) well informed concerning research into animal intelligence

(C) having a deep fondness for border collies and therefore overestimating them

(D) having little respect for traditional scientific research

(E) having a narrow understanding of what constitutes intelligence

17 In Passage 2, lines 67-68 (“Granted . . . chimp”) principally serve to

(A) acknowledge a flaw in a prevalent theory

(B) digress from a primary claim

(C) evoke an air of mystery

(D) dismiss a scientific hypothesis as unfounded

页码,14/16The Official SAT Online Course

2006-11-12file://E:\新建文件夹\f9.htm

UnRe

gistered

Page 117: SAT test 6

严禁用于商业用途!

ANSWERS AND EXPLANATIONS Explanation for Correct Answer E :  Choice (E) is correct. In admitting that chimps are superior to canines in some traditional measures of intelligence, the author anticipates a possible objection to the argument that dogs possess an unmatched ability to work and communicate with people.

Explanation for Incorrect Answer A :  Choice (A) is incorrect. A chimp's ability to navigate a maze faster than a dog can does not contradict any theories mentioned in the passage.

Explanation for Incorrect Answer B :  Choice (B) is incorrect. A "digression" is a departure from the main argument in a text. Lines 67–68, however, are very much related to the author's claim that dogs possess a keen intelligence.

Explanation for Incorrect Answer C :  Choice (C) is incorrect. On the contrary, the author's use of concession in Passage 2 is meant to clarify the argument made in favor of canine intelligence.

Explanation for Incorrect Answer D :  Choice (D) is incorrect. Lines 67–68 make no attempt to discredit a scientific theory. Instead, they attempt to place the results of a specific scientific finding in context.

(E) anticipate a potential objection to an argument

18 The authors of both passages mention chimpanzees in order to

ANSWERS AND EXPLANATIONS Explanation for Correct Answer E :  Choice (E) is correct. The author of Passage 1 suggests that chimpanzees are more intelligent than dogs, while the author of Passage 2 argues that the two animals exhibit different types of advanced intelligence.

Explanation for Incorrect Answer A :  Choice (A) is incorrect. The author of Passage 2 does not discuss how "the public" views chimpanzee intelligence.

Explanation for Incorrect Answer B :  Choice (B) is incorrect. Both authors refer to chimpanzees' reasoning abilities, but neither discusses the emotions of primates.

Explanation for Incorrect Answer C :  Choice (C) is incorrect. The author of Passage 1 asserts that people have a generally flawed understanding of animal intelligence, and the author of Passage 2 doesn't discuss public opinion at all.

Explanation for Incorrect Answer D :  Choice (D) is incorrect. Neither author mentions chimpanzees in reference to "an eccentric scientific claim."

(A) suggest that the public has a distorted view of chimpanzee intelligence

(B) compare the emotions of primates to those of dogs

(C) justify the beliefs of the public regarding the intelligence of certain animals

(D) criticize an eccentric scientific claim about animal intelligence

(E) provide an example of an animal considered to be highly intelligent

页码,15/16The Official SAT Online Course

2006-11-12file://E:\新建文件夹\f9.htm

UnRe

gistered

Page 118: SAT test 6

严禁用于商业用途!

19 How do the authors of the two passages differ in their assumptions about animal intelligence?

ANSWERS AND EXPLANATIONS Explanation for Correct Answer A :  Choice (A) is correct. The author of Passage 1 seems to recognize a single form of intelligence by which all animals may be evaluated. The author of Passage 2, however, argues that dogs exhibit an intelligence that is different from that of chimpanzees, but no less useful.

Explanation for Incorrect Answer B :  Choice (B) is incorrect. Both authors clearly believe that animals can possess intelligence. The author of Passage 1 alludes to the "cleverness of a highly intelligent primate."

Explanation for Incorrect Answer C :  Choice (C) is incorrect. At no point does the author of Passage 2 delve into the ethics of measuring animal intelligence.

Explanation for Incorrect Answer D :  Choice (D) is incorrect. Neither author directly discusses assumptions about whether intelligence is "innate" or "acquired."

Explanation for Incorrect Answer E :  Choice (E) is incorrect. Neither author argues that intelligence is a static or dynamic trait in animals.

(A)The author of Passage 1 implies that intelligence is a single entity, whereas the author of Passage 2 suggests that intelligence can be demonstrated in many distinct ways.

(B)The author of Passage 1 believes that no animal can be considered truly intelligent, whereas the author of Passage 2 celebrates the reasoning power of all animals.

(C)The author of Passage 1 believes that intelligence can be measured, whereas the author of Passage 2 believes that such quantification would be unethical.

(D) The author of Passage 1 suggests that intelligence is innate, whereas the author of Passage 2 argues that it is acquired.

(E) The author of Passage 1 considers intelligence to be developed over time, whereas the author of Passage 2 shows that it is largely static.

    

Back to Score Report  

Copyright © 2006 The College Board. All rights reserved. Privacy Policy Terms of Use Contact Us

 

页码,16/16The Official SAT Online Course

2006-11-12file://E:\新建文件夹\f9.htm

UnRe

gistered

Page 119: SAT test 6

严禁用于商业用途!

Help | Profile | My Organizer | My Bookmarks | Logout

Answers and Explanations

Test Sections

Section 1

Section 2

Section 4

Section 5

Section 6

Section 7

Section 8

Section 9

Section 10

Back to Score Report  

View Answers and Explanations     Online - Practice Test #6

1 The sales assistant arranged the gems on the counter, he proceeded to tell us about the origins of each stone.

ANSWERS AND EXPLANATIONS Explanation for Correct Answer D :  Choice (D) is correct. It avoids the error of the original by reducing the first complete thought to a phrase.

Explanation for Incorrect Answer A :  Choice (A) involves improper coordination. Two complete thoughts ("The sales...counter” and "he...stone") are joined by only a comma.

Explanation for Incorrect Answer B :  Choice (B) creates a fragment. There is no main verb in the sentence.

Explanation for Incorrect Answer C :  Choice (C) creates an illogical sentence. It is illogical that "gems" "proceeded to tell us" something.

Explanation for Incorrect Answer E :  Choice (E) involves an error in pronoun use. The pronoun "he" is unnecessary.

(A) The sales assistant arranged the gems on the counter, he

(B) The gems, which were arranged on the counter by the sales assistant, who

(C) The gems were first arranged on the counter by the sales assistant, then

(D) After arranging the gems on the counter, the sales assistant

(E) The sales assistant, having arranged the gems on the counter, he

2 A whistle-blower is when an employee reports fraud or mismanagement in a company.

ANSWERS AND EXPLANATIONS Explanation for Correct Answer B :  Choice (B) is correct. It avoids the error of the original by correctly referring to a whistle-blower as "an employee."

Explanation for Incorrect Answer A :  Choice (A) creates an illogical sentence. A whistle-blower is a what, not a when.

(A) when an employee reports fraud or mismanagement

(B) an employee who reports fraud or mismanagement

(C) reporting by an employee of fraud or mismanagement

(D) if an employee reports fraud or mismanagement

(E) fraud or mismanagement being reported by an employee

页码,1/8The Official SAT Online Course

2006-11-12file://E:\新建文件夹\f10.htm

UnRe

gistered

Page 120: SAT test 6

严禁用于商业用途!

Explanation for Incorrect Answer C :  Choice (C) creates an illogical sentence. A whistle-blower is a person, not "reporting."

Explanation for Incorrect Answer D :  Choice (D) creates an illogical sentence. A whistle-blower is a what, not an "if."

Explanation for Incorrect Answer E :  Choice (E) creates an illogical sentence. It calls a whistle-blower "fraud or mismanagement."

3 After Eliza, the heroine of Shaw’s Pygmalion, is transformed from a flower girl into a gentlewoman, she realizes that one’s social class matters less than your character.

ANSWERS AND EXPLANATIONS Explanation for Correct Answer B :  Choice (B) is correct. It avoids the error of the original by using the pronoun "one" consistently.

Explanation for Incorrect Answer A :  Choice (A) involves a pronoun shift. The pronoun changes from "one" to second-person "your" for no reason.

Explanation for Incorrect Answer C :  Choice (C) creates a fragment. There is no main verb in the sentence.

Explanation for Incorrect Answer D :  Choice (D) creates a fragment. There is no main verb in the sentence.

Explanation for Incorrect Answer E :  Choice (E) involves an error in pronoun reference. "There" does not stand for anything in the sentence.

(A) she realizes that one’s social class matters less than your

(B) she realizes that one’s social class matters less than one’s

(C) then realizing that one’s social class matters less than their

(D) having realized how social class matters less than

(E) there is her realization about how social class matters less than

4Knowing the roots of words that are hard to spell helps students to become a better speller.

ANSWERS AND EXPLANATIONS Explanation for Correct Answer C :  Choice (C) is correct. It avoids the error of the original by using plural nouns consistently ("students," "spellers").

Explanation for Incorrect Answer A : 

(A) helps students to become a better speller

(B) is helpful to students who want to be a better speller

(C) helps students to become better spellers

(D) is helpful to students in becoming a better speller

(E) helps a student be better spellers

页码,2/8The Official SAT Online Course

2006-11-12file://E:\新建文件夹\f10.htm

UnRe

gistered

Page 121: SAT test 6

严禁用于商业用途!

Choice (A) involves an error in agreement. The singular noun "a better speller" cannot correctly refer to the plural noun "students."

Explanation for Incorrect Answer B :  Choice (B) involves an error agreement. The singular noun "a better speller" cannot correctly refer to the plural noun "students."

Explanation for Incorrect Answer D :  Choice (D) involves an error in agreement. "Students" cannot logically become "a better speller."

Explanation for Incorrect Answer E :  Choice (E) involves an error in agreement. The plural noun "better spellers" cannot correctly refer to the singular noun "a student."

5 Most experts believe that young children’s not being given physical affection, this interferes with their normal development.

ANSWERS AND EXPLANATIONS Explanation for Correct Answer E :  Choice (E) is correct. It avoids the error of the original by not separating the subject ("the withholding...children") from its verb ("interferes").

Explanation for Incorrect Answer A :  Choice (A) involves incorrect punctuation. The comma unnecessarily separates the subject ("young...affection") from its verb ("interferes").

Explanation for Incorrect Answer B :  Choice (B) involves unclear pronoun reference. What the pronoun "it" refers to is not clear.

Explanation for Incorrect Answer C :  Choice (C) involves improper use of a definite article. The phrase "the failure" suggests that the failure has been referred to previously, but it has not.

Explanation for Incorrect Answer D :  Choice (D) involves improper phrasing. The verb "withholding" has no subject.

(A) young children’s not being given physical affection, this interferes

(B) for young children who have had physical affection withheld from them, it interferes

(C) the failure at giving young children physical affection would interfere

(D) when withholding physical affection from young children, it interferes

(E) the withholding of physical affection from young children interferes

6Electronic bulletin boards, combining the convenience of a telephone with the massive information storage capacity of a computer, present messages on diverse subjects as astronomy, artificial intelligence, and skydiving.

ANSWERS AND EXPLANATIONS Explanation for Correct Answer D :  Choice (D) is correct. It avoids the error of the original by using the idiomatic "as . . . as" construction to indicate the variety of subjects addressed on electronic bulletin

(A) diverse subjects as

(B) diverse subjects that are

(C) subjects of such diversity as

(D) subjects as diverse as

(E) a subject as diverse as

页码,3/8The Official SAT Online Course

2006-11-12file://E:\新建文件夹\f10.htm

UnRe

gistered

Page 122: SAT test 6

严禁用于商业用途!

boards.

Explanation for Incorrect Answer A :  Choice (A) involves improper phrasing. "As" requires "such" before it.

Explanation for Incorrect Answer B :  Choice (B) involves improper phrasing. "That are" should be "such as."

Explanation for Incorrect Answer C :  Choice (C) involves an improper idiom. "Such diversity as" should be "diversity such as."

Explanation for Incorrect Answer E :  Choice (E) creates an error in agreement (in number). The phrase "a subject" cannot refer to the multiple subjects named.

7 Free from British rule after the American Revolution, a strong central government was an idea that many of the representatives attending the Constitutional Convention were wary of.

ANSWERS AND EXPLANATIONS Explanation for Correct Answer C :  Choice (C) is correct. It avoids the error of the original by making clear that it was "the representatives" who were "free from British rule."

Explanation for Incorrect Answer A :  Choice (A) creates an illogical sentence. "A strong central government" is not what had become "free from British rule."

Explanation for Incorrect Answer B :  Choice (B) creates an illogical sentence. "The idea of a strong central government" is not what had become "free from British rule."

Explanation for Incorrect Answer D :  Choice (D) involves an improper preposition. One is wary of something, not toward something.

Explanation for Incorrect Answer E :  Choice (E) creates a fragment. There is no main verb in the sentence.

(A) a strong central government was an idea that many of the representatives attending the Constitutional Convention were wary of

(B) the idea of a strong central government made wary many of the representatives attending the Constitutional Convention

(C) many of the representatives attending the Constitutional Convention were wary of a strong central government

(D) many representatives at the Constitutional Convention felt wary toward a strong central government

(E) many representatives at the Constitutional Convention, wary of a strong central government

8 Being cleaner and longer-burning compared with bituminous coal, anthracite was the first coal widely used in the United States for both domestic and industrial purposes.

(A) Being cleaner and longer-burning compared with

(B) Both cleaner and more longer-burning compared to

(C) Cleaner and longer-burning than

(D) By burning longer and more clean than

(E) Cleaner as well as longer-burning, unlike

页码,4/8The Official SAT Online Course

2006-11-12file://E:\新建文件夹\f10.htm

UnRe

gistered

Page 123: SAT test 6

严禁用于商业用途!

ANSWERS AND EXPLANATIONS Explanation for Correct Answer C :  Choice (C) is correct. It avoids the error of the original by correctly completing the comparison with the word "than."

Explanation for Incorrect Answer A :  Choice (A) involves improper comparison. "Compared with" should be "than."

Explanation for Incorrect Answer B :  Choice (B) involves improper comparison. "Compared to" should be "than."

Explanation for Incorrect Answer D :  Choice (D) involves improper modification. To modify the verb "burning," "clean" would have to be "cleanly."

Explanation for Incorrect Answer E :  Choice (E) involves improper comparison. It lacks "than" and adds the unnecessary word "unlike."

9 At graduation, the speaker assured us that our many courses in the liberal arts had prepared us equally well for the challenges of working and further study.

ANSWERS AND EXPLANATIONS Explanation for Correct Answer B :  Choice (B) is correct. It avoids the error of the original by making the two items joined by "and" parallel. "Work" and "further study" are both nouns.

Explanation for Incorrect Answer A :  Choice (A) involves an error in parallelism. The two items joined by "and" should be parallel, so "working" should be the noun "work."

Explanation for Incorrect Answer C :  Choice (C) involves an error in agreement. The singular verb "has supplied" does not agree with its plural subject, "our many courses in the liberal arts."

Explanation for Incorrect Answer D :  Choice (D) involves an error in agreement. The singular verb "leaves" does not agree with its plural subject, "our many courses in the liberal arts."

Explanation for Incorrect Answer E :  Choice (E) involves awkward phrasing. "Were the preparation for making" should simply be "prepared us for."

(A) had prepared us equally well for the challenges of working and further study

(B) had prepared us equally well for the challenges of work and of further study

(C) has supplied the preparation for challenging work along with further study

(D) leaves us prepared for the challenges of work and further study both

(E) were the preparation for making the challenges of work or further study easier

10 Modern bluegrass songs, telling of love and despair and celebrating mountain beauty, reflect the genre’s rural origins.

(A) Modern bluegrass songs, telling of love and despair and celebrating mountain beauty,

Modern bluegrass songs through their telling of love and despair and

页码,5/8The Official SAT Online Course

2006-11-12file://E:\新建文件夹\f10.htm

UnRe

gistered

Page 124: SAT test 6

严禁用于商业用途!

ANSWERS AND EXPLANATIONS Explanation for Correct Answer A :  Choice (A) is correct. It uses correct verb forms and punctuation.

Explanation for Incorrect Answer B :  Choice (B) involves an error in punctuation. The phrase "through...beauty" should be enclosed in commas.

Explanation for Incorrect Answer C :  Choice (C) creates an illogical sentence. It suggests that "because modern bluegrass songs tell of love," the songs are therefore rural.

Explanation for Incorrect Answer D :  Choice (D) involves unclear pronoun reference. What the pronoun "they" refers to is not clear.

Explanation for Incorrect Answer E :  Choice (E) uses an improper verb form. "Celebrating" should be "celebrate."

(B) celebrating mountain beauty,

(C) Because modern bluegrass songs tell of love and despair and also celebrating mountain beauty, they

(D) With modern bluegrass songs that tell of love and despair and celebrate mountain beauty, they

(E) Telling of love and despair, modern bluegrass songs celebrating mountain beauty, and they also

11 The fruit fly is often used to study genetic mechanisms, because it reproduces rapidly scientists can observe the effects of experiments on several generations.

ANSWERS AND EXPLANATIONS Explanation for Correct Answer D :  Choice (D) is correct. It avoids the error of the original by correctly joining two complete thoughts ("The fruit fly...mechanisms" and "because...generations") with a semicolon.

Explanation for Incorrect Answer A :  Choice (A) involves improper coordination. Two complete thoughts ("The fruit fly...mechanisms" and "because...generations") are joined by only a comma.

Explanation for Incorrect Answer B :  Choice (B) involves improper coordination. Two complete thoughts ("The fruit fly...mechanisms" and "since...generations") are joined by only a comma.

Explanation for Incorrect Answer C :  Choice (C) involves incorrect punctuation. The subordinating work "since" should not be preceded by a comma.

Explanation for Incorrect Answer E :  Choice (E) involves improper phrasing. "Allows" should be followed by "to," not by "can."

(A) mechanisms, because it reproduces rapidly

(B) mechanisms, since it reproduces rapidly,

(C) mechanisms, since, with its rapid reproduction,

(D) mechanisms; because it reproduces rapidly,

(E) mechanisms; then rapid reproduction allows

页码,6/8The Official SAT Online Course

2006-11-12file://E:\新建文件夹\f10.htm

UnRe

gistered

Page 125: SAT test 6

严禁用于商业用途!

12 Benin was the first sub-Saharan African country to experience a “civilian coup”: they were a regime that was dominated by the armed forces and obliged by citizens to implement democratic reforms.

ANSWERS AND EXPLANATIONS Explanation for Correct Answer E :  Choice (E) is correct. It avoids the error of the original by using the singular verb "was" to agree with the singular noun "regime."

Explanation for Incorrect Answer A :  Choice (A) involves an error in agreement. The plural pronoun "they" cannot correctly refer to the singular noun "Benin."

Explanation for Incorrect Answer B :  Choice (B) involves an error in agreement. The plural pronoun "they" cannot correctly refer to the singular noun "Benin."

Explanation for Incorrect Answer C :  Choice (C) involves an error in agreement. The plural verb "were" cannot agree with the singular subject "Benin."

Explanation for Incorrect Answer D :  Choice (D) involves an error in agreement. The plural pronoun "them" cannot correctly refer to the singular noun "Benin."

(A) they were a regime that was dominated by the armed forces and obliged by

(B) they had been a regime that was dominated by the armed forces, when they were obliged to

(C) it had a regime, armed forces dominating, but then were obliged to

(D) armed forces dominated them until this regime were obliged by

(E) a regime, dominated by the armed forces, was obliged by

13This legend about Admiral Nelson, like other naval heroes, are based only partially on fact.

ANSWERS AND EXPLANATIONS Explanation for Correct Answer E :  Choice (E) is correct. It avoids the error of the original by comparing "this legend" to "legends about other naval heroes."

Explanation for Incorrect Answer A :  Choice (A) involves an illogical comparison. It compares "this legend" to "other naval heroes."

Explanation for Incorrect Answer B :  Choice (B) involves an error in agreement. It uses the plural verb "are" for the singular subject "this legend."

Explanation for Incorrect Answer C :  Choice (C) involves an illogical comparison. It compares "this legend" to "other

(A) like other naval heroes, are

(B) like those of other naval heroes, are

(C) like other naval heroes, is

(D) like legends about other naval heroes, are

(E) like legends about other naval heroes, is

页码,7/8The Official SAT Online Course

2006-11-12file://E:\新建文件夹\f10.htm

UnRe

gistered

Page 126: SAT test 6

严禁用于商业用途!

naval heroes."

Explanation for Incorrect Answer D :  Choice (D) involves an error in agreement. It uses the plural verb "are" for the singular subject "this legend."

14 Bats and mosquitoes come out at twilight, and the bats would look for mosquitoes and the mosquitoes would look for people.

ANSWERS AND EXPLANATIONS Explanation for Correct Answer E :  Choice (E) is correct. It avoids the error of the original by using consistent verb forms ("to look").

Explanation for Incorrect Answer A :  Choice (A) involves an error in verb tense. It shifts from present tense ("come") to past tense ("would look") for no reason.

Explanation for Incorrect Answer B :  Choice (B) involves an error in verb form. It shifts from "come to look" to "look" for no reason.

Explanation for Incorrect Answer C :  Choice (C) involves an error in verb form. It shifts from "look" to "are looking" for no reason.

Explanation for Incorrect Answer D :  Choice (D) involves an error in verb form. The participle "looking" is an incomplete verb.

(A) and the bats would look for mosquitoes and the mosquitoes would look

(B) and the bats come to look for mosquitoes while the mosquitoes look

(C) the bats look for mosquitoes and the mosquitoes are looking

(D) the bats looking for mosquitoes while mosquitoes would look

(E) the bats to look for mosquitoes and the mosquitoes to look

  

Back to Score Report  

Copyright © 2006 The College Board. All rights reserved. Privacy Policy Terms of Use Contact Us

 

页码,8/8The Official SAT Online Course

2006-11-12file://E:\新建文件夹\f10.htm

UnRe

gistered